SlideShare a Scribd company logo
1 of 299
Download to read offline
A. CARBOHYDRATES
1.

Describe and classify carbohydrates
• Description
• Contain C, H and O molecules
• Contain a C=O (ketone) and an –OH(aldehyde) functional group
•

Classification
• Based on certain properties
• The size of the base carbon chain
• Location of the CO functional group
• Number of sugar units
• Stereochemistry of compound

•

Chemical Properties
• Some ( not all ) carbs are reducing substances (donate electrons)
• Chemical reduction of other substances
• These sugars must contain an aldehyde or ketone group
• Reducing sugars
o Glucose
o Maltose
o Lactose
o Fructose
o Galactose
Sucrose is not a reducing substance

•
2.

Describe carbohydrate metabolism
• Glucose is primary energy source
• Nervous tissue cannot concentrate or store carbohydrates, so a steady supply of
glucose is needed
• Once the level of glucose falls below a certain range, normal function is impaired
• Carbohydrate Breakdown Ultimate Goal
o Convert glucose to CO2 and water with ATP as a by-product

3.

Discuss glycolysis as it pertains to carbohydrate metabolism and carbohydrate
detection methods
• Hydrolysis of glucose by an enzyme into pyruvate or lactate;
• This process is anaerobic

4.

Fasting blood glucose levels
• Hyperglycemic
o Fasting blood glucose > 100 mg/dL
• Hypoglycemic
o Fasting blood glucose < 50 mg/dL

Page 1
5.

Describe glycolysis
• Glycolysis – the conversion of glucose and other hexoses into lactate or pyruvate
• Breakdown of glucose for energy production

6.

Describe carbohydrate breakdown

•

•

7.

Ultimate Goal
o Convert glucose to CO2 and water with ATP as a by-product
o Possible channels
o Converted to liver glycogen and stored
o Metabolized to CO2 and H2O
o Converted to keto-acids, amino acids, and proteins
o Converted to fats and stored in adipose tissue
Biochemical pathways in carbohydrate breakdown
o Embden-Meyerhoff pathway
o Converts glucose to pyruvate/lactate
o Primary energy source for humans
o Hexose monophosphate shunt
o Oxidizes glucose to ribose and CO2
o Produces NADPH as an energy source
o Glycogenesis
o Converts glucose to glycogen

Describe the role of the liver in maintenance of glucose levels.
•

The liver maintains the glucose levels by:
o Glycogenesis
 Converts glucose to glycogen
Page 2
o

o

•

Glycogenolysis –
 Breakdown of glycogen to form glucose
 Glycogenolysis occurs when plasma glucose is decreased
 Occurs quickly if additional glucose is needed
 Controlled by hormones & enzymes
Gluconeogenesis
 Formation of glucose from non-carbohydrate sources, such as
amino acids, glycerol & fatty acids into glucose
• Occurs mainly in the liver
 During long fasts, gluconeogenesis is required to maintain blood
glucose levels because glycogen stores are up in about 24 hours

During a fast, the blood glucose level is kept constant by mobilizing the glycogen
stores in the liver
Page 3
8.

What hormones does the liver use to maintain glucose levels?
•

Insulin
o Produced by the beta cells of the islets of Langerhans in the pancreas
o Promotes the entry of glucose into liver, muscle, and adipose tissue to be
stored as glycogen and fat;
o Inhibits the release of glucose from the liver
o Insulin secretion controlled by:
 Blood glucose level
 Certain Amino Acids ie. leucine, & arginine

•

Glucagon
o Secreted by the alpha cells of the pancreatic islets of Langerhans
o Increases blood glucose by stimulating glycogenolysis and gluconeogenesis
o 2nd most important glucose regulatory hormone
o Referred to as a hyperglycemic agent
o Synthesized in alpha cells of the islets of Langerhans
o Action/Effect of
o Stimuli – decreased plasma glucose
o Action
 Increases glycogenolysis & gluconeogenesis
 Promotes breakdown of fatty acids
 Promotes breakdown of proteins to form amino acids
 Increases plasma glucose concentration

•

Somatostatin
o Origin-Delta cells of the islets of Langerhans in the pancreas
o Effect - increase plasma glucose
o Actions
 antagonistic to insulin,
Page 4
o
•

9.

 inhibits endocrine hormones including glucagon & growth hormone
Inhibits secretion of insulin, glucagon, and growth hormone resulting in an
increase in plasma glucose levels

Other regulatory hormones
o Epinephrine
 One of two glucose regulating hormones from the adrenal gland
 Origin – adrenal medulla
 Action/effect
• Inhibits insulin secretion & release
• Promotes lipolysis
• Stimulates glycogenolysis
• Immediate release of glucose
 Stimuli
• Neurogenic - based on physical / emotional stress.
• Adrenal tumors
o Glucocorticoids - such as cortisol
 Origin – adrenal cortex
 Effect – antagonistic to insulin
• increases blood glucose
• promotes gluconeogenesis from breakdown of proteins
• inhibits the entry of glucose into muscle cells
 Stimuli – anterior pituitary’s ACTH
o Growth Hormone (GH) and Adrenocorticotropic Hormone (ACTH)
 Origin – anterior pituitary gland
 Effect – antagonistic to insulin
• Increases plasma glucose levels
• inhibits insulin secretion
• inhibits entry of glucose into muscle cells
• inhibits glycolysis
• inhibits formation of triglycerides from glucose
 Stimuli
• decreased glucose stimulates its release
• increased glucose inhibits its release
o Thyroid hormones (such as thyroxine)
 Origin – thyroid gland
 Effect
• Increases absorption of glucose from intestines
• Promotes conversion of liver glycogen to glucose
o Stimuli – pituitary gland’s TSH

What hormones does the pancreas produce that regulate carbohydrate
metabolism?
•

Insulin
o Produced by the beta cells of the islets of Langerhans in the pancreas
o Promotes the entry of glucose into liver, muscle, and adipose tissue to be
stored as glycogen and fat;
Page 5
•
•

10.

o Inhibits the release of glucose from the liver
Glycagon
o Secreted by the alpha cells of the pancreatic islets of Langerhans
o Increases blood glucose by stimulating glycogenolysis and gluconeogenesis
Somatostatin
o Synthesized by the delta cells of the pancreatic islets of Langerhans
o Inhibits secretion of insulin, glucagon, and growth hormone resulting in an
increase in plasma glucose levels

What impact does cortisol, catecholamine hormones and thyroid hormones have
on glucose levels.
• Cortisol
o
o
o

Secreted by the adrenal glands;
Stimulates glycogenolysis, lipolysis, and gluconeogenesis
Increases plasma glucose by decreasing intestinal entry into cells and
increasing gluconeogenesis.
• Epinephrine
o Increases plasma glucose by inhibiting insulin secretion.
o Secreted by the medulla of the adrenal glands.
o It stimulates glycogenolysis and lipolysis;
o It inhibits secretion of insulin.
o Physical or emotional stress causes increased secretion of
epinephrine and an immediate increase in blood glucose levels.
• Thyroid hormone
o Secreted by the thyroid gland;
o Stimulates glycogenolysis and gluconeogenesis;
o Increases glucose absorption from the intestines

11.

Describe the metabolic defect in Diabetes Mellitus?
• Glucose does not get into the cells

12.

What are typical glucose levels, insulin levels, and ketone levels in this disease?
1) Type I Diabetes Mellituso Glucose levels are increased
o Insulin levels are decreased
o Ketones present
2) Type II Diabetes Mellitus
o Glucose levels are increased
o Insulin levels are normal to decreased
o Glucagon response is decreased
o No ketones present.

13.

What is glycosylated hemoglobin?
•

Glycated hemoglobin is formed from the nonenzymatic, irreversible attachment of
glucose to hemoglobin A1.
Page 6
•
•
•
14.

Measurement of glycated hemoglobin reflects blood glucose levels for the past 2– 3
months.
It is useful in monitoring effectiveness of treatment and compliance of diabetic
individual to treatment protocol.
The primary determinant in the rate of hemoglobin A-1c synthesis is the life span of
the Red Blood Cell and the level of average glucose concentration

What are the normal glucose levels in a fasting individual?
• Normal glucose level (fasting)- 70 to 110 mg/dl

15.

What are panic or critical glucose values?
• Panic Values- >126 (fasting), > 200 (random or glucose tolerance test)

16.

Describe the relationship between glucose levels in urine and serum.
•
•
•
•
•

Glucose will never be found in urine unless the serum glucose is high enough to
exceed the renal threshold and spill over into the urine
Glucose is filtered by the glomeruli, reabsorbed by the tubules, and normally not
present in urine.
If the blood glucose level is elevated, glucose appears in the urine, a condition
known as glucosuria.
An individual’s renal threshold for glucose varies between 160 and 180 mg/ dL.
When blood glucose reaches this level or exceeds it, the renal tubular transport
mechanism becomes saturated, which causes glucose to be excreted into the
urine.

17.

Name inherited disorder of carbohydrate metabolism
• Glucose 6 phosphatase deficiency AKA von Glerke
• Galactosemia
o Characterized by a deficiency or absence of galactokinase,
o Enzyme defect prevents metabolism of galactose.
o Galactose is found in milk as a component of lactose, with galactosemia
generally identified in infants.

18.

Why should serum for glucose be removed from the red cells as soon after
collection as possible?
•

19.

Serum should be removed from RBC’s ASAP because the cells will use the
glucose and falsely decrease the glucose level.

What anticoagulant preservatives are used for glucose specimens? Why?
•

Sodium fluoride is the anticoagulant of choice because it inhibits glycolytic
enzymes.

Page 7
20.

What is a two-hour post prandial glucose? Why is it performed?
•
•

21.

Why are D-xylose tolerance tests performed?
•

22.

This test is done to differentiate malabsorption from pancreatic insufficiency.

What is the normal range of CSF glucose?
•
•
•
•

23.

This is a sample taken two hours after eating and can determine how well the body
is using the glucose.
This sample will show insulin function which is the main reason it is performed.

Normal CSF glucose- 40-70 mg/dl
CSF glucose is 2/3 of the plasma glucose which is due to the glucose entering the
CSF by facilitative transport.
The carrier mechanism is responsible for transporting glucose across the downhill
gradient.
Meningitis can cause a change in the CSF glucose levels.

Describe states that result in alteration of serum glucose.
•
•
•
•

Diabetes mellitus
PancreatitisRecent meal intake
Fasting

Increases glucose (insulin is not working)
Increases glucose (insulin not produced)
Increases glucose
Decreases glucose

24.

Lactate
• The normal end product of glucose metabolism is pyruvate;
• Lactate is produced under conditions of oxygen deficit (anaerobic metabolism).
• The production and accumulation of lactate in the blood and its measurement aid in
assessing the degree of oxygen deprivation that is occurring.
• Change in the blood lactate level precedes a change in blood pH.
• Lactate is metabolized by the liver via gluconeogenesis.

25.

What are normal glycosylated hemoglobin ranges?
•
•

Normal glycosylated hemoglobin range
o 4.5-8.0
High values indicate that the patient has not been following the proper diet

Page 8
Page 9
B. LIPIDS
1.

Describe the cholesterol
• Function
o Used to manufacture and repair cell membranes,
o Used in synthesis of bile acids and vitamin D
• Synthesized in the liver and obtained from the diet.
• Precursor for synthesis of bile acids, steroid hormones, and vitamin D
• Transport mechanism
o Transported through the blood by LDL (low density lipoproteins)
• Storage sites
o Stored in the skin, adipose tissue, and muscle cells.
• Esterified cholesterol-2/3 of the total cholesterol is esterified

2.

Describe triglycerides
• Formed from one glycerol molecule with three fatty acid molecules attached via
ester bonds
• Comprises 95% of all fats stored in adipose tissue
• Transport mechanism
• Transported through the body by chylomicrons and VLDL

3.

HDL-cholesterol
• Synthesized by the liver and by the intestine
• In normal lipid metabolism, HDL removes excess cholesterol from peripheral
tissues and transports it to other catabolic sites providing an antiatherogenic effect..

4.

What is the role of lipase?
• Lipase and bile acids are used to break down fats in lipid absorption.
• Found in pancreas, with lesser amounts in gastric mucosa, intestinal mucosa,
adipose tissue
• Clinical significance:
o Increased serum levels in acute pancreatitis occur in 4– 8 hours after the
onset of pain, with peak values in 24 hours, and return to normal in 8– 14
days.

5.

List the bile acids
• Cholic acid,
• Glycocholic acid
• Taurocholic acid

6.

Where are bile acids synthesized?
• Liver

7.

What does the presence of bile acids in serum indicate?
• . The presence of bile acids in serum can indicate liver disease.

Page 10
8.

What are chylomicrons?
• Triglycerides are transported through the body by chylomicrons
• Chylomicrons- the largest lipoprotein particles with diameters ranging from 80-1200
nm. They are 90-95% triglycerides, 2-6% phospholipids, 2-4% cholesteryl ester,
1% free cholesterol, and 1-2% apolipoprotein.
•

9.

How does serum appear with increased chylomicrons? increased cholesterol?
increased triglycerides?
• Increased Chylomicrons- a creamy layer appears on top of the serum.
• Increased Cholesterol- serum appears milky white
• Increased Triglycerides- serum appears turbid.

10.

Given a value of 38 mg/dl for the HDL, 140 for triglycerides and 210 for total
cholesterol, calculate the LDL and VLDL.
•
•
•
•

VLDL= triglycerides/5
LDL= total cholesterol- VLDL- HDL
VLDL= 140/5 =28
LDL= 210-28-38= 144

C. LIVER FUNCTION AND HEME-DERIVATIVES
1.

Describe liver function in reference to each of the following:
3) Carbohydrate synthesis and metabolism
• The liver uses glycogenesis to make glycogen from glucose, simple sugars
and amino acids.
• The glycogen produced is stored and used as needed.
• The liver also uses glucose for maintenance of mitochondrial NADH and to
generate ATP in the Embden Myerhoff pathway.
4) Protein synthesis and metabolism
• The liver makes various proteins (transferrin, prothrombin, and ceruloplasmin)
which are used to transport materials like iron, and copper.
• The liver also uses the proteins for nutrition, to regulate oncotic blood pressure
(albumin), and for coagulation.
o These proteins include albumin, HDL, LDL, VLDL, haptoglobin,
angiotensin, erythropoiten, and many others.
o These proteins also are used for nitrogen excretion by processing
ammonia, urea, creatinine and uric acid.
5) Lipid synthesis and metabolism• This is where cholesterol, triglycerides and phospholipids are synthesized.
• Free fatty acids are metabolized in the citric acid cycle into NADH, and bile
acids are produced form cholesterol.
6) Porphyrin synthesis
• The liver produces the enzyme aminolevulinic acid synthase which controls
the synthesis of porphyrins which eventually form heme molecules.
Page 11
7) Bile acid synthesis
• the bile acids are conjugated with amino acids to form bile salt.
• It is synthesized by cholesterol in the bile ducts and ends up in the intestine
where the lipids are digested.
8) iron and vitamin storage
• Iron is stored in the liver and transported wherever needed by transferrin, and
vitamins are stored in the liver and available to be used whenever needed.
9) Excretion of metabolic end product and detoxification
• It converts ammonia to urea.
10) Bile pigment formation
• Bilirubin is the principal pigment in bile and is derived from the breakdown of
hemoglobin when aged red blood cells are phagocytized by the
reticuloendothelial system, primarily in the spleen, liver and bone marrow.
2.

3.

Where is bilirubin produced
• Bilirubin is produced in the reticuloendothelial system from the breakdown of
hemoglobin from senescent red blood cells
• Bilirubin forms a complex with albumin for transport to the liver.
• In this form, bilirubin is unconjugated and not water soluble.
Describe heme catabolism by the reticuloendothelial system (extravascular
hemolysis). List states associated with increased extravascular hemolysis.
• Hemoglobin is broken down extravascularly into globin and heme (iron and
protoporphrin IX)
• Protoporphrin breaks down further into unconjugated bilirubin which is carried to the
liver by albumin and conjugated with glucuronyl transferase.
o This is associated with RBC membrane defects and defects in the heme
structure.

4.

Identify the function of haptoglobin, hemopexin albumin and methemalbumin in
heme catabolism in intravascular hemolysis.
• Haptoglobin binds free hemoglobin and takes it back to the liver so it is not lost in
the urine,
• Hemopexin albumin removes circulating heme from the blood and delivers it to the
liver,
• Methemalbumin is free heme oxidized and bound to albumin which is carried to the
liver and acts as storage that can be used until enough hemopexin is produced
and made available to the liver.

5.

List states associated with increased intravascular hemolysis.
• Intravascular hemolysis is associated with increased immunologic processes,
mechanical injury, and toxins.

6.

Differentiate between unconjugated and conjugated bilirubin
• Conjugated bilirubin

Page 12
Bilirubin is conjugated in the hepatocyte endoplasmic reticulum with
glucuronic acid to form bilirubin diglucuronide ( conjugated bilirubin).
o The reaction is catalyzed by UDP
o Conjugated bilirubin is water soluble.
o Conjugated bilirubin is excreted into the bile for storage in the gallbladder,
secreted into the duodenum in response to gallbladder stimulation, and
reduced by anaerobic bacteria in the intestine to urobilinogen.
o Some intestinal urobilinogen is reabsorbed;
 A portion returns to the liver and some enters the circulation for
excretion in the urine, whereas the remaining portion in the
intestines is oxidized by anaerobic bacteria for excretion in the stool
as urobilin.
o Urobilin is an orange- brown pigment that gives stool its characteristic color.
Unconjugated bilirubin
o Bilirubin is produced in the reticuloendothelial system from the breakdown
of hemoglobin from senescent red blood cells
o Bilirubin forms a complex with albumin for transport to the liver.
o In this form, bilirubin is unconjugated and not water soluble.
o

•

7.

Describe the process of bilirubin conjugation. What is the role of glucuronyl
transferase and where is this enzyme synthesized? What relationship exists
between enzyme synthesis and neonatal physiologic jaundice?
• Hemoglobin is broken down into portoporphrin which is converted into
unconjugated bilirubin which is bound to albumin and carried to the liver where it is
converted to conjugated bilirubin by the enzyme glucuronyl transferase
(synthesized in the liver).

8.

What is Kernicterus? Why does it develop?
• This is a serious newborn condition that occurs in the central nervous system
because of high bilirubin levels.
• It is caused by an under developed blood brain barrier, and because newborns do
not produce enough glucuronyl transferase.

9.

Describe excretion of bilirubin and resulting formation of urobilinogen? Why is
urobilinogen normally present in urine and serum?
• Conjugated bilirubin is taken to the intestines where bacteria convert it into
urobilinogen.
• Urobilinogen is normally present in the urine because a small amount of it is filtered
back to the liver where it is recirculated and sent to the kidney where it is excreted
in the urine.

10.

Prehepatic jaundice
• Prehepatic jaundice occurs when there is excessive erythrocyte destruction, as
seen in hemolytic anemias, spherocytosis, toxic conditions, hemolytic disease of
the newborn caused by Rh or ABO incompatibility, etc
Page 13
•
•

In these cases, the rate of hemolysis exceeds the liver’s ability to take up the
bilirubin for conjugation.
Prehepatic jaundice is characterized by an increased level of unconjugated bilirubin
in the serum.

11.

Hepatic jaundice
• occurs when the liver cells malfunction and cannot take up, conjugate, or secrete
bilirubin.
o Gilbert syndrome:
 Defect in the ability of hepatocytes to take up bilirubin; due to
transport problem of bilirubin from the sinusoidal membrane to the
microsomal region; characterized by mild increase in serum level of
unconjugated bilirubin
o Neonatal physiological jaundice:
 Level of UDP- glycuronyltransferase is low at birth;
 Takes several days for the liver to synthesize an adequate amount
of the enzyme to catalyze bilirubin conjugation; causes increased
serum level of unconjugated bilirubin

12.

Posthepatic jaundice
• Occurs when an obstruction blocks the flow of bile into the intestines.
• This is referred to as extrahepatic cholestasis and may be caused by gallstones
obstructing the common bile duct, neoplasms such as carcinoma of the ampulla of
Vater or carcinoma of the pancreas, and inflammatory conditions such as acute
cholangitis or acute pancreatitis.
• Posthepatic jaundice is characterized by:
o Significantly increased level of conjugated bilirubin in serum,
o Increased level of unconjugated bilirubin in serum,
o Increased conjugated bilirubin in the urine,
o Decreased urine and fecal urobilinogen
o Stool that appears pale in color.

13.

Why are bilirubin determinations performed on amniotic fluid?

D.

B. Porphyrins and Heme Derivates
1.

What are the porphyrias?
• These are deficiencies in the enzyme production that can be acquired or inherited.
They result in increased production of one of the heme precursors to which they
are intermediates.

2.

What is the physiologic function of porphyrins.
• The most important function of porphyrias are to chelate iron from heme.

Page 14
3.

When are plasma hemoglobin levels increased and how are they measured?
• The plasma hemoglobin levels are increased during thalessemias and
hemoglobinopathies.
• They can be measured by cellulose acetate electrophoresis or citrate agar
electrophoresis.

4.

What reagent is used for the detection of urobilinogen? When would
urobilinogen levels be decreased? Increased?
• Ehrlrich's reagent is most often used for urobilinogen detection
•
• Increased Urobilinogen levels
o Excess hemolysis,
o Liver damage by hypoxia
o Exposure to various toxic agents.
• Decreased Urobilinogen levels
o Obstructive jaundice because there is a limited delivery of bilirubin to the
gut.

5.

When would myoglobin be increased? How are serum myoglobin levels
measured?
• Myoglobin is increased when there is trauma to skeletal or cardiac muscle.
(myocardial infarction).
• Serum myoglobin levels are measured by Electrophoresis.

E. PROTEINS AND NON-PROTEIN NITROGEN
1.

What are the functions of proteins in normal physiology?
• Proteins function as transport carriers for other substances.
• They transport substances to the proper sites for absorption, modification, or other
utilization.

2.

Salt fractionation• The proteins are fractioned out by the use of salts. The salts decrease the water
available for hydration of the hydrophilic groups and cause precipitation of the
globulins.

3.

Zwitterion• An ion that has both positive and negative regions of charge.

4.

Zeta potential
• This is the potential difference between the negative charges on the surface of the
red blood cell membrane and the cations in the aqueous medium.
• Cations are divided into two groups, those that always move with the RBC and
those that can move freely in the medium.
• The zeta potential is measured from the boundary of these two cations to the
negative charge on the membrane.
Page 15
5.

Polypeptide
• Amino acids that combine to form proteins which link together to form peptides.
• Many peptides linked together form a polypeptide.

6.

Oligoclonal banding
• Electrophoretic pattern of CSF form patients with multiple sclerosis with distinct
bands in the globulin zone.

7.

Briefly describe the Kjeldahl techniques for determination of protein and nonprotein nitrogen.
• In this method the serum proteins are precipitated with an organic acid.
• The nonprotein nitrogen is removed with the supernatent.
• The protein pellet is digested in H2SO4 with heat and a catalyst (cupric sulfate).
• Potassium sulfate can also be used to improve the digestion.
• The H2SO4 oxidizes the C, H, and S in protein into CO2, CO, H2O, and SO2.
• The nitrogen in the protein is then converted to ammonium bisulfite which is
measured by adding alkali and distilling the ammonia into a standard boric acid
solution.
• The ammonium borate formed is then titrated with a standard solution of HCL to
determine the amount of nitrogen in the original protein solution.

8.

What are the major causes of increased and decreased albumin? Increased and
decreased globulins?

9.

What is the theory of refractometry?
• Refractometry- the velocity of light is changed as it passes the boundary between 2
transparent layers (air and water) causing light to be bent.
• When solute is added to water the refractive index at 20*C of 1.33 is increased by an
amount proportional to the concentration of the solute in the solution.

10.

What are major interfering substances in the determination of serum protein by
refractometry?
• Interfering substances- Nonprotien solids (electrolytes, urea, and glucose)

11.

Name three ways to separate albumins from globulins.
• Electrophoresis,
• Chromatography
• Precipitation

Page 16
12.

Discuss the reasons for determining spinal fluid protein and glucose.

13.

What are normal values for spinal fluid protein and glucose?

14.

What results are expected from spinal fluid in meningitis ?
• In meningitis, encephalitis, and neurosyphilis there would be a decreased glucose
level with increased protein levels (IgG).

15.

What results are expected from spinal fluid in encephalitis?
• In meningitis, encephalitis, and neurosyphilis there would be a decreased glucose
level with increased protein levels (IgG).

16.

What results are expected from spinal fluid in neurosyphilis?
• In meningitis, encephalitis, and neurosyphilis there would be a decreased glucose
level with increased protein levels (IgG).

17.

Discuss the BCG method for determining albumin? Why is the pH important?
• BCG (Bromocresol Green) method for determining albumin is a dye binding
procedure where positively charged albumin is attracted to and binds to the anionic
dye.
• Once bound to the albumin, the dye has a different absorption maximum than free
dye.
• The amount of albumin can be quantitated by measuring the absorbance of the
albumin-dye complex to which it is directly proportional.
• The pH must be adjusted on the solution to make the albumin positively charged so it
will bind to the dye.

18.

What is Biuret reagent? Explain its function in determination of total protein.
What are the major interfering substances?
• The biuret reagent contains sodium potassium tartrate to complex cupric ions to
prevent their precipitation in the alkaline solution, and potassium iodide which acts as
an antioxidant.
• In this procedure small peptides react and the color of the chelate produced has a
different shade that seen with larger peptides (color varies from pink to reddish violet
and is measured at 510nm).
• Major interfering substances are any compound with 2 or more of the following
groups NHCH2 , and NHCS.

19.

What is a protein-free filtrate? List 3 precipitating reagents used.
• A protein free filtrate removes proteins from whole blood, serum, urine, or other body
fluids by precipitation with a precipitant and then filtration or centrifugation.
• Some precipitating reagents are:
o Tungstic acid
o O-toluidine,
o Horseradish peroxidase,
o Molybdate
Page 17
o

Trichloroacetic acid

20.

What is Nessler's reagent? In what other reactions may it be used?
• Nessler's reagent:
o Double iodide of mercury and potassium.
o This reagent is used in the determination of non-protein nitrogens (NPN).
o It is also used in Nessleration reactions of urea nitrogen.

21.

What is an A/G ratio? How is it used diagnostically?
• A/G ratio determines how much albumin is in the body compared to globulins.
• You use it in correlation with Total protein to determine if albumin or globulins are
low.

F. Specific serum proteins
1.

What is alpha-1 antitrypsin? Ceruloplasmin? Name and describe disease
processes involved with each protein.
• Alpha-1 antitrypsin
o Acute phase reactant
o Main function
 Neutralizes trypsin-like enzymes that can cause hydrolytic damage to
structural protein.
o Disease process:
 Severe deficiency of this is associated with severe degenerative
emphysematous pulmonary disease.
• Ceruloplasmin
o Copper containing glycoprotein that stores 90% of the total serum copper.
o Disease process:
 Low serum concentrations of copper are associated with Wilson’s
disease.

2.
•
3.
•

4.

What is the function of haptoglobin?
Haptoglobin functions to bind free Hemoglobin by the alpha chain.
Name three methods for measuring haptoglobin.
3 methods to measure Haptoglobin:
o Starch gel electrophoresis,
o Radial immunodiffusion,
o Immunonephelometric methods.

When is haptoglobin decreased? Increased?
• Increased Haptoglobin levels:
o Inflammatory conditions,
Page 18
•

5.
•

o Burns, and
o Nephrotic syndrome
Decreased Haptoglobin levels:
o Intravascular hemolysis,
o Transfusion reactions,
o HDN,
o Mechanical breakdown of RBC's,
o Athletic trauma.

Define Troponin
Troponin- is a complex of 3 proteins that bind to the thin filaments of striated muscle
(cardiac and skeletal) but are not present in smooth muscle.

6.

List three isoforms that make up the troponin complex.
o 3 isoforms:
 TnT,
 TnI,
 TnC

7.

Describe the advantages of troponin over CKMB and name the method currently
available to measure cTnI.
o Cardiac troponin I is highly specific for myocardial tissue, and because it does
not normally circulate in blood it is 13x more abundant in the myocardium than
CKMB on a weight basis.
o cTnI is very sensitive and can indicate even a minor amount of cardiac necrosis.
o The relative increase of cTnI is greater than that of CKMB.
o cTnI can be measured by:
o Immunoenzymetric assays using 2 monoclonal Ab's directed against different
epitopes on the protein.

G. Protein Electrophoresis
1.

What is a monoclonal gammopathy? polyclonal gammopathy?
A monoclonal gammopathy is a sharp narrow band in the late beta or gamma region
that suggests a monoclonal M spike.
• The M spike is a spike of one class of Ig’s that is possibly metastasizing or producing
clones of itself and suggests cancer, possibly multiple myeloma.
• A polyclonal gammopathy is a broad gamma band that is increased. It looks
abnormal and is usually caused by an infection. It is made up of more than one
serum protein being increased.

•

2.

Where are the sites of synthesis for the following proteins:
a. albumin- liver
b. alpha-1 globulin- liver
c. alpha-2 globulin- liver
Page 19
d. beta globulin- liver
e. gamma globulins- (IgA, IgG, and IgM)- made by B cells that become plasma
cells.
3.

Discuss the clinical picture of multiple myeloma. What are the expected results
of electrophoretic patterns on serum and urine of a myeloma patient?
• In multiple myeloma you would see:
o Bence jones proteins found in the urine (free kappa and lambda light
chains)
o "M" spike in electrophoretic pattern

4.

Briefly describe serum protein electrophoresis.
• Serum electrophoresis
o Serum samples are applied to the cathode end of a support medium strip
that is saturated with an alkaline buffer (8.6). The strip is connected to 2
electrodes and a current is passed. All serum proteins are negatively
charged at the 8.6 pH so they migrate toward the anode end.

5.

Name the five bands that occur in serum and list the major proteins that migrate
in the five bands.

6.

Diagram a normal pattern as they migrate from anode to cathode in a barbital
buffer and label each peak.

7.

What type of electrophoretic patterns can be expected in the following disease
states? Explain the patterns and sketch them.
a. Multiple myeloma

b. Nephrotic syndrome

c. Liver disease

d. Chronic infection

e. Acute phase reaction

f.

Malnutrition

Page 20
8.

Describe CSF protein electrophoresis.
• CSF protein electrophoresis is done with the same technique as serum
electrophoresis, except agarose gel is used most often because it is a high
resolution technique.

9.

What bands are normally seen in CSF protein electrophoresis
• Oligoclonal bands are: distinct bands seen in the globulin zone and they are
associated with multiple sclerosis (90%), and Inflammatory infectious neurological
disease.

10.

What are oligoclonal bands and what disease process are they associated with?
• pre-albumin,
• albumin,
• alpha-1 globulin (antitrypsin),
• insignificant alpha-2 globulin,
• Beta-1 zone (transferrin)
• Beta-2 zone.

11.

Describe protein electrophoresis patterns would appear in the following
situations and explain why the pattern appears as it does.

a. Electrophoresis of plasma instead of serum

b. Serum containing alpha-feto protein

c. Fresh serum containing complement

d. Bisalbuminemia

e. Serum containing C-reactive protein

Page 21
H. Creatinine, Bun, Uric Acid

1.

What is NPN? Which compounds comprise 50% of the total NPN's?
• Non-protein nitrogen (NPN)- nitrogen containing compounds that remain in the blood
sample after the removal of protein constituents.
• Urea is the compound that comprises 50% of the total NPN’s.

2.

What is BUN? What is Azotemia? How is urea nitrogen converted to urea
mathematically?
• Blood urea nitrogen (BUN)- nitrogen in the blood in the form of urea. This is a
measurement that is used to analyze the urea level.
• Azotemia- an elevated level of urea in the blood.
• Bun x 2.14 = urea

3.

What is creatine? How is it measured?
• Creatine- a compound found in muscle synthesized from several amino acids. It
combines with high energy phosphate to form creatine phosphate which functions as
an energy compound.
• We can measure creatine by the Jaffe method which measures creatine based on
analyzing the sample for creatinine before and after heating in acid solution. The
heating converts the creatine to creatinine and the difference between the two
samples is the creatine concentration.

4.

Relate elevations in uric acid to the following disease states:
•

•
•
•

•

Primary gout- this is caused by increases of uric acid which cause sodium urates
to precipitate in the joints. This can be caused by overproduction of uric acid,
drugs, and alcoholism.
Secondary gout- this gout is formed as a secondary infection caused by a larger
problem like leukemia.
Leukemia- this causes the increased breakdown of cell nuclei caused by
chemotherapy which causes the uric acid levels to increase.
Polycythemia- this causes the increased breakdown of cell nuclei caused by
chemotherapy, much like leukemia does, which also increases the uric acid
levels to increase.
Glomerular nephritis- in this disease the nephrons of the glomerulus are
damaged in the kidney which causes poor filtration and increased levels of the
uric acid.

Page 22
•

Multiple myeloma- this disease is also treated with chemotherapy which breaks
down the nuceli and causes the uric acid to increase much like leukemia and
polycythemia.

5.

Relate uric acid production to purine catabolism.
• Uric acid is formed from the catabolism of purines like adenosine and guanine in the
liver. This uric acid is transported by the plasma from the liver to the kidney where it
is filtered by the glomerulus into the proximal tubules where most is reabsorbed and
only small amounts are secreted into the urine.

6.

What is the clinical significance of BUN? (renal, prerenal, postrenal). What
creatinine values are expected in these conditions?
• BUN- (direct urea measurement from serum or plasma)
• It is used extensively in the determination of renal function.
BUN

RATIO
Pre-renalRenalPost renal7.

Increased
Normal
Increased
Increased
Normal Increased

BUN/CR
Increased
Normal
Decreased

What are normal values for BUN and creatinine? What is the normal ratio of BUN
to creatinine? When is the ratio altered?
• Normal BUN- 7-18mg/dl
• Normal Creatinine- 0.5-1.2mg/dl
• Normal BUN/Creatinine ratio- 10:1-20:1
• The ratio is altered in:
o Low protein uptake
o Acute tubular necrosis
o Severe liver disease
•

8.

CREATINE

Diagram and describe the Berthelot reaction for BUN. Berthelot Reaction for
BUNo Urea is hydrolyzed with urease, and the ammonia ion formed is reacted with
phenol and hypochlorite in alkaline medium to form indophenol.
Nitroprusside is used to catalyze the reaction.
o Absorbance of dissociated indophenol (blue chromogen) is measured at
560nm.
o REACTION:
 NH4 + 5NaOCC+ 2 phenol◊ indophenol + 5NaCl + 5H2O
What is the purpose of the following reagents?
o urease- used ot prepare the stock suspension
Page 23
o
o
o

sodium nitroprusside- catalyzes the reaction
phenol- converted to indophenol
alkaline hypochloride- aids in conversion of phenol to indophenol.

9.

What are some advantages and disadvantages of this method? What
anticoagulant must be avoided when using any urease method?
o Advantages- can use serum, plasma, or urine.
o Disadvantages- contamination of urine with bacteria is common and can
cause decreased urea and formation of ammonia.

10.

What kidney functions do the following clearance tests measure?
inulin- Reference substance for measuring GFR (glomerular filtration rate)
creatinine- universely used in assessment of GFR.
urea- not useful in monitoring GFR, but serum urea may provide useful clinical
metabolic
information.
p-amino-hippurate- reference substance for the measurement of renal plasma flow.

•
•
•
•
•
11.

12.

Diagram and describe the Jaffe reaction for creatinine. What substances give
false positive reaction?
• Jaffe reactiono The reaction occurs between creatinine and the picrate ion formed in the
alkaline medium and a red-orange adduct develops. Teh observed rate of
the hydroxyl ion concentrations over a broad range of picric acid
concentrations. This is measured spectrophotometricaly at wavelengths of
485-520nm.
• Substances that give false positives are:
• Protein,
• Glucose,
• Vitamin C (ascorbic acid),
• Acetone,
• cephalosporin
Why is creatinine preferred to urea for clearance tests? What data are necessary
to calculate creatinine clearance? Write the formula. What are the normal values
for creatinine clearance?
•
•

•
•
•

Creatinine is more specific for kidney function than urea is.
Data necessary for calculating creatinine clearance:
o Urine volume,
o Creatinine concentration. in urine,
o Creatinine concentration in plasma
Creatine Clearance = (Urine Cr x Urine volume)/ (Plasma Cr)
Normals: Male= 97-137 ml/min
Females = 88-128 ml/min

Page 24
13.

•

A creatinine clearance was performed on a male patient 1.5m tall and weighing
65kg. His blood contained 2.5 mg/dl creatinine. The urine creatinine was 50
mg/dl and the urine volume was 300 ml/4hrs. What was the creatinine clearance
for this man?
Creatine Clearance = (50 x 300)/ 2.5 x 1.76/ 1.60 (body surface area *see chart)
•

14.

15.

I.

What is creatinine? What is the normal range? What single disease state is
associated with elevated creatinine.
• Creatinine is a compound formed when creatine or creatine phosphate
spontaneously loses water or phosphoric acid.
• It is excreted into the plasma at a relatively constant rate in a given individual and
excreted in the urine. Its decrease is associated with renal dysfunction as in
glomerulonephritis.
Diagram the reaction, list reagents used and describe the principle of the
oxidation reduction method for uric acid using phosphotungstic acid.
• This is the most common method used.
• It is based on the oxidation of uric acid in a protein-free filtrate with subsequent
reduction of phosphotungstic acid to tungsten blue.
• It uses Na carbonate to provide the alkaline pH necessary for the color
development.
• The blue color produced can be intensified by adding cyanide or by keeping the
proper pH.

Miscellaneous Proteins
1.
•
•

Describe the method for detecting phenolketonuria? What enzyme deficiency
results in phenolketonuria?
Phenolketonuria results from a total absence of or absence of activity of the enzyme
phenylalanine hydrolase (AKA phenylalanine-4-mono-oxygenase).
You can use the Guthrie test to detect PKU.

2.

What are cryoglobulins? How are they measured?
• Cryoglobulins are serum protein that precipitates at temperatures lower than body
temperature.

3.

What is alpha-fetoprotein? What does its presence signify? How is it detected?
• Alpha-fetoprotein is a globulin protein synthesized in the fetal yolk sac and then by
the parenchymal cells of the liver.

Page 25
•
•

It is measured to determine if there is increased passage of fetal proteins into the
amniotic fluid.
We also measure the alpha-fetoprotein levels in association with spinabifiida, renal
tube defects, and general fetal distress.

4.

What is carcinoembryonic antigen? What types of tumors is it most frequently
associated with? What types of methods are used to measure it?
• Carcinoembryonic anitgens (CEA) are glycoproteins which are associated with
numerous cancers (colon, lung, pancreas, stomach, or breast tissue tumors)

5.

For each of the following tumor markers describe the types of tumors they are
most often associated with and how they are measured.
• CA125- ovarian cancer.
• PSA- (prostate specific antigen)- Prostate cancer

J. ENZYMES

1.

(L) Give the substrates for the following: LD, CK, AST, ALT, GGT, CK-MB
• LD
Lactate
o Catalyzes oxidation of Lactate to Pyruvate and the reverse reaction of
Pyruvate to Lactate
o ischemia,
o myocarditis,
o cardiac congestion;
• CK
Creatinine
o Catalyzes the reversible phosphorylation of ATP
o Muscular dystrophy,
o muscle malignancies,
o heart disease,
o thyroid disease,
o CNS disease.
• AST
Aspartate
o Transfers amino acids
o This is higher in neonates due to their immature liver
o Liver disease (20-100 times normal in hepatitis),
o carcinoma,
o cirrhosis,
o liver disease
o heart disease,
o muscle disease,
o gallbladder disease,
o AMI,
o pulmonary embolism
Page 26
•

•

•

2.

ALT
o
o
o
o
o
o
o
GGT
o

Alanine
Catalyzes the transfer of an amino group of alanine to alpha-ketoglutarate
Enzymatic-UV Monitoring
Liver disease,
carcinoma
gallbladder disease,
cirrhosis,
hepatotoxicity
Glutathione
Transfers gamma-glutamyl residue from gamma-glutamyl peptides to amino
acids, water, and other small peptides
o Liver disease,
o obstructions of the internal liver or gallbladder,
o alcoholism,
o pancreatic problems
CK-MB Creatinine
o Catalyzes the reversible phosphorylation of ATP
o Myocardial problems

(L) Define:
a. Isoenzymeo one of several forms in which an enzyme can exist in various tissues.
o Although they are similar they can be separated from each other by special
chemical tests (electrophoresis) to give more specific information.
b. Coenzymeo These are enzyme activators that are usually heat stable and of low
molecular weight.
o When these are combined with an inactive protein called an apoenzyme they
form an active compound or a complete enzyme called holoenzyme.
c. Catalysto Substance that speeds up the rate of a chemical reaction without itself being
permanently altered of used up in the reaction.
o They are effective in small quantities and are not used up in the reaction.
They can be recovered unchanged.
d. Activator
o Substance in the body that converts an inactive substance into an active
agent.
o Example: the hydrogen ions on pepsinogen converting it to pepsin.
e. Inhibitoro Chemical substance that stops the enzyme activity.
f. Hydrolaseo Enzyme that causes hydrolysis. These catalyze bond cleavage by the
addition of water.
g. OxidoreductasePage 27
h. Enzyme that removes electorns and their corresponding electrons.
i. Transferaseo These enzymes move chemical grouping from one compound to another.
3.

Relate amylase and lipase activity to the following disease states:
a. Acute pancreatitis- both increase in this.
b. Malabsorption- increased in both.
c. Chronic pancreatitis- increased in both
d. Pancreatic carcinoma- increased in both
e. Cystic fibrosis- increased in both because it leads to malabsorption

4.

What are the sources of acid phosphate in the body? What are normal ranges for
acid phosphatase in males and females?
o Acid phosphate is found in most tissues in the body like bone, bone marrow,
liver, spleen, RBC’s, platelets, and in the highest concentration in the prostate
gland of the male.

5.

8. (L) Explain the clinical significance of alkaline phosphatase in the following
disease states:

a.
b.
c.
d.
e.

Obstructive jaundice- increased ALP levels
Parenchymal jaundice- increased ALP levels
Paget's disease- increased ALP levels
Hyperparathyroidism- increased ALP levels
Pregnancy- increased ALP levels

6.

List enzymes elevated in hemolysis? What enzyme might be depressed with
refrigeration and freezing?
• Enzymes elevated in hemolysis:
o CK, LD, AST (aspartate transferase), ACP (acid phosphatase), ALP
(alkaline phosphatase), and LIPASE
• Enzymes depressed by refrigeration:
o LD (occurs at 4*C within 24 hours), and LIPASE (occurs if stored at 4*C for
3 weeks)

7.
•

What is cholinesterase? Why is it important in presurgery cases?
Cholinesterase: an enzyme found in RBC’s, lungs, spleen, nerve endings, and brain. It
is responsible for the prompt hydrolysis of acetylcholine released at the nerve endings to
mediate transmission of the neural impulse across the synapse. The degradation of
acetylcholine is necessary to the depolarization of the nerve so that it can be repolarized
in the next conduction.
o This is important to measure cholinesterase in presurgery to determine the
amount of succinyl dicholine (muscle relaxer) that can be given in surgery without
complications. (You can only give the amount of succinyl dicholine to the patient
that the patients’ cholinesterase can rid their body of.)
Page 28
8.

What is the relationship of amylase and lipase in pancreatic disease? Why are
both tests necessary in the monitoring of the disease?
o Amylase and Lipase are both elevated in pancreatic disease.
o You must monitor both amylase and lipase in pancreatic disease because:
 Amylase is more sensitive, but less specific (b/c also found in other parts
of the body)
 Lipase is more specific, but found in small quantities (less sensitive b/c
only found in the pancreas)

9.

Discuss the following in relation to amylase activity: activators, pH,
temperature.
o Activatiors: Calcium and Chloride ions
o pH- optimal is 6.9 - 7.0
o Temperature- optimal is 37-40*C

10.

Discuss prostatic disease and acid phosphatase levels in serum.
• Total activities of ACP may reach 40-50 times the normal in severe stages of
prostate cancer. If the carcinoma is highly localized to the prostate there may only
be slight increases in ACP activity. In benign hypertrophy of prostate, enzyme
levels are normal.

11.

How is the L(+) tartrate utilized in the determination of acid phosphatase?
• Tartrate inhibits the activity of non-prostatic ACP so that specificity is enhanced
when it is used.

12.

Explain heat separation of alkaline phosphate isoenzymes. How does heat effect
the liver fraction? bone? placenta?
• Heat separation of alkaline phosphatase Isoenzymes
 ALP activity is determined by measuring ALP before and after heating serum at
56*C for 10 minutes.
 Placental ALP is the most heat stable followed by intestinal, liver, then bone.
 Placental ALP will resist heat denaturation at 65*C for 30 min.
 If the residual activity after heating is <20% of the total prior to heating then it is
bone phosphatase.
 If the residual activity after heating is >20% of the total prior to heating then it is
Liver phosphatase.

K. Enzyme Electrophoresis

1.

List the CK isoenzymes. Describe the makeup of each fraction and organs
associated with each fraction. How are isoenzymes separated?
• CK-1 ( brain, brain subunits)- brain, prostate, uterus, bladdar, placenta
• CK-2 (muscle, brain subunits)- heart muscle and skeletal muscle
• CK-3 (muscle, muscle subunits)- sketal muscle and heart muscle
Page 29
2.

How can hemolysis affect the LDH electrophoretic pattern? What is the clinical
significance of this?
• Hemolysis can cause LDH electrophoresis to have an LD-1 to LD-2 flip.
• Using a hemolyzed sample would cause the results to have a LD-1 to LD-2 flip as
seen in cases of Myocardial Infarctions and Hemolytic anemia.

L. ELECTROLYTES AND TRACE ELEMENTS
1.

For each of the following give normal range, panic values, categorized as anion
or cation if applicable and categorize as intracellular or extracellular if
applicable.
• Potassium – Cation
• Sodium – Cation
• Calcium – Cation
• Magnesium – Cation
-

•

Bicarbonate (HCO 3) – Anion

•

Chloride (Cl) - Anion

2.

Describe the relationship between electrolyte balance and water balance.
Include the roles of the kidney, hypothalamus, ADH and the renin-aldosterone
system.
• The electrolyte balance and water balance are directly related.
• The plasma sodium concentration depends greatly on the intake and excretion of
water.
• If the sodium increases it stimulates thirst which will increase the intake of water,
and the kidneys have the ability to conserve or excrete large amounts of sodium
depending on the blood volume which is directly related to the water volume.
• The excretion of water is largely affected by the ADH (which is secreted from the
hypothalamus) release in response to the increase in blood volume.
• The renin-aldosterone system acts in the kidney to increase the retention of
sodium and increase the excretion of potassium which will eventually increase
the blood pressure by using the electrolyte to balance the water level.

3.

List anticoagulants of choice and the effect of hemolysis if any on the following
ions: K, Na, Ca, Mg, HCO3, Cl, P04, Fe.
• All electrolytes should be determined using serum or heparinized plasma.
o K- hemolysis increases K levels
o Mg- hemolysis increases Mg levels
o HCO3- hemolysis increases HCO3 levels
o P04- hemolysis increases PO4 levels
o Fe- hemolysis increases Fe levels
•
o Na-hemolysis does not effect this significantly
Page 30
o
o

Ca- hemolysis does not effect this significantly
Cl- hemolysis does not effect this significantly

4.

Identify the major functions of sodium, chloride, bicarbonate, and potassium.
• Sodium- this electrolyte largely determines the osmolality of the plasma.
• Chloride- this electrolyte maintains the electrical balance by balancing the
sodium charge, and using the chloride shift with bicarbonate.
• Bicarbonate-this electrolyte is used to maintain the acid base balance and buffer
the blood.
• Potassium- this electrolyte regulates the neuromuscular excitability, contraction
of the heart, ICF volume, and hydrogen ion concentration (pH).

5.

Define and list conditions associated with each of the following:
•

Hyponatremia- this is decreased levels of sodium in the blood and is associated
with the blood volume status. It results from sodium loss in excess of water loss.

•

Hypernatremia- elevated levels of sodium in the blood and is associated with
increased sodium concentration because of excess water loss. It can be caused
by increased sodium intake or decreased water intake.

•

Hypokalemia- decreased levels of potassium in the blood and is associated with
GI or urinary loss of potassium , or with increased cellular use of potassium. This
can be caused by vomiting, diarrhea, etc.

•

Hyperkalemia- increased levels of potassium in the blood and is associated with
diabetes mellitus, or metabolic acidosis.

•

•

•

6.

What disease process results with increased bicarbonate? decreased
bicarbonate?
• Metabolic acidosis is related to the decrease in bicarbonate.
• Metabolic alkalosis is related to the increase in bicarbonate.

7.

Define titration. Explain its application to chloride methods.
• Titration is the diluting out of a sample with a liquid reagent of a known strength
and measuring the volume necessary to convert the sample through a given
reaction.
• In the chloride titration method the chloride ions combine with the mercuric ions
to form soluble and undissociated mercuric chloride. The proteins in the serum
are precipitated with the tungstic acid and an aliquot of the filtrate is titrated with
an acidic solution of mercuric nitrate using a color indicator. This color indicator
turns violet-blue at the first excess of mercuric ion.
Page 31
8.

List disease processes in which hyperchloremia and hypochloremia occur but
sodium is normal.
• Hyperchloremia- this is an increase of serum chloride and occurs in situations
where there is an excess loss of bicarbonate ion due to GI losses, RTA, or
metabolic acidosis.
• Hypochloremia-this is an decrease of serum chloride and occurs with the excess
loss of chloride from prolonged vomiting, diabetic ketoacidosis, or aldosterone
deficiency.

9.

List causes of an increased anion gap? decreased anion gap?
• Decreased anion gap is rare, but may be seen in multiple meyloma because of
abnormal proteins. It can also be caused by instrument error.
• Increased anion gap- this may be caused by uremia, ketoacidosis (seen in
starvation or diabetes), posioning due to ingestion of substances like methanol or
ethylene glycol, lactic acidosis, or severe dehydration which causes increased
plasma proteins or instrument error.

10.

Explain why stock standards of sodium and potassium are kept in plastic
containers.
• Stock standards of Na and K are kept in plastic containers because glass
containers leach out the Na and K from the sample.

11.

What is an "anion gap". List formulas for its determination.
• Anion gap- the difference between unmeasured anions and unmeasured cations.
It is useful for indicating an increase in one or more of the unmeasured anions in
serum.
• AG = Na - (Cl + HCO3)
• AG = (Na + K) - (Cl + HCO3)

12.

Why is an anion gap routinely performed on all sets of electrolytes? What is an
unacceptable gap? What is standard operating procedure when an anion gap is
unacceptable?
o An anion gap is routinely performed on all sets of electrolytes because it is useful
in indicating an increase in one or more of the unmeasured anions in serum, and
for QC on th analyzer (an abnormal gap can indicate an analyzer problem if
performed on a person in good health).
o An acceptable gap is 10-20:1, so greater than or less than that would be
considered unacceptable and would need to be rerun.

13.

Describe the impact of each of the following on serum potassium levels.
• administration of insulin- decrease serum K levels (increases the cellular uptake
of K)
Page 32
•
•

acidosis- increase serum K levels (excess H enters cell to be buffered and
causes K to leave the cell to maintain electro neutrality)
alkalosis- decreases the serum K (increases the cellular uptake of K)

14.

Discuss the following factors influencing serum calcium and phosphorus levels:
• parathyroid hormone- this hormone is used to increase the absorption of calcium
and increase the excretion of phosphorus. To increase calcium it breaks down
the bone to release Ca (bone resprption), it conserves Ca by increasing the
tubular reabsorption in the kidney and it stimulates the renal production of vitamin
D which also increases the Ca absorption. To decrease phosphorus the blood
concentration the PTH increases the renal excretion.
• calcitonin- this hormone is used to decrease calcium levels and increase the
phosphorus levels which inhibits the actions of PTH and vitamin D.
• Vitamin D (calcitriol)-when the calcium is decreased or the phosphorus is
increased this hormone is used to increase calcium by aiding the effects of PTH
by causing more calcium to be stored or released. It decreases phosphorus by
increasing the absorption of it in the intestines and increasing the reabsroption in
the kidneys.
• plasma proteins-Albumin is the plasma protein that maintains the appropriate
fluid in the tissues, and it binds various substances in the blood like calcium.
• serum pH- a decrease in pH will increase the phosphate levels in the serum like
seen with antiacids.

15.

Discuss calcium, phosphorus and PTH levels related to the following disease
states:
• bone disease-calcium will be normal to low, phosphorus will be normal to low,
and PTH will be normal to high.
• malabsorption- calcium will be decreased, Phosphorus will be decreased, and
PTH will be increased.
• renal failure- calcium will be low to normal, phosphorus will by high, and PTH will
also be high.
• liver disease- calcium will be decreased, Phosphorus will be decreased, and PTH
will be increased.
• primary hyperparathyroidism-Calcium will be Increased, Phosphorus will be
decreased, and PTH will be high.
• secondary hyperparathyroidism-Calcium will be decreased, Phosphorus will be
low to normal to high, and PTH will be increased.
• primary hypoparathyroidism-Calcium will be decreased, Phosphorus will be
increased, and PTH will be decreased.
• secondary hypoparathyroidism-Calcium will be decreased, Phosphorus will be
increased or decreased, and PTH will be decreased.

Page 33
16.

What is the physiologic relationship between calcium and phosphorus? Why?
What are the physiologic functions of calcium and phosphorus?
• Calcium and phosphorus are inversely related in the serum because phosphate
is an intracellular anion, and calcium is an extracellular cation..
• Calcium functions in bone matrix, as an enzyme activator, in coagulation and
complement, and in muscle contraction.
• Phosphorus functions in production of ATP, GTP, CTP, UTP, and DNA
structures, as a major body buffer, and in the bone matrix.

17.

What is the relationship of calcium levels to alkaline phosphatase activity?
• Alkaline phosphatase levels are increased in periods of bone growth or
reconstruction which uses up the calcium and causes the serum levels of calcium
to decrease. They both also function in bone matrix.

18.

Describe three forms of body calcium. To which form is PTH most sensitive?
How can only "active" calcium be measured?
• Calcium forms- free-ionized Ca, protein bound calcium, and as complexed salts.
PTH is most sensitive to ionized calcium. To measure only the active calcium you
must measure it under anaerobic conditions because an increase in pH can cause
the protein bound Ca to increase which decreases the ionized Ca, and decreasing
the pH can cause the protein bound Ca to decrease and the ionized Ca to increase.

19.

Define and describe tetany? What are the relationship of magnesium and
calcium to tetany.
• Tetany- irregular muscle spasms.
• Calcium- a rapid decrease in ionized calcium concentration will cause tetany.
• Magnesium- this is required along with ATPase for normal Ca uptake following a
contration. It is also required for muscle cell stimulation by regulating the
acetylcholine which is a potent neurotransmitter.

20.

Describe the relationship between parathyroid hormone and magnesium levels.
• The parathyroid hormone increases the renal absorption of magnesium and
enhanses the absorption of magnesium in the intestine. PTH regulates Ca, Na, and
Mg.

21.

What is the physiologic role of magnesium? Where is magnesium stored in the
body?
o Magnesium functions as a cofactor for more than 300 enzymes including
those important in glycolosis, transcellular ion transport, neuromuscular
transmission, synthesis of carbohydrates and many others.
o Magnesium is stored in the bone (53%) and the rest (46%) is in the muscle,
soft tissue and other organs.
Page 34
22.

Discuss the following as they pertain to magnesium:
o alcoholism- people who are alcoholics tend to have diets deficient in
magnesium or have problems with malabsorption (hypomagnesemia).
o malabsorption- this causes a decrease in magnesium because it is not
absorbed (hypomagnesemia).
o magnesium sulfate therapy- this is given parenterally to severely ill patients.
o secondary hypoparathyroidism- this may cause an increased renal excretion
of magnesium due to an excess of calcium ions.

23.

What is the physiologic function of iron? How is iron transported in the body?
How is iron stored in the body?
• Iron functions as part of heme in hemoglobin. It is transported by transferrin and
stored in the body as ferratin and hemosidrin.

24.

Describe iron levels, %Saturation, TIBC, and ferratin levels in the following
disease states:
• iron deficiency anemia-the % saturation is decreased, TIBC is increased, and ferratin
is decreased.
• anemia of chronic infection- the % saturation is normal, TIBC is decreased, and
ferratin is normal to increased.
• hemochromatosis- the % saturation is increased, TIBC is increased, and ferratin is
increased.

25.

What is the physiologic function of copper? What is Wilson's disease?
ceruloplasmin?
• Copper functions as an enzyme indicator, it acts on ferroxidase, and it acts it is a
component of enzymes or proteins involved in redox reactions.
• Wilson’s disease- also known as hepatolenticular degeneration is a genetically
determined defect in ATPase where the copper is transported normally from the
intestine into the liver, but cannot be transported from the liver into the bile.

26.

What is the physiologic function of zinc? How is it measured?
• Zinc functions as a cofactor for more than 300 enzymes. It can be measured by
atomic absorption.

27.

Define:
• total iron binding capacity- (amount of transferrin bound already)
• An estimate of serum transferrin levels obtained by measuring the total iron binding
capability of a patients serum. Since transferrin represents most of the iron binding
capacity of serum TIBC it is generally a good estimate of serum transferrin levels.
• % saturation- (transferrin saturation in the patients sample)
• % saturation = total iron/TIBC x 100

Page 35
•
•
•
•
28.

•

•
•
•

unsaturated iron binding capacity- (the amount of sites available to bind iron in
patients sample)
UIBC = iron added – excess iron
Serum iron = TIBC – UIBC
latent iron binding capacity- (estimate of non-reacting iron bound to transferrin)

What is the source of blood ammonia? How is it detoxified? What disease
processes are associated with increased ammonia? How is ammonia related to
Reye's Syndrome?
Blood ammonia arises or comes from the deamination of amina acids through the action
of degestive and bacterial enzymes on proteins in the intestinal tract. Ammonia is
released from metabolic reactions that occur in skeletal muscles during excercize.
Ammonia is detoxified by the liver in the urea cycle where ammonia is converted to urea.
Hepatic failure, Reye’s syndrome, and urea cycle enzyme deficiencies are associated
with increased ammonia.
Reye’s syndrome is usually following a viral infection and it uses ammonia levels to
correlate with the severity of the disease and prognosis.

29. What is the purpose of renal synthesis of ammonia?
• The kidney synthesizes ammonia to compensate for metabolic acidosis.
30.

What are the three ketone bodies. When are they formed? List disease
processes associated with increased ketones.
o acetone, acetoacetic acid, and beta-hydroxybutyric acid.
o These are formed as a product of incomplete fat metabolism, and are associated
with diabetes mellitus, starvation, and prolonged vomiting.

31.

What is lactic acidosis? When does it occur?
o Lactic acidosis- an increase or accumulation of lactic acid in the blood.
o This occurs if there is improper oxidation of skeletal muscle and other tissues.

32.

What role does hemoglobin play as an important buffer system in the body?
What is carbonic anhydrase? What is chloride shift?
• Hemoglobin buffers the blood by delivering oxygen to the tissues and then taking
the carbon dioxide to the lungs to be exhaled.
• Carbonic anhydrase- this enzyme catalyzes the reaction of carbon dioxide to
bicarbonate and Hydrogen ion.
• The chloride shift- the carbon dioxide goes into the RBC and forms carbonic acid,
this acid splits into hydrogen ion and bicarbonate. The bicarbonate leaves the cell
and makes it more negative outside the cell, and more positive on the inside
because of the hydrogen ion. At this point the chloride shifts into the cell to balance
the electorneutrality of the cell.

Page 36
33.

34.
•

What are the main factors which influence the oxygen binding ability of
hemoglobin? Specifically how do acidosis and alkalosis affect O2 saturation?
• If there is increased oxygen then the hemoglobin binds more oxygen and if there is
less oxygen the hemoglobin picks up less oxygen and wants to hold on to it. The
ability for hemoglobin to bind oxygen depends mostly on the availability of oxygen.
• Acidosis- the pH drops drastically and increases the hemoglobin affinity for O2.
• Alkalosis- the pH is increased and it decreases the hemoglobin affinity for O2.
Why is heparin the anti-coagulant of choice for pH and blood gas work? How
does it work?
Heparin is used because it holds the ph constant in blood and prevents the change in
gals levels in the sample.

35. Why are blood gas specimens placed in ice immediately after collection?
• Blood gas specimens are placed on ice immediately after collection because:
• The pH decreases with time if it is not placed on ice immediately after drawn. The lower
temperatures prevent the cells from undergoing glycolysis.
IX. ENDOCRINOLOGY

36.

37.
•

Give expected T4, T3 uptake, FTI and TSH levels in the following:

What is the function of the thyroid hormones in body metabolism? Describe the
following disease processes:
The thyroid hormone function is to stimulate the metabolism. Without the thyroid
hormone the metabolism will decrease and the patient will experience obesity, mental
retardation, edema (water in the tissues), decreased body temperature, and anemia.
Page 37
•
•
•
•
•
•

•

cretinism- condition where the patient has a dysfunctional or no thyroid causing mental
retardation and death. Autosomal recessive.
juvenile myxedema-A dysfunction of the thyroid after birth that has very severe
consequences. All 50 states require a T4 screening test for neonates.
adult myxedema- A dysfunction of the thyroid later in life (>30 years) causing hair loss,
dry skin, yellow pallor, thick tongue, and arterosclerosos.
endemic goiter- an enlarged thyroid due to lack of iodine.
hashimoto's disease- the most common form of thyroid disease occurring mostly in
women 40to 60 years old. Treat with thyroxine. Autoimmune disease.
grave's disease- the patient has and AB to TSH receptor on the thyroid gland. This AB
causes the thyroid gland to think it is TSH and start to release T3 and T4 even though
there is a decrease in actual TSH.
thyroid tumors (goiters)- Tumors that cause the dysfunction of the thyroid gland allowing
abnormal amounts of hormone to be released.

38.

What are the roles of TSH? What amino acids are necessary for synthesis?
• Body temperature stimulates the thyroid hormone which stimulates the
hypothalamus and causes it to release TRH to the pituitary gland. The TRH
causes the pituitary gland to release TSH to stimulate the thyroid gland which then
releases T3 and T4. TheT3 and T4 cause the body temperature to rise and act as
the shunt to tell the hypothalamus to stop making the TRH and the pituitary to stop
making TSH.

39.

What is thyroglobulin? thyroid binding globulin?
• Thyroglobulin- an iodine contaning protein secreted by the thyroid gland.
• Thyroid binding globulin- A protein that binds thyroid hormones. The TBG assay is
used to confirm results of T3 and T4, or abnormalities it the relationship of the TT4
and T3U test. They can also be used as tumor markers for thyroid cancer.

40.

Briefly describe each of the following groups synthesized by the adrenal cortex.
What molecule is necessary for their synthesis?
• glucocorticoids- (cortisol) these stimulate gluconeogenesis to increase glucose,
encourage the glycogen production and release from the liver, block epinephrine,
act as and anti-inflammatory, inhibit WBC migration, phagocytosis, increase
hematopoiesis and stabilize lysozome.
• mineralocorticoids- (aldosterone) these regulate potassium metabolism and
regulate extracellular fluid volume. They also control water secretion by regulating
Na absorption and K secretion.
• androgens- (testosterone, dehydroepiandrosterone, dehydroepiandosterone
sulfate) These function in spermatogenesis and in the formation of the secondary
male sex hormones.
• estrogens- (estrogen, estradiol, estriol) these function in ovulation, embryo
preparation for implantation, and menstruation regulation
Page 38
41.

Describe the feedback mechanism for cortisol. What is the function of cortisol?
What is the role of ACTH?
• The hypothalamus is stimulated by low glucose to secrete CRH which causes the
anterior pituitary to secrete ACTH to help make 11-deoxycortisol. Cortisol is
formed and is found bound to transcortin or as free cortisol. The free cortisol
stimulates he glucose production and when the need is met it acts as a shunt to tell
the hypothalamus to stop making CRH.

42.

Diagram the renin-angiotensin-aldosterone system. What is the purpose of this
system?
• The afferent arterioles secrete rennin when there is a decrease in blood pressure.
The rennin converts angiotensinogen to Angiotensin I which is converted to
Angiotensin II. Angiotensin II stimulates aldosterone to be produced and secreted
to increase the blood pressure by retaining sodium to passively hold water and
increase the blood volume which increases the blood pressure.

43.

What hormones make up the 17-hydroxysteriods? the 17-ketosteroids? Why are
these tests performed?
• Cortisone, cortisol, and 11-dehydroxycorticoids make up 17-hydroxysteroids.
• Androgens make up the 17-keytosetriods.
• These tests are performed to observe the levels for glucosteriods, testosterone,
and other androgens.

44.

What are the catecholamines? Where are they synthesized? What is the
precursor amino acid? What is their basic function?
Catecholamines are epinephrine, norepinephrine, and dopamine. They are
synthesized in the adrenal medulla of the adrenal glands. The precursor amino acid is
tyrosine. Their basic function is to increase the blood pressure and heart rate when the
body is excited in the fight or flight mechanism. They have the ability to break down
adipose tissue and glycogen to be used for energy in the fight or flight mechanism.

•

45.
•
•

Describe catecholamine levels in the following diseases:
Pheochromocytomas-Tumors of the adrenal medulla or the sympathetic ganglia of the
adrenal glands which cause overproduction of the catecholamines.
Infantile Neuroblastoma- Tumors of the adrenal medulla that cause an increase in
catecholamines.

46.

What is the function of prolactin? When is it measured?
• Prolactin acts to initiate lactation and maintain it in a pregnant female. It is
measured to evaluate pituitary tumors, galactorrhea, and fertility.

47.

Describe levels of FSH and LH during follicular development, ovulation and the
luteal phase? Why and how are these hormones measured?
• FSH increases during follicular phase, goes down and rises again just after
ovulation, and drops slowly toward luteal phase.

Page 39
•
•

LH is baseline in follicular phase, increases in ovulation, and decreases back to
baseline during the luteal phase.
These hormones are measured by RIA to assess pituitary and gonadal axis,
fertility problems, to investigate puberty problems (late puberty), and to
investigate pituitary tumors.

48.

What is the function of gastrin? How is it related to Zollinger Ellison Syndrome?
• Gastrin functions to secrete HCl to digest food, pepsin, intrinsic factor, pancreatic
hormones, and bile from the liver.
• Zollinger Ellison (ulcers in the stomach or duodenum) syndrome has increased
Gastrin levels, so it can be separated from other peptic ulcers because they don’t
increase in production of gastrin.

49.

What is the function of the androgens? How are they measured? How is
testosterone measured?
• Androgens function in spermatogenesis and formation of secondary sex
characteristics of males. Androgens like testosterone are measured by RIA.

50.

Briefly describe the function of the estrogens and progesterone. What test is
used for total urinary estrogens? What is its principle?
• Estrogen and progesterone both function in getting the uterus ready for embryo
implantation.
• The Brown method (a colorimetric method using Kober reagent to look for a pink
color indicating that estriol is present) is used to measure total estrogen levels.

51.

What is the primary site of synthesis of estradiol, and esterone? Explain why
these two estrogens are measured?
• Estradiol is produced in the ovary of a pre-menopausal woman.
• Esterone is produced in the adrenal cortex of the adrenal glands of a postmenopausal woman.
• Both are measured to determine menstrual cycle disorders and ovulation in
women.

52.

What does the T3 uptake measure? Why is this important?
• T3 uptake measures- the # of free binding sites in the Thyroxine Binding Globulin.
• This is important because it indirectly gives the amount of thyroxine in the serum
sample.

53.

What is the free thyroxine index? T7? T12? How are they calculated? What is the
importance of this calculation.
• Free thyroxine index- estimates how much free T4 is in the blood.
• T7 and T12- are pseudonyms for FTI (T7 = T3+ T4, and T12 = T3 x T4)
• FTI- an index of thyroid status, it provides T4 and T3U which are useful in
diagnosing thyroid problems.
• Free thyroxine index = (T4) x (% T3 uptake as a decimal)
Page 40
54.

Discuss the purpose of each of the following steps in hormone determinations:
• hydrolysis- remove and solubilize attachments (chemically or enzymatically)
• purification- organic solvents purify the hormones
• extraction- centrifugation, washing, and ion exchange
• estimation- react, detect, and quantitate by various methods

55.

What is diurnal variation? How does this impact cortisol levels and the collection
of cortisol specimens?
• Diurnal variation- levels of analytes rise and fall, they peak early in the day when
most of us are asleep.
• Cortisol levels and ACTH (anterior pitutitary hormone, Cortocotropin)
• They rise between 0600 and 0800 hours, then decrease all day long. At 2000 hrs
(8 pm), the level is 2/3 of what it is at 0800 hrs, so the best analysis is made form
early morning specimens.

56.

Describe Cushing's Syndrome and Addison's disease. Include their impact on
diurnal variation, cortisol levels, glucose levels, aldosterone levels, electrolyte
and water balance. How may secondary disease states be diagnosed?
• Cushinn’s syndrome- Increased- cortisol, diurnal variation, aldosterone,
hypertension, and hyperglycemia (glucose), and decreased potassium.
• Addison’s disease- Decreased- cortisol, diurnal variation, aldoserone, hypotension,
and hypoglycemia (glucose). Increased- potassium.
• Addisons disease- Cosyntropin is given to th epatient which causes the cortisol
release, which aids in determining if patient has Addisons disease, because you
look to see if cotrisol is released by the cortisol stimulating drug. If still no release
of cortisol, it is a primary disease like addisons.
• Cushings disease- use hiht dexamethasome suppression test where there is a
suppression of urine and plasma cortisol which only occurs in cushings disease.

57.

When are plasma epinephrine and norepinephrine levels useful?
• Epinephrine and Norepinephrine levels are useful in diagnosing stress, increased
catacholamines, decreased blood pressure, decreased blood volume, thyroid
hormone deficiency, and congestive heart failure.
• Decreased catacholimines are found in hypotension.

58.

When is it valuable to quantify HCG? What trimester of pregnancy are HCG
levels used to monitor fetal health?
• It is valuable to quantitate HCG to indicate how far along in pregnancy a patient is,
or if pregnant at all. Fetal health can be determined in the 1st and 3rd trimesters by
looking at BHCG levels.

59.

What is the function of serotonin? When does it increase? What is 5-hydroxy
indolacetic acid? How is 5-HIAA measured?
• Serotonin is released during coagulation by platelets and is involved in smooth
muscle stimulation and vasoconstriction. It is increased with carcinoid tumors that
Page 41
occur in the Ileum and appendix, 5-HIAA is a metabolite of serotonin and is
excreted in the urine. It is measured colorimetrically after reacting it with 1-nitroso2-napthol and nitrois acid (purple color).
60.

Why are estriol levels measured? What trimester of pregnancy are these levels
useful in?
• Estriol levels are measured to help determine pregnancies. Levels in non pregnant
females can be measured in the ug’s, but pregnant women have levels in the mg
range. These levels are useful in the third trimester (last 4-6 weeks of pregnancy)

M. TOXICOLOGY
1.

List methods in which urines are screened for drugs of abuse? Why is urine
preferred? What is the purpose of extraction?
• You can use thin layer chromatography, gas chromatography, and immunoassays
to screen for drugs of abuse. Urine is used most often because the drugs are
filtered through the kidneys and show up in the urine.

2.

Why are chloramphenicol levels monitored? What disease process may result
from chloramphenicol.
• Chloramphenicol levels are monitored to make sure that ALA synthase is being
produced in enough quantity that heme synthase can be produced because
chloramphenicol causes a decrease in ALA synthase, heme synthatase, and DNA
synthase. This can be associated with disease processes like sideroblastic
anemia.

3.

What is the principle of the renish heavy metal test? What disease process is
associated with lead poisoning?
• The principal of the renisch heavy metal is to place a clean coiled copper wire in a
solution of 5-10 ml of gastric acid or urine with an equal amount of 2M HCL, then
place in a hot water bath for 10 minutes, let sit one hour, and examine the copper
wire for color change. ( blue or purple black- antimony; dull black- arsenic; shiny
black- bismuth; and silver gray- mercury) Lead poisioning is associated with
encephalopathy characterized by cerebral edema and hypoxia.

4.

How is carbon monoxide poisoning usually detected?
• This is usually detected by spot tests looking for carboxyhemoglobin (giving the
patient a cherry red appearance due to its color), or gas chromatography also
looking for carboxyhemoglobin.

5.

When does bromide toxicity result? How is it measured? How does bromide
affect chloride determinations?
• This toxicity results from organic and inorganic medication. It is measured by
immunoassay or thin layer chromotography, and it gives a false high in chloride
determinations.
Page 42
6.

What is the therapeutic usage of cyclosporine? methotrexate?
• Cyclosporin- this is an immunosuppressive drug that is used to suppress host vs.
graft rejection of transplant organs.
• Methotrexate- this is an antineoplastic drug that is used in therapy and involves the
rate of mitosis in normal cells versus neoplastic cells.

7.

What are digoxin? digitoxin? Why are their levels so critical?
• Digoxin- one of a group of cardiac glycosides obtained from digitalis plants which
restores the force of cardiac contraction in congestive heart failure. (drug used to
treat cardiovascular problems)
• Digitoxin- another cardiac glycoside (less common than digoxin) that is used to
treat cardiovascular problems.
• Critical levelso Low- digoxin caused the atrium to be less electrically excitable
o Moderate- reduce the rate of depolarization in the spontaneously
depolarizing conductive fibers.
o High- diminishes the depolarization of the ventricular myocardium.

8.

Define:
• therapeutic range- concentration range of a drug which is beneficial to the patient
without being toxic.
• peak level- one hour after the dose is given when the drug reaches peak
concentration in the body.
• trough level- the lowest concentration of drug obtained in the blood, drawn
immediately prior to the next dose.
• toxic value- drug levels outside of the therapeutic range.
• bioavailability- tge fraction of a drug that is absorbed into the systemic circulation.

9.

What is theophyline? When is it used?
• A bronchodialator used to treat asthma or other chronic obstructive pulmonary
diseases

N. VITAMINS

1.

What disease process is associated with decreased B12 and folic acid? What is
the relationship between B12 and folic acid?
• Pernicious anemia is associated with decreases B12 and folic acid.
• In relationship between B12 an folic acid, B12 is used in the metabolism and
needed for the synthesis of folate which is needed for the production of nucleic
acids (DNA)

Page 43
2.

What is the function of Vitamin A? How is it related to beta-carotene? How are
both measured? Why are serum beta carotene levels measured?
• Vitamin A functions in growth, dim light vision, reproduction, immunity and mucous
secretion. Beta-carotene(pro-vitamin A) is the precursor to vitamin A and is
composed of two moles of vitamin A.
• They are both measured by immunoassay or HPLC.
• Beta carotene is measured in serum to indirectly quantitate
4. (P) Describe
the methods for measuring Vitamin B12? What is the function of
cyanocobalamin?

Page 44
BLOOD GASES


Purpose
 Represents the acid/base status of entire body
 Provides information of lung function



Sample type
 Whole Blood
 Arterial Sample – ABG
 Preferred sample
 Sites are radial, femoral or brachial artery
 Venous & Capillary Blood
 Can be used, but not preferred



Assessment performed STAT
SPECIMEN COLLECTION & HANDLING
 Collected

in heparinized plastic syringe
(no air bubbles & no clots!!!)

 Often

Collected by Respiratory Therapy

 Collected

anaerobically and put on ice. Ice
serves to slow cell metabolism.

performed at 37o C, to emulate body
temperature

 Testing
PREANALYTICAL CONSIDERATIONS


Air bubbles
Causes increases in pO2, pH
 Causes decreased in pCO2




Clots




Can not run clotted whole blood on instrumentation

Glycolysis
Cell respiration causes a decrease in pH, pO2
 pCO2 increases




Temperature


pH is temperature dependent. For every 1 degree rise in
temperature, the pH decreases about 0.015 units
REFERENCE VALUES (ABG)
Component

Arterial Blood

Mixed Venous Blood

pH

7.35-7.45

7.31-7.41

pO2

80-100 mmHg

35-40 mmHg

O2 Saturation

> 95%

70-75%

pCO2

35-45 mmHg

41-51 mmHg

HCO3-

22-26 mEq/L

22-26 mEq/L

Total CO2

23-27 mmol/L

23-27 mmol/L

Base excess

-2 to +2

-2 to +2
INSTRUMENTATION


Electrochemistry




Ion Selective Electrodes

Hemoglobin Concentration


Spectrophotometry
DETERMINATION


Three components are directly measured
pH
 pO2
 pCO2




Values that can be calculated and reported include:
Total CO2 or bicarbonate ion
 Base excess
 Oxygen saturation

PH


MEASUREMENT

Measure of the hydrogen ion activity based on
bicarbonate-carbonic acid buffer system

pH electrode has a thin membrane of glass
separating two differing H+ concentrations, a H+
exchange occurs in the outer layers of the glass,
causing a potential to develop.
 A calomel half-cell or reference electrode is also
immersed in the solution.
 Both the pH and reference electrode are connected
through a pH meter. The meter can measure
voltage difference between the two and convert to
pH units.

PO2 MEASUREMENT
Partial pressure of oxygen in the blood
 Measured by the O2 electrode to determine oxygen
content
 pO2 electrode or Clark electrode measures the
current that flows when a constant voltage is
applied to the system
 As dissolved O2 diffuses from the blood a change in
current occurs which offers a direct pO2
measurement

PCO2

MEASUREMENT

Partial pressure of carbon dioxide in the blood
 pCO2 measured in mmHg x 0.03 indicates carbonic
acid (H2CO3)


pCO2 > 50 mmHg = HYPO ventilation
 pCO2< 30 mmHg= HYPER ventilation

PCO2

MEASUREMENT

The pCO2 electrode or Severinghaus electrode
consists of a pH electrode with a CO2 permeable
membrane covering the glass surface. Between the
two is a thin layer of dilute bicarbonate buffer.
 Once the blood contacts the membrane and the
CO2 diffuses into the buffer, the pH of the buffer is
lowered
 Change in pH is proportional to the concentration of
dissolved CO2 in the blood

SiggaardAnderson
nomogram
CALCULATED PARAMETERS


Siggaard-Anderson nomogram
Base Excess
 Total CO2 and bicarbonate concentration

BASE EXCESS
Determination of amount of base in the blood
 Determines the source of acid-base disturbance
 Base deficit usually indicates metabolic acidosis




Causes of:
Excess bicarbonate
 Deficit of bicarbonate

O2 SATURATION


Calculation/Derived




Requires measured pH and pO2 values

Measured


Requires a hgb measurement usually obtained by cooximetry


Co-oximetry: measuring at multiple wavelengths to get light
absorption spectra
REFERENCES








Bishop, M., Fody, E., & Schoeff, l. (2010). Clinical Chemistry:
Techniques, principles, Correlations. Baltimore: Wolters
Kluwer Lippincott Williams & Wilkins.
Carreiro-Lewandowski, E. (2008). Blood Gas Analysis and
Interpretation. Denver, Colorado: Colorado Association for
Continuing Medical Laboratory Education, Inc.
Jarreau, P. (2005). Clinical Laboratory Science Review (3rd
ed.). New Orleans, LA: LSU Health Science Center.
Sunheimer, R., & Graves, L. (2010). Clinical Laboratory
Chemistry. Upper Saddle River: Pearson .

16
ELECTROLYTES


Electrolytes
 Substances whose molecules dissociate into ions
when they are placed in water.
 Osmotically active particles
 Classification of ions: by charge


CATIONS (+)





In an electrical field, move toward the cathode
Sodium (Na), Potassium (K), Calcium(Ca), Magnesium(Mg)

ANIONS (-)



In an electrical field, move toward the anode
Chloride(Cl), Bicarbonate, PO4, Sulfate
2
ELECTROLYTES
 General




dietary requirements

Most need to be consumed only in small
amounts as utilized
Excessive intake leads to increased excretion via
kidneys
Excessive loss may result in need for corrective
therapy


loss due to vomiting / diarrhea; therapy required - IV
replacement, Pedilyte, etc.

3
ELECTROLYTE FUNCTIONS
Volume and osmotic regulation
 Myocardial rhythm and contractility
 Cofactors in enzyme activation
 Regulation of ATPase ion pumps
 Acid-base balance
 Blood coagulation
 Neuromuscular excitability
 Production of ATP from glucose


4
ELECTROLYTE PANEL


Panel consists of:
 sodium (Na)
 potassium (K)
 chloride (Cl)
 bicarbonate CO2 (in its ion form = HCO3- )

5
ANALYTES OF THE ELECTROLYTE PANEL
 Sodium




(Na)–

the major cation of extracellular fluid
Most abundant (90 %) extracellular cation
Diet


Easily absorbed from many foods

6
FUNCTION: SODIUM






Influence on regulation of body water
 Osmotic activity
 Sodium determines osmotic activity
 Main contributor to plasma osmolality
Neuromuscular excitability
 extremes in concentration can result in neuromuscular
symptoms
Na-K ATP-ase Pump
 pumps Na out and K into cells
 Without this active transport pump, the cells would fill with
Na+ and subsequent osmotic pressure would rupture the
cells

7
REGULATION OF SODIUM




Concentration depends on:
 intake of water in response to thirst
 excretion of water due to blood volume or osmolality
changes
Renal regulation of sodium
 Kidneys can conserve or excrete Na+ depending on ECF
and blood volume
 by aldosterone
 and the renin-angiotensin system


this system will stimulate the adrenal cortex to secrete
aldosterone.
8
REFERENCE RANGES:
SODIUM


Serum




136-145 mEq/L or mmol/L

Urine (24 hour collection)


40-220 mEq/L

9
SODIUM
 Urine

testing & calculation:



Because levels are often increased, a dilution of the
urine specimen is usually required.



Once a number is obtained, it is multiplied by the
dilution factor and reported as (mEq/L or mmol/L) in
24 hr.

10
DISORDERS OF SODIUM HOMEOSTASIS




Hyponatremia: < 136 mmol/L
 Causes of:
 Increased Na+ loss
 Increased water retention
 Water imbalance
Hypernatremia:> 150 mmol/L
 Causes of:
 Excess water loss
 Increased intake/retention
 Decreased water intake

11
HYPONATREMIA
1.

Increased Na+ loss
 Aldosterone deficiency
 hypoadrenalism
 Diabetes mellitus
 In acidosis of diabetes, Na is excreted with
ketones
 Potassium depletion
 K normally excreted , if none, then Na
 Loss of gastric contents
12
HYPONATREMIA
2.

Increased water retention
Dilution of plasma Na+
Renal failure
Nephrotic syndrome
Hepatic cirrhosis
Congestive heart failure
13
HYPONATREMIA
3.

Water imbalance
Excess water intake
Chronic condition

14
SODIUM
Note:
 Increased lipids or proteins may cause false
decrease in results. This would be classified as
artifactual/pseudo-hyponatremia

15
CLINICAL SYMPTOMS OF HYPONATREMIA


Depends on the serum level


Can affect
GI tract
 Neurological
 Nausea, vomiting, headache, seizures,coma


16
HYPERNATREMIA
1.

Excess water loss
 Sweating
 Diarrhea
 Burns
 Diabetes insipidus

17
HYPERNATREMIA
Increased intake/retention

2.
•

Excessive IV therapy
Decreased water intake

3.
•
•
•

Elderly
Infants
Mental impairment

18
CLINICAL SYMPTOMS OF HYPERNATREMIA


Involve the CNS






Altered mental status
Lethargy
Irritability
Vomiting
Nausea

19
SPECIMEN COLLECTION: SODIUM
Serum (sl hemolysis is OK, but not gross)
 Heparinized plasma
 Timed and random urine
 Sweat
 GI fluids
 Liquid feces (would be only time of excessive
loss)


20
ANALYTES OF THE ELECTROLYTE PANEL
Potassium


(K+)

the major cation of intracellular fluid
Only 2 % of potassium is in the plasma
 Potassium concentration inside cells is 20 X greater than it is
outside.
 This is maintained by the Na-K pump
 exchanges 3 Na for 1 K




Diet


easily consumed by food products such as bananas

21
FUNCTION: POTASSIUM
 Critically

important to the functions of
neuromuscular cells
 Acid-base balance
 Intracellular fluid volume
 Controls heart muscle contraction
 Promotes muscular excitability
Decreased potassium decreases
excitability (paralysis and arrhythmias)
22
REGULATION OF POTASSIUM


Kidneys


Responsible for regulation. Potassium is readily
excreted, but gets reabsorbed in the proximal tubule under the control of ALDOSTERONE

Diet
 Cell Uptake/Exchange


23
REFERENCE RANGES:
POTASSIUM


Serum (adults)




Newborns




3.5 - 5.1 mEq/L or mmol/L
3.7 - 5.9 mEq/L

Urine (24 hour collection)


25 - 125 mEq/L

24
DISORDERS OF POTASSIUM
HOMEOSTASIS




Hypokalemia
 < 3.5 mmol/L
 Causes of:
 Non-renal loss
 Renal Loss
 Cellular Shift
 Decreased intake
Hyperkalemia
 >5.1 mmol/L
 Causes of
 Decreased renal excretion
 Cellular shift
 Increased intake
 Artifactual/False elevations

25
HYPOKALEMIA
1.

Non-renal loss
 Excessive fluid loss ( diarrhea, vomiting,
diuretics )
 Increased Aldosterone promote Na reabsorption
… K is excreted in its place

26
HYPOKALEMIA
2.

Renal Loss
 Nephritis, renal tubular acidosis,
hyperaldosteronism, Cushing’s
Syndrome

3.

Cellular Shift
 Alkalosis, insulin overdose

4.

Decreased intake

27
MECHANISM OF HYPOKALEMIA


Increased plasma pH ( decreased Hydrogen ion )

RBC
H+
K+
K+ moves into RBCs to preserve electrical balance,
causing plasma potassium to decrease.
28
( Sodium also shows a slight decrease )
CLINICAL SYMPTOMS OF HYPOKALEMIA
Neuromuscular weakness
 Cardiac arrhythmia
 Constipation


29
HYPERKALEMIA
1.

Decreased renal excretion




2.

Renal disease
Addison’s disease
Hypoaldosteronism

Cellular Shift



Such as acidosis, chemotherapy, leukemia,
muscle/cellular injury
Hydrogen ions compete with potassium to get into
the cells

30
HYPERKALEMIA
3.

Increased intake


4.

Insulin IVs promote rapid cellular potassium uptake

Artifactual
•
Sample hemolysis
•
Prolonged tourniquet use
•
Excessive fist clenching

31
CLINICAL SYMPTOMS OF HYPERKALEMIA
Muscle weakness
 Tingling
 Numbness
 Mental confusion
 Cardiac arrhythmias
 Cardiac arrest


32
SPECIMEN COLLECTION:POTASSIUM
Non-hemolyzed serum
 heparinized plasma
 24 hr urine


33
ANALYTES OF THE ELECTROLYTE PANEL
 Chloride


(Cl-)

the major anion of extracellular fluid



Chloride moves passively with Na+ or against
HCO3- to maintain neutral electrical charge



Chloride usually follows Na


if one is abnormal, so is the other

34
FUNCTION: CHLORIDE
 Body

hydration/water balance
 Osmotic pressure
 Electrical neutrality

35
REGULATION OF CHLORIDE


Regulation via diet and kidneys
In the kidney, Cl is reabsorbed in the renal proximal
tubules, along with sodium.
 Deficiencies of either one limits the reabsorption of the
other.


36
REFERENCE RANGES: CHLORIDE


Serum
 98 -107 mEq/L or mmol/L



24 hour urine
 110-250 mEq/L
 varies with intake



CSF
 120 - 132 mEq/L
 Often CSF Cl is decreased when CSF protein is increased,
as often occurs in bacterial meningitis.
37
DETERMINATION: CHLORIDE


Specimen type






Serum
Plasma
24 hour urine
CSF
Sweat


Sweat Chloride Test
 Used to identify cystic fibrosis patients




Increased salt concentration in sweat
Pilocarpine= chemical used to stimulate sweat production
Iontophoresis= mild electrical current that stimulates sweat
production
DISORDERS OF CHLORIDE HOMEOSTASIS


Hypochloremia
 Decreased blood chloride
 Causes of :
 Conditions where output exceeds input



Hyperchloremia
 Increased blood chloride
 Causes of:
 Conditions where input exceeds output

39
HYPOCHLOREMIA


Decreased serum Cl
loss of gastric HCl
 salt loosing renal diseases
 metabolic alkalosis/compensated respiratory acidosis




increased HCO3- & decreased Cl-

40
HYPERCHLOREMIA


Increased serum Cl






dehydration (relative increase)
excessive intake (IV)
congestive heart failure
renal tubular disease
metabolic acidosis


decreased HCO3- & increased Cl-

41
SPECIMEN COLLECTION: CHLORIDE
Serum
 Heparinized plasma
 24 hr urine
 Sweat


42
ANALYTES OF THE ELECTROLYTE PANEL


Carbon dioxide/bicarbonate (HCO3-)
 the major anion of intracellular fluid
 2nd most abundant anion of extracellular fluid
 Total plasma CO2= HCO3- + H2CO3- + CO2


HCO3- (bicarbonate ion)
 accounts for 90% of total plasma CO2



H2CO3- (carbonic acid)
43
FUNCTION:
BICARBONATE ION


CO2 is a waste product
 continuously produced as a result of cell metabolism,
 the ability of the bicarbonate ion to accept a hydrogen
ion makes it an efficient and effective means of
buffering body pH
 dominant buffering system of plasma

44
REGULATION OF
BICARBONATE ION
 Bicarbonate

is regulated by
secretion / reabsorption of the renal
tubules
 Acidosis :
↓ renal excretion
 Alkalosis
:
↑ renal excretion

45
REGULATION OF BICARBONATE
ION


Kidney regulation requires the enzyme carbonic anhydrase which is present in renal tubular cells & RBCs

Reaction: CO2 + H2O ⇋ H2CO3 → H+ + HCO–3
carbonic anhydrase

Pulmonary Control
Renal
Control

46
REFERENCE RANGE:
BICARBONATE ION


Total Carbon dioxide (venous)


23-29 mEq/L or mmol/L




includes bicarb, dissolved & undissociated H2CO3 - carbonic
acid (bicarbonate)

Bicarbonate ion (HCO3–)


22-26 mEq/L or mmol/L

47
SPECIMEN COLLECTION: BICARBONATE ION
heparinized plasma
 arterial whole blood
 fresh serum
 Anaerobic collection preferred


48
ELECTROLYTE BALANCE
Anion gap – an estimate of the
unmeasured anion concentrations such
as sulfate, phosphate, and various
organic acids.



49
ELECTROLYTE SUMMARY



cations (+)
Na 142
 K
5
 Ca
5
 Mg
2
154 mEq/L




anions (-)
Cl
105
 HCO324
 HPO422
 SO4-2
1
 organic acids
6
 proteins 16


154 mEq/L

50
ANION GAP


Anion Gap Calculations
1.

Na - (Cl + CO2 or HCO3-)


Reference range: 7-16 mEq/L

Or

2.

(Na + K) - (Cl + CO2 or HCO3-)


Reference range: 10-20 mEq/L

51
FUNCTIONS OF THE ANION GAP


Causes in normal patients
 what causes the anion gap?




Increased AG –







2/3 plasma proteins & 1/3 phosphate& sulfate ions, along
with organic acids

uncontrolled diabetes (due to lactic & keto acids)
severe renal disorders
Hypernatremia
lab error

Decreased AG 52



a decrease AG is rare, more often it occurs when one
test/instrument error
REFERENCES







Bishop, M., Fody, E., & Schoeff, l. (2010). Clinical Chemistry:
Techniques, principles, Correlations. Baltimore: Wolters
Kluwer Lippincott Williams & Wilkins.
http://thejunction.net/2009/04/11/the-how-to-authority-fordonating-blood-plasma/
http://www.nlm.nih.gov/medlineplus/ency/article/002350.htm
Sunheimer, R., & Graves, L. (2010). Clinical Laboratory
Chemistry. Upper Saddle River: Pearson .

53
Carbohydrate Metabolism and Glucose Regulation
Carbohydrate Metabolism and Glucose Regulation
Carbohydrate Metabolism and Glucose Regulation
Carbohydrate Metabolism and Glucose Regulation
Carbohydrate Metabolism and Glucose Regulation
Carbohydrate Metabolism and Glucose Regulation
Carbohydrate Metabolism and Glucose Regulation
Carbohydrate Metabolism and Glucose Regulation
Carbohydrate Metabolism and Glucose Regulation
Carbohydrate Metabolism and Glucose Regulation
Carbohydrate Metabolism and Glucose Regulation
Carbohydrate Metabolism and Glucose Regulation
Carbohydrate Metabolism and Glucose Regulation
Carbohydrate Metabolism and Glucose Regulation
Carbohydrate Metabolism and Glucose Regulation
Carbohydrate Metabolism and Glucose Regulation
Carbohydrate Metabolism and Glucose Regulation
Carbohydrate Metabolism and Glucose Regulation
Carbohydrate Metabolism and Glucose Regulation
Carbohydrate Metabolism and Glucose Regulation
Carbohydrate Metabolism and Glucose Regulation
Carbohydrate Metabolism and Glucose Regulation
Carbohydrate Metabolism and Glucose Regulation
Carbohydrate Metabolism and Glucose Regulation
Carbohydrate Metabolism and Glucose Regulation
Carbohydrate Metabolism and Glucose Regulation
Carbohydrate Metabolism and Glucose Regulation
Carbohydrate Metabolism and Glucose Regulation
Carbohydrate Metabolism and Glucose Regulation
Carbohydrate Metabolism and Glucose Regulation
Carbohydrate Metabolism and Glucose Regulation
Carbohydrate Metabolism and Glucose Regulation
Carbohydrate Metabolism and Glucose Regulation
Carbohydrate Metabolism and Glucose Regulation
Carbohydrate Metabolism and Glucose Regulation
Carbohydrate Metabolism and Glucose Regulation
Carbohydrate Metabolism and Glucose Regulation
Carbohydrate Metabolism and Glucose Regulation
Carbohydrate Metabolism and Glucose Regulation
Carbohydrate Metabolism and Glucose Regulation
Carbohydrate Metabolism and Glucose Regulation
Carbohydrate Metabolism and Glucose Regulation
Carbohydrate Metabolism and Glucose Regulation
Carbohydrate Metabolism and Glucose Regulation
Carbohydrate Metabolism and Glucose Regulation
Carbohydrate Metabolism and Glucose Regulation
Carbohydrate Metabolism and Glucose Regulation
Carbohydrate Metabolism and Glucose Regulation
Carbohydrate Metabolism and Glucose Regulation
Carbohydrate Metabolism and Glucose Regulation
Carbohydrate Metabolism and Glucose Regulation
Carbohydrate Metabolism and Glucose Regulation
Carbohydrate Metabolism and Glucose Regulation
Carbohydrate Metabolism and Glucose Regulation
Carbohydrate Metabolism and Glucose Regulation
Carbohydrate Metabolism and Glucose Regulation
Carbohydrate Metabolism and Glucose Regulation
Carbohydrate Metabolism and Glucose Regulation
Carbohydrate Metabolism and Glucose Regulation
Carbohydrate Metabolism and Glucose Regulation
Carbohydrate Metabolism and Glucose Regulation
Carbohydrate Metabolism and Glucose Regulation
Carbohydrate Metabolism and Glucose Regulation
Carbohydrate Metabolism and Glucose Regulation
Carbohydrate Metabolism and Glucose Regulation
Carbohydrate Metabolism and Glucose Regulation
Carbohydrate Metabolism and Glucose Regulation
Carbohydrate Metabolism and Glucose Regulation
Carbohydrate Metabolism and Glucose Regulation
Carbohydrate Metabolism and Glucose Regulation
Carbohydrate Metabolism and Glucose Regulation
Carbohydrate Metabolism and Glucose Regulation
Carbohydrate Metabolism and Glucose Regulation
Carbohydrate Metabolism and Glucose Regulation
Carbohydrate Metabolism and Glucose Regulation
Carbohydrate Metabolism and Glucose Regulation
Carbohydrate Metabolism and Glucose Regulation
Carbohydrate Metabolism and Glucose Regulation
Carbohydrate Metabolism and Glucose Regulation
Carbohydrate Metabolism and Glucose Regulation
Carbohydrate Metabolism and Glucose Regulation
Carbohydrate Metabolism and Glucose Regulation
Carbohydrate Metabolism and Glucose Regulation
Carbohydrate Metabolism and Glucose Regulation
Carbohydrate Metabolism and Glucose Regulation
Carbohydrate Metabolism and Glucose Regulation
Carbohydrate Metabolism and Glucose Regulation
Carbohydrate Metabolism and Glucose Regulation
Carbohydrate Metabolism and Glucose Regulation
Carbohydrate Metabolism and Glucose Regulation
Carbohydrate Metabolism and Glucose Regulation
Carbohydrate Metabolism and Glucose Regulation
Carbohydrate Metabolism and Glucose Regulation
Carbohydrate Metabolism and Glucose Regulation
Carbohydrate Metabolism and Glucose Regulation
Carbohydrate Metabolism and Glucose Regulation
Carbohydrate Metabolism and Glucose Regulation
Carbohydrate Metabolism and Glucose Regulation
Carbohydrate Metabolism and Glucose Regulation
Carbohydrate Metabolism and Glucose Regulation
Carbohydrate Metabolism and Glucose Regulation
Carbohydrate Metabolism and Glucose Regulation
Carbohydrate Metabolism and Glucose Regulation
Carbohydrate Metabolism and Glucose Regulation
Carbohydrate Metabolism and Glucose Regulation
Carbohydrate Metabolism and Glucose Regulation
Carbohydrate Metabolism and Glucose Regulation
Carbohydrate Metabolism and Glucose Regulation
Carbohydrate Metabolism and Glucose Regulation
Carbohydrate Metabolism and Glucose Regulation
Carbohydrate Metabolism and Glucose Regulation
Carbohydrate Metabolism and Glucose Regulation
Carbohydrate Metabolism and Glucose Regulation
Carbohydrate Metabolism and Glucose Regulation
Carbohydrate Metabolism and Glucose Regulation
Carbohydrate Metabolism and Glucose Regulation
Carbohydrate Metabolism and Glucose Regulation
Carbohydrate Metabolism and Glucose Regulation
Carbohydrate Metabolism and Glucose Regulation
Carbohydrate Metabolism and Glucose Regulation
Carbohydrate Metabolism and Glucose Regulation
Carbohydrate Metabolism and Glucose Regulation
Carbohydrate Metabolism and Glucose Regulation
Carbohydrate Metabolism and Glucose Regulation
Carbohydrate Metabolism and Glucose Regulation
Carbohydrate Metabolism and Glucose Regulation
Carbohydrate Metabolism and Glucose Regulation
Carbohydrate Metabolism and Glucose Regulation
Carbohydrate Metabolism and Glucose Regulation
Carbohydrate Metabolism and Glucose Regulation
Carbohydrate Metabolism and Glucose Regulation
Carbohydrate Metabolism and Glucose Regulation
Carbohydrate Metabolism and Glucose Regulation
Carbohydrate Metabolism and Glucose Regulation
Carbohydrate Metabolism and Glucose Regulation
Carbohydrate Metabolism and Glucose Regulation
Carbohydrate Metabolism and Glucose Regulation
Carbohydrate Metabolism and Glucose Regulation
Carbohydrate Metabolism and Glucose Regulation
Carbohydrate Metabolism and Glucose Regulation
Carbohydrate Metabolism and Glucose Regulation
Carbohydrate Metabolism and Glucose Regulation
Carbohydrate Metabolism and Glucose Regulation
Carbohydrate Metabolism and Glucose Regulation
Carbohydrate Metabolism and Glucose Regulation
Carbohydrate Metabolism and Glucose Regulation
Carbohydrate Metabolism and Glucose Regulation
Carbohydrate Metabolism and Glucose Regulation
Carbohydrate Metabolism and Glucose Regulation
Carbohydrate Metabolism and Glucose Regulation
Carbohydrate Metabolism and Glucose Regulation
Carbohydrate Metabolism and Glucose Regulation
Carbohydrate Metabolism and Glucose Regulation
Carbohydrate Metabolism and Glucose Regulation
Carbohydrate Metabolism and Glucose Regulation
Carbohydrate Metabolism and Glucose Regulation
Carbohydrate Metabolism and Glucose Regulation
Carbohydrate Metabolism and Glucose Regulation
Carbohydrate Metabolism and Glucose Regulation
Carbohydrate Metabolism and Glucose Regulation
Carbohydrate Metabolism and Glucose Regulation
Carbohydrate Metabolism and Glucose Regulation
Carbohydrate Metabolism and Glucose Regulation
Carbohydrate Metabolism and Glucose Regulation
Carbohydrate Metabolism and Glucose Regulation
Carbohydrate Metabolism and Glucose Regulation
Carbohydrate Metabolism and Glucose Regulation
Carbohydrate Metabolism and Glucose Regulation
Carbohydrate Metabolism and Glucose Regulation
Carbohydrate Metabolism and Glucose Regulation
Carbohydrate Metabolism and Glucose Regulation
Carbohydrate Metabolism and Glucose Regulation
Carbohydrate Metabolism and Glucose Regulation
Carbohydrate Metabolism and Glucose Regulation
Carbohydrate Metabolism and Glucose Regulation
Carbohydrate Metabolism and Glucose Regulation
Carbohydrate Metabolism and Glucose Regulation
Carbohydrate Metabolism and Glucose Regulation
Carbohydrate Metabolism and Glucose Regulation
Carbohydrate Metabolism and Glucose Regulation
Carbohydrate Metabolism and Glucose Regulation
Carbohydrate Metabolism and Glucose Regulation
Carbohydrate Metabolism and Glucose Regulation
Carbohydrate Metabolism and Glucose Regulation
Carbohydrate Metabolism and Glucose Regulation
Carbohydrate Metabolism and Glucose Regulation
Carbohydrate Metabolism and Glucose Regulation
Carbohydrate Metabolism and Glucose Regulation

More Related Content

What's hot

clinical chemistry
clinical chemistry clinical chemistry
clinical chemistry Moha Haji
 
PANCREATIC FUNCTION TESTS
PANCREATIC FUNCTION TESTSPANCREATIC FUNCTION TESTS
PANCREATIC FUNCTION TESTSYESANNA
 
LIVER FUNCTION TEST (ENZYME PART)
LIVER FUNCTION TEST (ENZYME PART)LIVER FUNCTION TEST (ENZYME PART)
LIVER FUNCTION TEST (ENZYME PART)Yaalok
 
INTERFERENCES IN CLINICAL CHEMISTRY ANALYSIS
INTERFERENCES IN CLINICAL CHEMISTRY  ANALYSISINTERFERENCES IN CLINICAL CHEMISTRY  ANALYSIS
INTERFERENCES IN CLINICAL CHEMISTRY ANALYSISRaffat Al Telbani
 
Basic Principles and Practice of Clinical Chemistry quality management lec 2...
 Basic Principles and Practice of Clinical Chemistry quality management lec 2... Basic Principles and Practice of Clinical Chemistry quality management lec 2...
Basic Principles and Practice of Clinical Chemistry quality management lec 2...biochembiochem
 
Estimation of Serum Urea
Estimation of Serum UreaEstimation of Serum Urea
Estimation of Serum UreaASHIKH SEETHY
 
Estimation of Blood Urea Nitrogen by Dr. Tehmas
Estimation of Blood Urea Nitrogen by Dr. TehmasEstimation of Blood Urea Nitrogen by Dr. Tehmas
Estimation of Blood Urea Nitrogen by Dr. TehmasTehmas Ahmad
 
Automation in Biochemistry (Autoanalyzers)
Automation in Biochemistry (Autoanalyzers)Automation in Biochemistry (Autoanalyzers)
Automation in Biochemistry (Autoanalyzers)Pradeep Singh Narwat
 
Gastric and Pancreatic function tests
Gastric  and Pancreatic function testsGastric  and Pancreatic function tests
Gastric and Pancreatic function testsMohit Adhikary
 
urea, creatinine,inulin clearance test.pptx
urea, creatinine,inulin clearance test.pptxurea, creatinine,inulin clearance test.pptx
urea, creatinine,inulin clearance test.pptxmrithyunjeyan
 
Clinical chemistry lecture slide show
Clinical chemistry lecture slide showClinical chemistry lecture slide show
Clinical chemistry lecture slide showGhie Santos
 
Metabolism of ketone bodies
Metabolism of ketone bodiesMetabolism of ketone bodies
Metabolism of ketone bodiesshivaakumar
 
Gastric function test
Gastric function testGastric function test
Gastric function testbinaya tamang
 

What's hot (20)

clinical chemistry
clinical chemistry clinical chemistry
clinical chemistry
 
Billirubin estimation
Billirubin estimationBillirubin estimation
Billirubin estimation
 
Bilirubin estimation
Bilirubin estimationBilirubin estimation
Bilirubin estimation
 
PANCREATIC FUNCTION TESTS
PANCREATIC FUNCTION TESTSPANCREATIC FUNCTION TESTS
PANCREATIC FUNCTION TESTS
 
LIVER FUNCTION TEST (ENZYME PART)
LIVER FUNCTION TEST (ENZYME PART)LIVER FUNCTION TEST (ENZYME PART)
LIVER FUNCTION TEST (ENZYME PART)
 
Lipids methodology
Lipids methodologyLipids methodology
Lipids methodology
 
INTERFERENCES IN CLINICAL CHEMISTRY ANALYSIS
INTERFERENCES IN CLINICAL CHEMISTRY  ANALYSISINTERFERENCES IN CLINICAL CHEMISTRY  ANALYSIS
INTERFERENCES IN CLINICAL CHEMISTRY ANALYSIS
 
gamma GT
gamma GTgamma GT
gamma GT
 
Glucose estimation
Glucose estimationGlucose estimation
Glucose estimation
 
Basic Principles and Practice of Clinical Chemistry quality management lec 2...
 Basic Principles and Practice of Clinical Chemistry quality management lec 2... Basic Principles and Practice of Clinical Chemistry quality management lec 2...
Basic Principles and Practice of Clinical Chemistry quality management lec 2...
 
Estimation of Serum Urea
Estimation of Serum UreaEstimation of Serum Urea
Estimation of Serum Urea
 
Estimation of serum urea
Estimation of serum ureaEstimation of serum urea
Estimation of serum urea
 
Estimation of Blood Urea Nitrogen by Dr. Tehmas
Estimation of Blood Urea Nitrogen by Dr. TehmasEstimation of Blood Urea Nitrogen by Dr. Tehmas
Estimation of Blood Urea Nitrogen by Dr. Tehmas
 
Automation in Biochemistry (Autoanalyzers)
Automation in Biochemistry (Autoanalyzers)Automation in Biochemistry (Autoanalyzers)
Automation in Biochemistry (Autoanalyzers)
 
Gastric and Pancreatic function tests
Gastric  and Pancreatic function testsGastric  and Pancreatic function tests
Gastric and Pancreatic function tests
 
urea, creatinine,inulin clearance test.pptx
urea, creatinine,inulin clearance test.pptxurea, creatinine,inulin clearance test.pptx
urea, creatinine,inulin clearance test.pptx
 
Clinical chemistry lecture slide show
Clinical chemistry lecture slide showClinical chemistry lecture slide show
Clinical chemistry lecture slide show
 
Kidney function test
Kidney function testKidney function test
Kidney function test
 
Metabolism of ketone bodies
Metabolism of ketone bodiesMetabolism of ketone bodies
Metabolism of ketone bodies
 
Gastric function test
Gastric function testGastric function test
Gastric function test
 

Viewers also liked

1. must to know in clinical chemistry 001
1. must to know in clinical chemistry 0011. must to know in clinical chemistry 001
1. must to know in clinical chemistry 001Nhil Abaya
 
Compilation of previous board examination questions
Compilation of previous board examination questionsCompilation of previous board examination questions
Compilation of previous board examination questionsRhem Rick Corpuz
 
Introduction to clinical chemistry
Introduction to clinical chemistryIntroduction to clinical chemistry
Introduction to clinical chemistrygoogle
 
Micro part1 study guide
Micro part1 study guideMicro part1 study guide
Micro part1 study guideDonna Kim
 
Medical Laboratory Scientist-microbiology summary
Medical Laboratory Scientist-microbiology summaryMedical Laboratory Scientist-microbiology summary
Medical Laboratory Scientist-microbiology summaryProtegeNithi
 
Clinical Chemistry
Clinical ChemistryClinical Chemistry
Clinical Chemistryguestbc65a9
 
The clinical laboratory
The clinical laboratoryThe clinical laboratory
The clinical laboratorypayneje
 
Clinical Chemistry Instrumentation ppt
Clinical Chemistry Instrumentation pptClinical Chemistry Instrumentation ppt
Clinical Chemistry Instrumentation pptJenaIsle
 
Laboratory Investigations
Laboratory InvestigationsLaboratory Investigations
Laboratory InvestigationsDr. Saurabh Roy
 
A case of rapidly progressive generalised weakness
A case of rapidly progressive generalised weaknessA case of rapidly progressive generalised weakness
A case of rapidly progressive generalised weaknessTikal Kansara
 
Biochemical tests in clinical medicine lect1
Biochemical tests in clinical medicine lect1Biochemical tests in clinical medicine lect1
Biochemical tests in clinical medicine lect1MUDASSAR ANWER
 
Dialysis statistics(Jan-April'11)
Dialysis statistics(Jan-April'11)Dialysis statistics(Jan-April'11)
Dialysis statistics(Jan-April'11)Vishal Golay
 
History of mt laws and code of ethics
History of mt laws and code of ethicsHistory of mt laws and code of ethics
History of mt laws and code of ethicsephyloveslav
 

Viewers also liked (16)

1. must to know in clinical chemistry 001
1. must to know in clinical chemistry 0011. must to know in clinical chemistry 001
1. must to know in clinical chemistry 001
 
Compilation of previous board examination questions
Compilation of previous board examination questionsCompilation of previous board examination questions
Compilation of previous board examination questions
 
Introduction to clinical chemistry
Introduction to clinical chemistryIntroduction to clinical chemistry
Introduction to clinical chemistry
 
Micro part1 study guide
Micro part1 study guideMicro part1 study guide
Micro part1 study guide
 
Microbiology Q & A for exam (short note)
Microbiology Q & A  for exam (short note)Microbiology Q & A  for exam (short note)
Microbiology Q & A for exam (short note)
 
Medical Laboratory Scientist-microbiology summary
Medical Laboratory Scientist-microbiology summaryMedical Laboratory Scientist-microbiology summary
Medical Laboratory Scientist-microbiology summary
 
Clinical Chemistry
Clinical ChemistryClinical Chemistry
Clinical Chemistry
 
The clinical laboratory
The clinical laboratoryThe clinical laboratory
The clinical laboratory
 
laboratory investigations
 laboratory  investigations laboratory  investigations
laboratory investigations
 
Clinical Chemistry Instrumentation ppt
Clinical Chemistry Instrumentation pptClinical Chemistry Instrumentation ppt
Clinical Chemistry Instrumentation ppt
 
Laboratory Investigations
Laboratory InvestigationsLaboratory Investigations
Laboratory Investigations
 
A case of rapidly progressive generalised weakness
A case of rapidly progressive generalised weaknessA case of rapidly progressive generalised weakness
A case of rapidly progressive generalised weakness
 
Biochemical tests in clinical medicine lect1
Biochemical tests in clinical medicine lect1Biochemical tests in clinical medicine lect1
Biochemical tests in clinical medicine lect1
 
Dialysis statistics(Jan-April'11)
Dialysis statistics(Jan-April'11)Dialysis statistics(Jan-April'11)
Dialysis statistics(Jan-April'11)
 
History of mt laws and code of ethics
History of mt laws and code of ethicsHistory of mt laws and code of ethics
History of mt laws and code of ethics
 
MLT Resume4
MLT Resume4MLT Resume4
MLT Resume4
 

Similar to Carbohydrate Metabolism and Glucose Regulation

Regulation of blood Glucose Part-I.pptx
Regulation of blood Glucose Part-I.pptxRegulation of blood Glucose Part-I.pptx
Regulation of blood Glucose Part-I.pptxABHIJIT BHOYAR
 
carbohydrate metabolism -bpt.pptx
carbohydrate metabolism -bpt.pptxcarbohydrate metabolism -bpt.pptx
carbohydrate metabolism -bpt.pptxAdarshBenny2
 
CARBOHYDRATE METABOLISM and Disorders.pdf
CARBOHYDRATE METABOLISM and Disorders.pdfCARBOHYDRATE METABOLISM and Disorders.pdf
CARBOHYDRATE METABOLISM and Disorders.pdfChetnaBhatotiya
 
Gluconeogenesis and Control of Blood Glucose.pptx
Gluconeogenesis and Control of Blood Glucose.pptxGluconeogenesis and Control of Blood Glucose.pptx
Gluconeogenesis and Control of Blood Glucose.pptxAssiddiqah
 
Glucose Metabolism and Diabetes
Glucose Metabolism and DiabetesGlucose Metabolism and Diabetes
Glucose Metabolism and DiabetesAmany Elsayed
 
Glucose metabolism.pptx
Glucose metabolism.pptxGlucose metabolism.pptx
Glucose metabolism.pptxNabdNabd
 
Regulation of blood glucose concentration
Regulation of blood glucose concentrationRegulation of blood glucose concentration
Regulation of blood glucose concentrationRamesh Gupta
 
Class 4 gluconeogenesis
Class 4 gluconeogenesisClass 4 gluconeogenesis
Class 4 gluconeogenesisDhiraj Trivedi
 
Blood Glucose Homeostasis.pptx
Blood Glucose Homeostasis.pptxBlood Glucose Homeostasis.pptx
Blood Glucose Homeostasis.pptxganeshbond
 
regulation of blood sugar
regulation of blood sugar regulation of blood sugar
regulation of blood sugar Bala Vidyadhar
 
Glucose-Determination-25-3-2013.ppt
Glucose-Determination-25-3-2013.pptGlucose-Determination-25-3-2013.ppt
Glucose-Determination-25-3-2013.pptDrRhutaShah1
 
Unit 7 : Carbohydrates metabolism & disorders
Unit 7 : Carbohydrates metabolism & disordersUnit 7 : Carbohydrates metabolism & disorders
Unit 7 : Carbohydrates metabolism & disordersDrElhamSharif
 
CHO Metabolism.ppt
CHO Metabolism.pptCHO Metabolism.ppt
CHO Metabolism.pptAnnaKhurshid
 

Similar to Carbohydrate Metabolism and Glucose Regulation (20)

Regulation of blood Glucose Part-I.pptx
Regulation of blood Glucose Part-I.pptxRegulation of blood Glucose Part-I.pptx
Regulation of blood Glucose Part-I.pptx
 
carbohydrate metabolism -bpt.pptx
carbohydrate metabolism -bpt.pptxcarbohydrate metabolism -bpt.pptx
carbohydrate metabolism -bpt.pptx
 
CARBOHYDRATE METABOLISM and Disorders.pdf
CARBOHYDRATE METABOLISM and Disorders.pdfCARBOHYDRATE METABOLISM and Disorders.pdf
CARBOHYDRATE METABOLISM and Disorders.pdf
 
Gluconeogenesis and Control of Blood Glucose.pptx
Gluconeogenesis and Control of Blood Glucose.pptxGluconeogenesis and Control of Blood Glucose.pptx
Gluconeogenesis and Control of Blood Glucose.pptx
 
Regulation of blood glucose
Regulation of blood glucoseRegulation of blood glucose
Regulation of blood glucose
 
Glucose Metabolism and Diabetes
Glucose Metabolism and DiabetesGlucose Metabolism and Diabetes
Glucose Metabolism and Diabetes
 
Glucose metabolism.pptx
Glucose metabolism.pptxGlucose metabolism.pptx
Glucose metabolism.pptx
 
Regulation of blood glucose concentration
Regulation of blood glucose concentrationRegulation of blood glucose concentration
Regulation of blood glucose concentration
 
Pancreas.ppt
Pancreas.pptPancreas.ppt
Pancreas.ppt
 
Class 4 gluconeogenesis
Class 4 gluconeogenesisClass 4 gluconeogenesis
Class 4 gluconeogenesis
 
Blood Glucose Homeostasis.pptx
Blood Glucose Homeostasis.pptxBlood Glucose Homeostasis.pptx
Blood Glucose Homeostasis.pptx
 
regulation of blood sugar
regulation of blood sugar regulation of blood sugar
regulation of blood sugar
 
Blood glucose level
Blood glucose levelBlood glucose level
Blood glucose level
 
CHO METABOL-1.ppt
CHO METABOL-1.pptCHO METABOL-1.ppt
CHO METABOL-1.ppt
 
Glucose-Determination-25-3-2013.ppt
Glucose-Determination-25-3-2013.pptGlucose-Determination-25-3-2013.ppt
Glucose-Determination-25-3-2013.ppt
 
Blood glucose homeostasis
Blood glucose homeostasisBlood glucose homeostasis
Blood glucose homeostasis
 
Unit 7 : Carbohydrates metabolism & disorders
Unit 7 : Carbohydrates metabolism & disordersUnit 7 : Carbohydrates metabolism & disorders
Unit 7 : Carbohydrates metabolism & disorders
 
CHO Metabolism.ppt
CHO Metabolism.pptCHO Metabolism.ppt
CHO Metabolism.ppt
 
Gluconeogenesis
GluconeogenesisGluconeogenesis
Gluconeogenesis
 
Blood sugar homeostasis
Blood sugar homeostasisBlood sugar homeostasis
Blood sugar homeostasis
 

Recently uploaded

The next social challenge to public health: the information environment.pptx
The next social challenge to public health:  the information environment.pptxThe next social challenge to public health:  the information environment.pptx
The next social challenge to public health: the information environment.pptxTina Purnat
 
call girls in aerocity DELHI 🔝 >༒9540349809 🔝 genuine Escort Service 🔝✔️✔️
call girls in aerocity DELHI 🔝 >༒9540349809 🔝 genuine Escort Service 🔝✔️✔️call girls in aerocity DELHI 🔝 >༒9540349809 🔝 genuine Escort Service 🔝✔️✔️
call girls in aerocity DELHI 🔝 >༒9540349809 🔝 genuine Escort Service 🔝✔️✔️saminamagar
 
Glomerular Filtration rate and its determinants.pptx
Glomerular Filtration rate and its determinants.pptxGlomerular Filtration rate and its determinants.pptx
Glomerular Filtration rate and its determinants.pptxDr.Nusrat Tariq
 
call girls in munirka DELHI 🔝 >༒9540349809 🔝 genuine Escort Service 🔝✔️✔️
call girls in munirka  DELHI 🔝 >༒9540349809 🔝 genuine Escort Service 🔝✔️✔️call girls in munirka  DELHI 🔝 >༒9540349809 🔝 genuine Escort Service 🔝✔️✔️
call girls in munirka DELHI 🔝 >༒9540349809 🔝 genuine Escort Service 🔝✔️✔️saminamagar
 
Apiculture Chapter 1. Introduction 2.ppt
Apiculture Chapter 1. Introduction 2.pptApiculture Chapter 1. Introduction 2.ppt
Apiculture Chapter 1. Introduction 2.pptkedirjemalharun
 
Presentation on General Anesthetics pdf.
Presentation on General Anesthetics pdf.Presentation on General Anesthetics pdf.
Presentation on General Anesthetics pdf.Prerana Jadhav
 
Presentació "Real-Life VR Integration for Mild Cognitive Impairment Rehabilit...
Presentació "Real-Life VR Integration for Mild Cognitive Impairment Rehabilit...Presentació "Real-Life VR Integration for Mild Cognitive Impairment Rehabilit...
Presentació "Real-Life VR Integration for Mild Cognitive Impairment Rehabilit...Badalona Serveis Assistencials
 
Introduction to Sports Injuries by- Dr. Anjali Rai
Introduction to Sports Injuries by- Dr. Anjali RaiIntroduction to Sports Injuries by- Dr. Anjali Rai
Introduction to Sports Injuries by- Dr. Anjali RaiGoogle
 
Hematology and Immunology - Leukocytes Functions
Hematology and Immunology - Leukocytes FunctionsHematology and Immunology - Leukocytes Functions
Hematology and Immunology - Leukocytes FunctionsMedicoseAcademics
 
Radiation Dosimetry Parameters and Isodose Curves.pptx
Radiation Dosimetry Parameters and Isodose Curves.pptxRadiation Dosimetry Parameters and Isodose Curves.pptx
Radiation Dosimetry Parameters and Isodose Curves.pptxDr. Dheeraj Kumar
 
Case Report Peripartum Cardiomyopathy.pptx
Case Report Peripartum Cardiomyopathy.pptxCase Report Peripartum Cardiomyopathy.pptx
Case Report Peripartum Cardiomyopathy.pptxNiranjan Chavan
 
Statistical modeling in pharmaceutical research and development.
Statistical modeling in pharmaceutical research and development.Statistical modeling in pharmaceutical research and development.
Statistical modeling in pharmaceutical research and development.ANJALI
 
SWD (Short wave diathermy)- Physiotherapy.ppt
SWD (Short wave diathermy)- Physiotherapy.pptSWD (Short wave diathermy)- Physiotherapy.ppt
SWD (Short wave diathermy)- Physiotherapy.pptMumux Mirani
 
PNEUMOTHORAX AND ITS MANAGEMENTS.pdf
PNEUMOTHORAX   AND  ITS  MANAGEMENTS.pdfPNEUMOTHORAX   AND  ITS  MANAGEMENTS.pdf
PNEUMOTHORAX AND ITS MANAGEMENTS.pdfDolisha Warbi
 
COVID-19 (NOVEL CORONA VIRUS DISEASE PANDEMIC ).pptx
COVID-19  (NOVEL CORONA  VIRUS DISEASE PANDEMIC ).pptxCOVID-19  (NOVEL CORONA  VIRUS DISEASE PANDEMIC ).pptx
COVID-19 (NOVEL CORONA VIRUS DISEASE PANDEMIC ).pptxBibekananda shah
 
POST NATAL EXERCISES AND ITS IMPACT.pptx
POST NATAL EXERCISES AND ITS IMPACT.pptxPOST NATAL EXERCISES AND ITS IMPACT.pptx
POST NATAL EXERCISES AND ITS IMPACT.pptxvirengeeta
 
VarSeq 2.6.0: Advancing Pharmacogenomics and Genomic Analysis
VarSeq 2.6.0: Advancing Pharmacogenomics and Genomic AnalysisVarSeq 2.6.0: Advancing Pharmacogenomics and Genomic Analysis
VarSeq 2.6.0: Advancing Pharmacogenomics and Genomic AnalysisGolden Helix
 
Glomerular Filtration and determinants of glomerular filtration .pptx
Glomerular Filtration and  determinants of glomerular filtration .pptxGlomerular Filtration and  determinants of glomerular filtration .pptx
Glomerular Filtration and determinants of glomerular filtration .pptxDr.Nusrat Tariq
 
Measurement of Radiation and Dosimetric Procedure.pptx
Measurement of Radiation and Dosimetric Procedure.pptxMeasurement of Radiation and Dosimetric Procedure.pptx
Measurement of Radiation and Dosimetric Procedure.pptxDr. Dheeraj Kumar
 
Lippincott Microcards_ Microbiology Flash Cards-LWW (2015).pdf
Lippincott Microcards_ Microbiology Flash Cards-LWW (2015).pdfLippincott Microcards_ Microbiology Flash Cards-LWW (2015).pdf
Lippincott Microcards_ Microbiology Flash Cards-LWW (2015).pdfSreeja Cherukuru
 

Recently uploaded (20)

The next social challenge to public health: the information environment.pptx
The next social challenge to public health:  the information environment.pptxThe next social challenge to public health:  the information environment.pptx
The next social challenge to public health: the information environment.pptx
 
call girls in aerocity DELHI 🔝 >༒9540349809 🔝 genuine Escort Service 🔝✔️✔️
call girls in aerocity DELHI 🔝 >༒9540349809 🔝 genuine Escort Service 🔝✔️✔️call girls in aerocity DELHI 🔝 >༒9540349809 🔝 genuine Escort Service 🔝✔️✔️
call girls in aerocity DELHI 🔝 >༒9540349809 🔝 genuine Escort Service 🔝✔️✔️
 
Glomerular Filtration rate and its determinants.pptx
Glomerular Filtration rate and its determinants.pptxGlomerular Filtration rate and its determinants.pptx
Glomerular Filtration rate and its determinants.pptx
 
call girls in munirka DELHI 🔝 >༒9540349809 🔝 genuine Escort Service 🔝✔️✔️
call girls in munirka  DELHI 🔝 >༒9540349809 🔝 genuine Escort Service 🔝✔️✔️call girls in munirka  DELHI 🔝 >༒9540349809 🔝 genuine Escort Service 🔝✔️✔️
call girls in munirka DELHI 🔝 >༒9540349809 🔝 genuine Escort Service 🔝✔️✔️
 
Apiculture Chapter 1. Introduction 2.ppt
Apiculture Chapter 1. Introduction 2.pptApiculture Chapter 1. Introduction 2.ppt
Apiculture Chapter 1. Introduction 2.ppt
 
Presentation on General Anesthetics pdf.
Presentation on General Anesthetics pdf.Presentation on General Anesthetics pdf.
Presentation on General Anesthetics pdf.
 
Presentació "Real-Life VR Integration for Mild Cognitive Impairment Rehabilit...
Presentació "Real-Life VR Integration for Mild Cognitive Impairment Rehabilit...Presentació "Real-Life VR Integration for Mild Cognitive Impairment Rehabilit...
Presentació "Real-Life VR Integration for Mild Cognitive Impairment Rehabilit...
 
Introduction to Sports Injuries by- Dr. Anjali Rai
Introduction to Sports Injuries by- Dr. Anjali RaiIntroduction to Sports Injuries by- Dr. Anjali Rai
Introduction to Sports Injuries by- Dr. Anjali Rai
 
Hematology and Immunology - Leukocytes Functions
Hematology and Immunology - Leukocytes FunctionsHematology and Immunology - Leukocytes Functions
Hematology and Immunology - Leukocytes Functions
 
Radiation Dosimetry Parameters and Isodose Curves.pptx
Radiation Dosimetry Parameters and Isodose Curves.pptxRadiation Dosimetry Parameters and Isodose Curves.pptx
Radiation Dosimetry Parameters and Isodose Curves.pptx
 
Case Report Peripartum Cardiomyopathy.pptx
Case Report Peripartum Cardiomyopathy.pptxCase Report Peripartum Cardiomyopathy.pptx
Case Report Peripartum Cardiomyopathy.pptx
 
Statistical modeling in pharmaceutical research and development.
Statistical modeling in pharmaceutical research and development.Statistical modeling in pharmaceutical research and development.
Statistical modeling in pharmaceutical research and development.
 
SWD (Short wave diathermy)- Physiotherapy.ppt
SWD (Short wave diathermy)- Physiotherapy.pptSWD (Short wave diathermy)- Physiotherapy.ppt
SWD (Short wave diathermy)- Physiotherapy.ppt
 
PNEUMOTHORAX AND ITS MANAGEMENTS.pdf
PNEUMOTHORAX   AND  ITS  MANAGEMENTS.pdfPNEUMOTHORAX   AND  ITS  MANAGEMENTS.pdf
PNEUMOTHORAX AND ITS MANAGEMENTS.pdf
 
COVID-19 (NOVEL CORONA VIRUS DISEASE PANDEMIC ).pptx
COVID-19  (NOVEL CORONA  VIRUS DISEASE PANDEMIC ).pptxCOVID-19  (NOVEL CORONA  VIRUS DISEASE PANDEMIC ).pptx
COVID-19 (NOVEL CORONA VIRUS DISEASE PANDEMIC ).pptx
 
POST NATAL EXERCISES AND ITS IMPACT.pptx
POST NATAL EXERCISES AND ITS IMPACT.pptxPOST NATAL EXERCISES AND ITS IMPACT.pptx
POST NATAL EXERCISES AND ITS IMPACT.pptx
 
VarSeq 2.6.0: Advancing Pharmacogenomics and Genomic Analysis
VarSeq 2.6.0: Advancing Pharmacogenomics and Genomic AnalysisVarSeq 2.6.0: Advancing Pharmacogenomics and Genomic Analysis
VarSeq 2.6.0: Advancing Pharmacogenomics and Genomic Analysis
 
Glomerular Filtration and determinants of glomerular filtration .pptx
Glomerular Filtration and  determinants of glomerular filtration .pptxGlomerular Filtration and  determinants of glomerular filtration .pptx
Glomerular Filtration and determinants of glomerular filtration .pptx
 
Measurement of Radiation and Dosimetric Procedure.pptx
Measurement of Radiation and Dosimetric Procedure.pptxMeasurement of Radiation and Dosimetric Procedure.pptx
Measurement of Radiation and Dosimetric Procedure.pptx
 
Lippincott Microcards_ Microbiology Flash Cards-LWW (2015).pdf
Lippincott Microcards_ Microbiology Flash Cards-LWW (2015).pdfLippincott Microcards_ Microbiology Flash Cards-LWW (2015).pdf
Lippincott Microcards_ Microbiology Flash Cards-LWW (2015).pdf
 

Carbohydrate Metabolism and Glucose Regulation

  • 1. A. CARBOHYDRATES 1. Describe and classify carbohydrates • Description • Contain C, H and O molecules • Contain a C=O (ketone) and an –OH(aldehyde) functional group • Classification • Based on certain properties • The size of the base carbon chain • Location of the CO functional group • Number of sugar units • Stereochemistry of compound • Chemical Properties • Some ( not all ) carbs are reducing substances (donate electrons) • Chemical reduction of other substances • These sugars must contain an aldehyde or ketone group • Reducing sugars o Glucose o Maltose o Lactose o Fructose o Galactose Sucrose is not a reducing substance • 2. Describe carbohydrate metabolism • Glucose is primary energy source • Nervous tissue cannot concentrate or store carbohydrates, so a steady supply of glucose is needed • Once the level of glucose falls below a certain range, normal function is impaired • Carbohydrate Breakdown Ultimate Goal o Convert glucose to CO2 and water with ATP as a by-product 3. Discuss glycolysis as it pertains to carbohydrate metabolism and carbohydrate detection methods • Hydrolysis of glucose by an enzyme into pyruvate or lactate; • This process is anaerobic 4. Fasting blood glucose levels • Hyperglycemic o Fasting blood glucose > 100 mg/dL • Hypoglycemic o Fasting blood glucose < 50 mg/dL Page 1
  • 2. 5. Describe glycolysis • Glycolysis – the conversion of glucose and other hexoses into lactate or pyruvate • Breakdown of glucose for energy production 6. Describe carbohydrate breakdown • • 7. Ultimate Goal o Convert glucose to CO2 and water with ATP as a by-product o Possible channels o Converted to liver glycogen and stored o Metabolized to CO2 and H2O o Converted to keto-acids, amino acids, and proteins o Converted to fats and stored in adipose tissue Biochemical pathways in carbohydrate breakdown o Embden-Meyerhoff pathway o Converts glucose to pyruvate/lactate o Primary energy source for humans o Hexose monophosphate shunt o Oxidizes glucose to ribose and CO2 o Produces NADPH as an energy source o Glycogenesis o Converts glucose to glycogen Describe the role of the liver in maintenance of glucose levels. • The liver maintains the glucose levels by: o Glycogenesis  Converts glucose to glycogen Page 2
  • 3. o o • Glycogenolysis –  Breakdown of glycogen to form glucose  Glycogenolysis occurs when plasma glucose is decreased  Occurs quickly if additional glucose is needed  Controlled by hormones & enzymes Gluconeogenesis  Formation of glucose from non-carbohydrate sources, such as amino acids, glycerol & fatty acids into glucose • Occurs mainly in the liver  During long fasts, gluconeogenesis is required to maintain blood glucose levels because glycogen stores are up in about 24 hours During a fast, the blood glucose level is kept constant by mobilizing the glycogen stores in the liver Page 3
  • 4. 8. What hormones does the liver use to maintain glucose levels? • Insulin o Produced by the beta cells of the islets of Langerhans in the pancreas o Promotes the entry of glucose into liver, muscle, and adipose tissue to be stored as glycogen and fat; o Inhibits the release of glucose from the liver o Insulin secretion controlled by:  Blood glucose level  Certain Amino Acids ie. leucine, & arginine • Glucagon o Secreted by the alpha cells of the pancreatic islets of Langerhans o Increases blood glucose by stimulating glycogenolysis and gluconeogenesis o 2nd most important glucose regulatory hormone o Referred to as a hyperglycemic agent o Synthesized in alpha cells of the islets of Langerhans o Action/Effect of o Stimuli – decreased plasma glucose o Action  Increases glycogenolysis & gluconeogenesis  Promotes breakdown of fatty acids  Promotes breakdown of proteins to form amino acids  Increases plasma glucose concentration • Somatostatin o Origin-Delta cells of the islets of Langerhans in the pancreas o Effect - increase plasma glucose o Actions  antagonistic to insulin, Page 4
  • 5. o • 9.  inhibits endocrine hormones including glucagon & growth hormone Inhibits secretion of insulin, glucagon, and growth hormone resulting in an increase in plasma glucose levels Other regulatory hormones o Epinephrine  One of two glucose regulating hormones from the adrenal gland  Origin – adrenal medulla  Action/effect • Inhibits insulin secretion & release • Promotes lipolysis • Stimulates glycogenolysis • Immediate release of glucose  Stimuli • Neurogenic - based on physical / emotional stress. • Adrenal tumors o Glucocorticoids - such as cortisol  Origin – adrenal cortex  Effect – antagonistic to insulin • increases blood glucose • promotes gluconeogenesis from breakdown of proteins • inhibits the entry of glucose into muscle cells  Stimuli – anterior pituitary’s ACTH o Growth Hormone (GH) and Adrenocorticotropic Hormone (ACTH)  Origin – anterior pituitary gland  Effect – antagonistic to insulin • Increases plasma glucose levels • inhibits insulin secretion • inhibits entry of glucose into muscle cells • inhibits glycolysis • inhibits formation of triglycerides from glucose  Stimuli • decreased glucose stimulates its release • increased glucose inhibits its release o Thyroid hormones (such as thyroxine)  Origin – thyroid gland  Effect • Increases absorption of glucose from intestines • Promotes conversion of liver glycogen to glucose o Stimuli – pituitary gland’s TSH What hormones does the pancreas produce that regulate carbohydrate metabolism? • Insulin o Produced by the beta cells of the islets of Langerhans in the pancreas o Promotes the entry of glucose into liver, muscle, and adipose tissue to be stored as glycogen and fat; Page 5
  • 6. • • 10. o Inhibits the release of glucose from the liver Glycagon o Secreted by the alpha cells of the pancreatic islets of Langerhans o Increases blood glucose by stimulating glycogenolysis and gluconeogenesis Somatostatin o Synthesized by the delta cells of the pancreatic islets of Langerhans o Inhibits secretion of insulin, glucagon, and growth hormone resulting in an increase in plasma glucose levels What impact does cortisol, catecholamine hormones and thyroid hormones have on glucose levels. • Cortisol o o o Secreted by the adrenal glands; Stimulates glycogenolysis, lipolysis, and gluconeogenesis Increases plasma glucose by decreasing intestinal entry into cells and increasing gluconeogenesis. • Epinephrine o Increases plasma glucose by inhibiting insulin secretion. o Secreted by the medulla of the adrenal glands. o It stimulates glycogenolysis and lipolysis; o It inhibits secretion of insulin. o Physical or emotional stress causes increased secretion of epinephrine and an immediate increase in blood glucose levels. • Thyroid hormone o Secreted by the thyroid gland; o Stimulates glycogenolysis and gluconeogenesis; o Increases glucose absorption from the intestines 11. Describe the metabolic defect in Diabetes Mellitus? • Glucose does not get into the cells 12. What are typical glucose levels, insulin levels, and ketone levels in this disease? 1) Type I Diabetes Mellituso Glucose levels are increased o Insulin levels are decreased o Ketones present 2) Type II Diabetes Mellitus o Glucose levels are increased o Insulin levels are normal to decreased o Glucagon response is decreased o No ketones present. 13. What is glycosylated hemoglobin? • Glycated hemoglobin is formed from the nonenzymatic, irreversible attachment of glucose to hemoglobin A1. Page 6
  • 7. • • • 14. Measurement of glycated hemoglobin reflects blood glucose levels for the past 2– 3 months. It is useful in monitoring effectiveness of treatment and compliance of diabetic individual to treatment protocol. The primary determinant in the rate of hemoglobin A-1c synthesis is the life span of the Red Blood Cell and the level of average glucose concentration What are the normal glucose levels in a fasting individual? • Normal glucose level (fasting)- 70 to 110 mg/dl 15. What are panic or critical glucose values? • Panic Values- >126 (fasting), > 200 (random or glucose tolerance test) 16. Describe the relationship between glucose levels in urine and serum. • • • • • Glucose will never be found in urine unless the serum glucose is high enough to exceed the renal threshold and spill over into the urine Glucose is filtered by the glomeruli, reabsorbed by the tubules, and normally not present in urine. If the blood glucose level is elevated, glucose appears in the urine, a condition known as glucosuria. An individual’s renal threshold for glucose varies between 160 and 180 mg/ dL. When blood glucose reaches this level or exceeds it, the renal tubular transport mechanism becomes saturated, which causes glucose to be excreted into the urine. 17. Name inherited disorder of carbohydrate metabolism • Glucose 6 phosphatase deficiency AKA von Glerke • Galactosemia o Characterized by a deficiency or absence of galactokinase, o Enzyme defect prevents metabolism of galactose. o Galactose is found in milk as a component of lactose, with galactosemia generally identified in infants. 18. Why should serum for glucose be removed from the red cells as soon after collection as possible? • 19. Serum should be removed from RBC’s ASAP because the cells will use the glucose and falsely decrease the glucose level. What anticoagulant preservatives are used for glucose specimens? Why? • Sodium fluoride is the anticoagulant of choice because it inhibits glycolytic enzymes. Page 7
  • 8. 20. What is a two-hour post prandial glucose? Why is it performed? • • 21. Why are D-xylose tolerance tests performed? • 22. This test is done to differentiate malabsorption from pancreatic insufficiency. What is the normal range of CSF glucose? • • • • 23. This is a sample taken two hours after eating and can determine how well the body is using the glucose. This sample will show insulin function which is the main reason it is performed. Normal CSF glucose- 40-70 mg/dl CSF glucose is 2/3 of the plasma glucose which is due to the glucose entering the CSF by facilitative transport. The carrier mechanism is responsible for transporting glucose across the downhill gradient. Meningitis can cause a change in the CSF glucose levels. Describe states that result in alteration of serum glucose. • • • • Diabetes mellitus PancreatitisRecent meal intake Fasting Increases glucose (insulin is not working) Increases glucose (insulin not produced) Increases glucose Decreases glucose 24. Lactate • The normal end product of glucose metabolism is pyruvate; • Lactate is produced under conditions of oxygen deficit (anaerobic metabolism). • The production and accumulation of lactate in the blood and its measurement aid in assessing the degree of oxygen deprivation that is occurring. • Change in the blood lactate level precedes a change in blood pH. • Lactate is metabolized by the liver via gluconeogenesis. 25. What are normal glycosylated hemoglobin ranges? • • Normal glycosylated hemoglobin range o 4.5-8.0 High values indicate that the patient has not been following the proper diet Page 8
  • 10. B. LIPIDS 1. Describe the cholesterol • Function o Used to manufacture and repair cell membranes, o Used in synthesis of bile acids and vitamin D • Synthesized in the liver and obtained from the diet. • Precursor for synthesis of bile acids, steroid hormones, and vitamin D • Transport mechanism o Transported through the blood by LDL (low density lipoproteins) • Storage sites o Stored in the skin, adipose tissue, and muscle cells. • Esterified cholesterol-2/3 of the total cholesterol is esterified 2. Describe triglycerides • Formed from one glycerol molecule with three fatty acid molecules attached via ester bonds • Comprises 95% of all fats stored in adipose tissue • Transport mechanism • Transported through the body by chylomicrons and VLDL 3. HDL-cholesterol • Synthesized by the liver and by the intestine • In normal lipid metabolism, HDL removes excess cholesterol from peripheral tissues and transports it to other catabolic sites providing an antiatherogenic effect.. 4. What is the role of lipase? • Lipase and bile acids are used to break down fats in lipid absorption. • Found in pancreas, with lesser amounts in gastric mucosa, intestinal mucosa, adipose tissue • Clinical significance: o Increased serum levels in acute pancreatitis occur in 4– 8 hours after the onset of pain, with peak values in 24 hours, and return to normal in 8– 14 days. 5. List the bile acids • Cholic acid, • Glycocholic acid • Taurocholic acid 6. Where are bile acids synthesized? • Liver 7. What does the presence of bile acids in serum indicate? • . The presence of bile acids in serum can indicate liver disease. Page 10
  • 11. 8. What are chylomicrons? • Triglycerides are transported through the body by chylomicrons • Chylomicrons- the largest lipoprotein particles with diameters ranging from 80-1200 nm. They are 90-95% triglycerides, 2-6% phospholipids, 2-4% cholesteryl ester, 1% free cholesterol, and 1-2% apolipoprotein. • 9. How does serum appear with increased chylomicrons? increased cholesterol? increased triglycerides? • Increased Chylomicrons- a creamy layer appears on top of the serum. • Increased Cholesterol- serum appears milky white • Increased Triglycerides- serum appears turbid. 10. Given a value of 38 mg/dl for the HDL, 140 for triglycerides and 210 for total cholesterol, calculate the LDL and VLDL. • • • • VLDL= triglycerides/5 LDL= total cholesterol- VLDL- HDL VLDL= 140/5 =28 LDL= 210-28-38= 144 C. LIVER FUNCTION AND HEME-DERIVATIVES 1. Describe liver function in reference to each of the following: 3) Carbohydrate synthesis and metabolism • The liver uses glycogenesis to make glycogen from glucose, simple sugars and amino acids. • The glycogen produced is stored and used as needed. • The liver also uses glucose for maintenance of mitochondrial NADH and to generate ATP in the Embden Myerhoff pathway. 4) Protein synthesis and metabolism • The liver makes various proteins (transferrin, prothrombin, and ceruloplasmin) which are used to transport materials like iron, and copper. • The liver also uses the proteins for nutrition, to regulate oncotic blood pressure (albumin), and for coagulation. o These proteins include albumin, HDL, LDL, VLDL, haptoglobin, angiotensin, erythropoiten, and many others. o These proteins also are used for nitrogen excretion by processing ammonia, urea, creatinine and uric acid. 5) Lipid synthesis and metabolism• This is where cholesterol, triglycerides and phospholipids are synthesized. • Free fatty acids are metabolized in the citric acid cycle into NADH, and bile acids are produced form cholesterol. 6) Porphyrin synthesis • The liver produces the enzyme aminolevulinic acid synthase which controls the synthesis of porphyrins which eventually form heme molecules. Page 11
  • 12. 7) Bile acid synthesis • the bile acids are conjugated with amino acids to form bile salt. • It is synthesized by cholesterol in the bile ducts and ends up in the intestine where the lipids are digested. 8) iron and vitamin storage • Iron is stored in the liver and transported wherever needed by transferrin, and vitamins are stored in the liver and available to be used whenever needed. 9) Excretion of metabolic end product and detoxification • It converts ammonia to urea. 10) Bile pigment formation • Bilirubin is the principal pigment in bile and is derived from the breakdown of hemoglobin when aged red blood cells are phagocytized by the reticuloendothelial system, primarily in the spleen, liver and bone marrow. 2. 3. Where is bilirubin produced • Bilirubin is produced in the reticuloendothelial system from the breakdown of hemoglobin from senescent red blood cells • Bilirubin forms a complex with albumin for transport to the liver. • In this form, bilirubin is unconjugated and not water soluble. Describe heme catabolism by the reticuloendothelial system (extravascular hemolysis). List states associated with increased extravascular hemolysis. • Hemoglobin is broken down extravascularly into globin and heme (iron and protoporphrin IX) • Protoporphrin breaks down further into unconjugated bilirubin which is carried to the liver by albumin and conjugated with glucuronyl transferase. o This is associated with RBC membrane defects and defects in the heme structure. 4. Identify the function of haptoglobin, hemopexin albumin and methemalbumin in heme catabolism in intravascular hemolysis. • Haptoglobin binds free hemoglobin and takes it back to the liver so it is not lost in the urine, • Hemopexin albumin removes circulating heme from the blood and delivers it to the liver, • Methemalbumin is free heme oxidized and bound to albumin which is carried to the liver and acts as storage that can be used until enough hemopexin is produced and made available to the liver. 5. List states associated with increased intravascular hemolysis. • Intravascular hemolysis is associated with increased immunologic processes, mechanical injury, and toxins. 6. Differentiate between unconjugated and conjugated bilirubin • Conjugated bilirubin Page 12
  • 13. Bilirubin is conjugated in the hepatocyte endoplasmic reticulum with glucuronic acid to form bilirubin diglucuronide ( conjugated bilirubin). o The reaction is catalyzed by UDP o Conjugated bilirubin is water soluble. o Conjugated bilirubin is excreted into the bile for storage in the gallbladder, secreted into the duodenum in response to gallbladder stimulation, and reduced by anaerobic bacteria in the intestine to urobilinogen. o Some intestinal urobilinogen is reabsorbed;  A portion returns to the liver and some enters the circulation for excretion in the urine, whereas the remaining portion in the intestines is oxidized by anaerobic bacteria for excretion in the stool as urobilin. o Urobilin is an orange- brown pigment that gives stool its characteristic color. Unconjugated bilirubin o Bilirubin is produced in the reticuloendothelial system from the breakdown of hemoglobin from senescent red blood cells o Bilirubin forms a complex with albumin for transport to the liver. o In this form, bilirubin is unconjugated and not water soluble. o • 7. Describe the process of bilirubin conjugation. What is the role of glucuronyl transferase and where is this enzyme synthesized? What relationship exists between enzyme synthesis and neonatal physiologic jaundice? • Hemoglobin is broken down into portoporphrin which is converted into unconjugated bilirubin which is bound to albumin and carried to the liver where it is converted to conjugated bilirubin by the enzyme glucuronyl transferase (synthesized in the liver). 8. What is Kernicterus? Why does it develop? • This is a serious newborn condition that occurs in the central nervous system because of high bilirubin levels. • It is caused by an under developed blood brain barrier, and because newborns do not produce enough glucuronyl transferase. 9. Describe excretion of bilirubin and resulting formation of urobilinogen? Why is urobilinogen normally present in urine and serum? • Conjugated bilirubin is taken to the intestines where bacteria convert it into urobilinogen. • Urobilinogen is normally present in the urine because a small amount of it is filtered back to the liver where it is recirculated and sent to the kidney where it is excreted in the urine. 10. Prehepatic jaundice • Prehepatic jaundice occurs when there is excessive erythrocyte destruction, as seen in hemolytic anemias, spherocytosis, toxic conditions, hemolytic disease of the newborn caused by Rh or ABO incompatibility, etc Page 13
  • 14. • • In these cases, the rate of hemolysis exceeds the liver’s ability to take up the bilirubin for conjugation. Prehepatic jaundice is characterized by an increased level of unconjugated bilirubin in the serum. 11. Hepatic jaundice • occurs when the liver cells malfunction and cannot take up, conjugate, or secrete bilirubin. o Gilbert syndrome:  Defect in the ability of hepatocytes to take up bilirubin; due to transport problem of bilirubin from the sinusoidal membrane to the microsomal region; characterized by mild increase in serum level of unconjugated bilirubin o Neonatal physiological jaundice:  Level of UDP- glycuronyltransferase is low at birth;  Takes several days for the liver to synthesize an adequate amount of the enzyme to catalyze bilirubin conjugation; causes increased serum level of unconjugated bilirubin 12. Posthepatic jaundice • Occurs when an obstruction blocks the flow of bile into the intestines. • This is referred to as extrahepatic cholestasis and may be caused by gallstones obstructing the common bile duct, neoplasms such as carcinoma of the ampulla of Vater or carcinoma of the pancreas, and inflammatory conditions such as acute cholangitis or acute pancreatitis. • Posthepatic jaundice is characterized by: o Significantly increased level of conjugated bilirubin in serum, o Increased level of unconjugated bilirubin in serum, o Increased conjugated bilirubin in the urine, o Decreased urine and fecal urobilinogen o Stool that appears pale in color. 13. Why are bilirubin determinations performed on amniotic fluid? D. B. Porphyrins and Heme Derivates 1. What are the porphyrias? • These are deficiencies in the enzyme production that can be acquired or inherited. They result in increased production of one of the heme precursors to which they are intermediates. 2. What is the physiologic function of porphyrins. • The most important function of porphyrias are to chelate iron from heme. Page 14
  • 15. 3. When are plasma hemoglobin levels increased and how are they measured? • The plasma hemoglobin levels are increased during thalessemias and hemoglobinopathies. • They can be measured by cellulose acetate electrophoresis or citrate agar electrophoresis. 4. What reagent is used for the detection of urobilinogen? When would urobilinogen levels be decreased? Increased? • Ehrlrich's reagent is most often used for urobilinogen detection • • Increased Urobilinogen levels o Excess hemolysis, o Liver damage by hypoxia o Exposure to various toxic agents. • Decreased Urobilinogen levels o Obstructive jaundice because there is a limited delivery of bilirubin to the gut. 5. When would myoglobin be increased? How are serum myoglobin levels measured? • Myoglobin is increased when there is trauma to skeletal or cardiac muscle. (myocardial infarction). • Serum myoglobin levels are measured by Electrophoresis. E. PROTEINS AND NON-PROTEIN NITROGEN 1. What are the functions of proteins in normal physiology? • Proteins function as transport carriers for other substances. • They transport substances to the proper sites for absorption, modification, or other utilization. 2. Salt fractionation• The proteins are fractioned out by the use of salts. The salts decrease the water available for hydration of the hydrophilic groups and cause precipitation of the globulins. 3. Zwitterion• An ion that has both positive and negative regions of charge. 4. Zeta potential • This is the potential difference between the negative charges on the surface of the red blood cell membrane and the cations in the aqueous medium. • Cations are divided into two groups, those that always move with the RBC and those that can move freely in the medium. • The zeta potential is measured from the boundary of these two cations to the negative charge on the membrane. Page 15
  • 16. 5. Polypeptide • Amino acids that combine to form proteins which link together to form peptides. • Many peptides linked together form a polypeptide. 6. Oligoclonal banding • Electrophoretic pattern of CSF form patients with multiple sclerosis with distinct bands in the globulin zone. 7. Briefly describe the Kjeldahl techniques for determination of protein and nonprotein nitrogen. • In this method the serum proteins are precipitated with an organic acid. • The nonprotein nitrogen is removed with the supernatent. • The protein pellet is digested in H2SO4 with heat and a catalyst (cupric sulfate). • Potassium sulfate can also be used to improve the digestion. • The H2SO4 oxidizes the C, H, and S in protein into CO2, CO, H2O, and SO2. • The nitrogen in the protein is then converted to ammonium bisulfite which is measured by adding alkali and distilling the ammonia into a standard boric acid solution. • The ammonium borate formed is then titrated with a standard solution of HCL to determine the amount of nitrogen in the original protein solution. 8. What are the major causes of increased and decreased albumin? Increased and decreased globulins? 9. What is the theory of refractometry? • Refractometry- the velocity of light is changed as it passes the boundary between 2 transparent layers (air and water) causing light to be bent. • When solute is added to water the refractive index at 20*C of 1.33 is increased by an amount proportional to the concentration of the solute in the solution. 10. What are major interfering substances in the determination of serum protein by refractometry? • Interfering substances- Nonprotien solids (electrolytes, urea, and glucose) 11. Name three ways to separate albumins from globulins. • Electrophoresis, • Chromatography • Precipitation Page 16
  • 17. 12. Discuss the reasons for determining spinal fluid protein and glucose. 13. What are normal values for spinal fluid protein and glucose? 14. What results are expected from spinal fluid in meningitis ? • In meningitis, encephalitis, and neurosyphilis there would be a decreased glucose level with increased protein levels (IgG). 15. What results are expected from spinal fluid in encephalitis? • In meningitis, encephalitis, and neurosyphilis there would be a decreased glucose level with increased protein levels (IgG). 16. What results are expected from spinal fluid in neurosyphilis? • In meningitis, encephalitis, and neurosyphilis there would be a decreased glucose level with increased protein levels (IgG). 17. Discuss the BCG method for determining albumin? Why is the pH important? • BCG (Bromocresol Green) method for determining albumin is a dye binding procedure where positively charged albumin is attracted to and binds to the anionic dye. • Once bound to the albumin, the dye has a different absorption maximum than free dye. • The amount of albumin can be quantitated by measuring the absorbance of the albumin-dye complex to which it is directly proportional. • The pH must be adjusted on the solution to make the albumin positively charged so it will bind to the dye. 18. What is Biuret reagent? Explain its function in determination of total protein. What are the major interfering substances? • The biuret reagent contains sodium potassium tartrate to complex cupric ions to prevent their precipitation in the alkaline solution, and potassium iodide which acts as an antioxidant. • In this procedure small peptides react and the color of the chelate produced has a different shade that seen with larger peptides (color varies from pink to reddish violet and is measured at 510nm). • Major interfering substances are any compound with 2 or more of the following groups NHCH2 , and NHCS. 19. What is a protein-free filtrate? List 3 precipitating reagents used. • A protein free filtrate removes proteins from whole blood, serum, urine, or other body fluids by precipitation with a precipitant and then filtration or centrifugation. • Some precipitating reagents are: o Tungstic acid o O-toluidine, o Horseradish peroxidase, o Molybdate Page 17
  • 18. o Trichloroacetic acid 20. What is Nessler's reagent? In what other reactions may it be used? • Nessler's reagent: o Double iodide of mercury and potassium. o This reagent is used in the determination of non-protein nitrogens (NPN). o It is also used in Nessleration reactions of urea nitrogen. 21. What is an A/G ratio? How is it used diagnostically? • A/G ratio determines how much albumin is in the body compared to globulins. • You use it in correlation with Total protein to determine if albumin or globulins are low. F. Specific serum proteins 1. What is alpha-1 antitrypsin? Ceruloplasmin? Name and describe disease processes involved with each protein. • Alpha-1 antitrypsin o Acute phase reactant o Main function  Neutralizes trypsin-like enzymes that can cause hydrolytic damage to structural protein. o Disease process:  Severe deficiency of this is associated with severe degenerative emphysematous pulmonary disease. • Ceruloplasmin o Copper containing glycoprotein that stores 90% of the total serum copper. o Disease process:  Low serum concentrations of copper are associated with Wilson’s disease. 2. • 3. • 4. What is the function of haptoglobin? Haptoglobin functions to bind free Hemoglobin by the alpha chain. Name three methods for measuring haptoglobin. 3 methods to measure Haptoglobin: o Starch gel electrophoresis, o Radial immunodiffusion, o Immunonephelometric methods. When is haptoglobin decreased? Increased? • Increased Haptoglobin levels: o Inflammatory conditions, Page 18
  • 19. • 5. • o Burns, and o Nephrotic syndrome Decreased Haptoglobin levels: o Intravascular hemolysis, o Transfusion reactions, o HDN, o Mechanical breakdown of RBC's, o Athletic trauma. Define Troponin Troponin- is a complex of 3 proteins that bind to the thin filaments of striated muscle (cardiac and skeletal) but are not present in smooth muscle. 6. List three isoforms that make up the troponin complex. o 3 isoforms:  TnT,  TnI,  TnC 7. Describe the advantages of troponin over CKMB and name the method currently available to measure cTnI. o Cardiac troponin I is highly specific for myocardial tissue, and because it does not normally circulate in blood it is 13x more abundant in the myocardium than CKMB on a weight basis. o cTnI is very sensitive and can indicate even a minor amount of cardiac necrosis. o The relative increase of cTnI is greater than that of CKMB. o cTnI can be measured by: o Immunoenzymetric assays using 2 monoclonal Ab's directed against different epitopes on the protein. G. Protein Electrophoresis 1. What is a monoclonal gammopathy? polyclonal gammopathy? A monoclonal gammopathy is a sharp narrow band in the late beta or gamma region that suggests a monoclonal M spike. • The M spike is a spike of one class of Ig’s that is possibly metastasizing or producing clones of itself and suggests cancer, possibly multiple myeloma. • A polyclonal gammopathy is a broad gamma band that is increased. It looks abnormal and is usually caused by an infection. It is made up of more than one serum protein being increased. • 2. Where are the sites of synthesis for the following proteins: a. albumin- liver b. alpha-1 globulin- liver c. alpha-2 globulin- liver Page 19
  • 20. d. beta globulin- liver e. gamma globulins- (IgA, IgG, and IgM)- made by B cells that become plasma cells. 3. Discuss the clinical picture of multiple myeloma. What are the expected results of electrophoretic patterns on serum and urine of a myeloma patient? • In multiple myeloma you would see: o Bence jones proteins found in the urine (free kappa and lambda light chains) o "M" spike in electrophoretic pattern 4. Briefly describe serum protein electrophoresis. • Serum electrophoresis o Serum samples are applied to the cathode end of a support medium strip that is saturated with an alkaline buffer (8.6). The strip is connected to 2 electrodes and a current is passed. All serum proteins are negatively charged at the 8.6 pH so they migrate toward the anode end. 5. Name the five bands that occur in serum and list the major proteins that migrate in the five bands. 6. Diagram a normal pattern as they migrate from anode to cathode in a barbital buffer and label each peak. 7. What type of electrophoretic patterns can be expected in the following disease states? Explain the patterns and sketch them. a. Multiple myeloma b. Nephrotic syndrome c. Liver disease d. Chronic infection e. Acute phase reaction f. Malnutrition Page 20
  • 21. 8. Describe CSF protein electrophoresis. • CSF protein electrophoresis is done with the same technique as serum electrophoresis, except agarose gel is used most often because it is a high resolution technique. 9. What bands are normally seen in CSF protein electrophoresis • Oligoclonal bands are: distinct bands seen in the globulin zone and they are associated with multiple sclerosis (90%), and Inflammatory infectious neurological disease. 10. What are oligoclonal bands and what disease process are they associated with? • pre-albumin, • albumin, • alpha-1 globulin (antitrypsin), • insignificant alpha-2 globulin, • Beta-1 zone (transferrin) • Beta-2 zone. 11. Describe protein electrophoresis patterns would appear in the following situations and explain why the pattern appears as it does. a. Electrophoresis of plasma instead of serum b. Serum containing alpha-feto protein c. Fresh serum containing complement d. Bisalbuminemia e. Serum containing C-reactive protein Page 21
  • 22. H. Creatinine, Bun, Uric Acid 1. What is NPN? Which compounds comprise 50% of the total NPN's? • Non-protein nitrogen (NPN)- nitrogen containing compounds that remain in the blood sample after the removal of protein constituents. • Urea is the compound that comprises 50% of the total NPN’s. 2. What is BUN? What is Azotemia? How is urea nitrogen converted to urea mathematically? • Blood urea nitrogen (BUN)- nitrogen in the blood in the form of urea. This is a measurement that is used to analyze the urea level. • Azotemia- an elevated level of urea in the blood. • Bun x 2.14 = urea 3. What is creatine? How is it measured? • Creatine- a compound found in muscle synthesized from several amino acids. It combines with high energy phosphate to form creatine phosphate which functions as an energy compound. • We can measure creatine by the Jaffe method which measures creatine based on analyzing the sample for creatinine before and after heating in acid solution. The heating converts the creatine to creatinine and the difference between the two samples is the creatine concentration. 4. Relate elevations in uric acid to the following disease states: • • • • • Primary gout- this is caused by increases of uric acid which cause sodium urates to precipitate in the joints. This can be caused by overproduction of uric acid, drugs, and alcoholism. Secondary gout- this gout is formed as a secondary infection caused by a larger problem like leukemia. Leukemia- this causes the increased breakdown of cell nuclei caused by chemotherapy which causes the uric acid levels to increase. Polycythemia- this causes the increased breakdown of cell nuclei caused by chemotherapy, much like leukemia does, which also increases the uric acid levels to increase. Glomerular nephritis- in this disease the nephrons of the glomerulus are damaged in the kidney which causes poor filtration and increased levels of the uric acid. Page 22
  • 23. • Multiple myeloma- this disease is also treated with chemotherapy which breaks down the nuceli and causes the uric acid to increase much like leukemia and polycythemia. 5. Relate uric acid production to purine catabolism. • Uric acid is formed from the catabolism of purines like adenosine and guanine in the liver. This uric acid is transported by the plasma from the liver to the kidney where it is filtered by the glomerulus into the proximal tubules where most is reabsorbed and only small amounts are secreted into the urine. 6. What is the clinical significance of BUN? (renal, prerenal, postrenal). What creatinine values are expected in these conditions? • BUN- (direct urea measurement from serum or plasma) • It is used extensively in the determination of renal function. BUN RATIO Pre-renalRenalPost renal7. Increased Normal Increased Increased Normal Increased BUN/CR Increased Normal Decreased What are normal values for BUN and creatinine? What is the normal ratio of BUN to creatinine? When is the ratio altered? • Normal BUN- 7-18mg/dl • Normal Creatinine- 0.5-1.2mg/dl • Normal BUN/Creatinine ratio- 10:1-20:1 • The ratio is altered in: o Low protein uptake o Acute tubular necrosis o Severe liver disease • 8. CREATINE Diagram and describe the Berthelot reaction for BUN. Berthelot Reaction for BUNo Urea is hydrolyzed with urease, and the ammonia ion formed is reacted with phenol and hypochlorite in alkaline medium to form indophenol. Nitroprusside is used to catalyze the reaction. o Absorbance of dissociated indophenol (blue chromogen) is measured at 560nm. o REACTION:  NH4 + 5NaOCC+ 2 phenol◊ indophenol + 5NaCl + 5H2O What is the purpose of the following reagents? o urease- used ot prepare the stock suspension Page 23
  • 24. o o o sodium nitroprusside- catalyzes the reaction phenol- converted to indophenol alkaline hypochloride- aids in conversion of phenol to indophenol. 9. What are some advantages and disadvantages of this method? What anticoagulant must be avoided when using any urease method? o Advantages- can use serum, plasma, or urine. o Disadvantages- contamination of urine with bacteria is common and can cause decreased urea and formation of ammonia. 10. What kidney functions do the following clearance tests measure? inulin- Reference substance for measuring GFR (glomerular filtration rate) creatinine- universely used in assessment of GFR. urea- not useful in monitoring GFR, but serum urea may provide useful clinical metabolic information. p-amino-hippurate- reference substance for the measurement of renal plasma flow. • • • • • 11. 12. Diagram and describe the Jaffe reaction for creatinine. What substances give false positive reaction? • Jaffe reactiono The reaction occurs between creatinine and the picrate ion formed in the alkaline medium and a red-orange adduct develops. Teh observed rate of the hydroxyl ion concentrations over a broad range of picric acid concentrations. This is measured spectrophotometricaly at wavelengths of 485-520nm. • Substances that give false positives are: • Protein, • Glucose, • Vitamin C (ascorbic acid), • Acetone, • cephalosporin Why is creatinine preferred to urea for clearance tests? What data are necessary to calculate creatinine clearance? Write the formula. What are the normal values for creatinine clearance? • • • • • Creatinine is more specific for kidney function than urea is. Data necessary for calculating creatinine clearance: o Urine volume, o Creatinine concentration. in urine, o Creatinine concentration in plasma Creatine Clearance = (Urine Cr x Urine volume)/ (Plasma Cr) Normals: Male= 97-137 ml/min Females = 88-128 ml/min Page 24
  • 25. 13. • A creatinine clearance was performed on a male patient 1.5m tall and weighing 65kg. His blood contained 2.5 mg/dl creatinine. The urine creatinine was 50 mg/dl and the urine volume was 300 ml/4hrs. What was the creatinine clearance for this man? Creatine Clearance = (50 x 300)/ 2.5 x 1.76/ 1.60 (body surface area *see chart) • 14. 15. I. What is creatinine? What is the normal range? What single disease state is associated with elevated creatinine. • Creatinine is a compound formed when creatine or creatine phosphate spontaneously loses water or phosphoric acid. • It is excreted into the plasma at a relatively constant rate in a given individual and excreted in the urine. Its decrease is associated with renal dysfunction as in glomerulonephritis. Diagram the reaction, list reagents used and describe the principle of the oxidation reduction method for uric acid using phosphotungstic acid. • This is the most common method used. • It is based on the oxidation of uric acid in a protein-free filtrate with subsequent reduction of phosphotungstic acid to tungsten blue. • It uses Na carbonate to provide the alkaline pH necessary for the color development. • The blue color produced can be intensified by adding cyanide or by keeping the proper pH. Miscellaneous Proteins 1. • • Describe the method for detecting phenolketonuria? What enzyme deficiency results in phenolketonuria? Phenolketonuria results from a total absence of or absence of activity of the enzyme phenylalanine hydrolase (AKA phenylalanine-4-mono-oxygenase). You can use the Guthrie test to detect PKU. 2. What are cryoglobulins? How are they measured? • Cryoglobulins are serum protein that precipitates at temperatures lower than body temperature. 3. What is alpha-fetoprotein? What does its presence signify? How is it detected? • Alpha-fetoprotein is a globulin protein synthesized in the fetal yolk sac and then by the parenchymal cells of the liver. Page 25
  • 26. • • It is measured to determine if there is increased passage of fetal proteins into the amniotic fluid. We also measure the alpha-fetoprotein levels in association with spinabifiida, renal tube defects, and general fetal distress. 4. What is carcinoembryonic antigen? What types of tumors is it most frequently associated with? What types of methods are used to measure it? • Carcinoembryonic anitgens (CEA) are glycoproteins which are associated with numerous cancers (colon, lung, pancreas, stomach, or breast tissue tumors) 5. For each of the following tumor markers describe the types of tumors they are most often associated with and how they are measured. • CA125- ovarian cancer. • PSA- (prostate specific antigen)- Prostate cancer J. ENZYMES 1. (L) Give the substrates for the following: LD, CK, AST, ALT, GGT, CK-MB • LD Lactate o Catalyzes oxidation of Lactate to Pyruvate and the reverse reaction of Pyruvate to Lactate o ischemia, o myocarditis, o cardiac congestion; • CK Creatinine o Catalyzes the reversible phosphorylation of ATP o Muscular dystrophy, o muscle malignancies, o heart disease, o thyroid disease, o CNS disease. • AST Aspartate o Transfers amino acids o This is higher in neonates due to their immature liver o Liver disease (20-100 times normal in hepatitis), o carcinoma, o cirrhosis, o liver disease o heart disease, o muscle disease, o gallbladder disease, o AMI, o pulmonary embolism Page 26
  • 27. • • • 2. ALT o o o o o o o GGT o Alanine Catalyzes the transfer of an amino group of alanine to alpha-ketoglutarate Enzymatic-UV Monitoring Liver disease, carcinoma gallbladder disease, cirrhosis, hepatotoxicity Glutathione Transfers gamma-glutamyl residue from gamma-glutamyl peptides to amino acids, water, and other small peptides o Liver disease, o obstructions of the internal liver or gallbladder, o alcoholism, o pancreatic problems CK-MB Creatinine o Catalyzes the reversible phosphorylation of ATP o Myocardial problems (L) Define: a. Isoenzymeo one of several forms in which an enzyme can exist in various tissues. o Although they are similar they can be separated from each other by special chemical tests (electrophoresis) to give more specific information. b. Coenzymeo These are enzyme activators that are usually heat stable and of low molecular weight. o When these are combined with an inactive protein called an apoenzyme they form an active compound or a complete enzyme called holoenzyme. c. Catalysto Substance that speeds up the rate of a chemical reaction without itself being permanently altered of used up in the reaction. o They are effective in small quantities and are not used up in the reaction. They can be recovered unchanged. d. Activator o Substance in the body that converts an inactive substance into an active agent. o Example: the hydrogen ions on pepsinogen converting it to pepsin. e. Inhibitoro Chemical substance that stops the enzyme activity. f. Hydrolaseo Enzyme that causes hydrolysis. These catalyze bond cleavage by the addition of water. g. OxidoreductasePage 27
  • 28. h. Enzyme that removes electorns and their corresponding electrons. i. Transferaseo These enzymes move chemical grouping from one compound to another. 3. Relate amylase and lipase activity to the following disease states: a. Acute pancreatitis- both increase in this. b. Malabsorption- increased in both. c. Chronic pancreatitis- increased in both d. Pancreatic carcinoma- increased in both e. Cystic fibrosis- increased in both because it leads to malabsorption 4. What are the sources of acid phosphate in the body? What are normal ranges for acid phosphatase in males and females? o Acid phosphate is found in most tissues in the body like bone, bone marrow, liver, spleen, RBC’s, platelets, and in the highest concentration in the prostate gland of the male. 5. 8. (L) Explain the clinical significance of alkaline phosphatase in the following disease states: a. b. c. d. e. Obstructive jaundice- increased ALP levels Parenchymal jaundice- increased ALP levels Paget's disease- increased ALP levels Hyperparathyroidism- increased ALP levels Pregnancy- increased ALP levels 6. List enzymes elevated in hemolysis? What enzyme might be depressed with refrigeration and freezing? • Enzymes elevated in hemolysis: o CK, LD, AST (aspartate transferase), ACP (acid phosphatase), ALP (alkaline phosphatase), and LIPASE • Enzymes depressed by refrigeration: o LD (occurs at 4*C within 24 hours), and LIPASE (occurs if stored at 4*C for 3 weeks) 7. • What is cholinesterase? Why is it important in presurgery cases? Cholinesterase: an enzyme found in RBC’s, lungs, spleen, nerve endings, and brain. It is responsible for the prompt hydrolysis of acetylcholine released at the nerve endings to mediate transmission of the neural impulse across the synapse. The degradation of acetylcholine is necessary to the depolarization of the nerve so that it can be repolarized in the next conduction. o This is important to measure cholinesterase in presurgery to determine the amount of succinyl dicholine (muscle relaxer) that can be given in surgery without complications. (You can only give the amount of succinyl dicholine to the patient that the patients’ cholinesterase can rid their body of.) Page 28
  • 29. 8. What is the relationship of amylase and lipase in pancreatic disease? Why are both tests necessary in the monitoring of the disease? o Amylase and Lipase are both elevated in pancreatic disease. o You must monitor both amylase and lipase in pancreatic disease because:  Amylase is more sensitive, but less specific (b/c also found in other parts of the body)  Lipase is more specific, but found in small quantities (less sensitive b/c only found in the pancreas) 9. Discuss the following in relation to amylase activity: activators, pH, temperature. o Activatiors: Calcium and Chloride ions o pH- optimal is 6.9 - 7.0 o Temperature- optimal is 37-40*C 10. Discuss prostatic disease and acid phosphatase levels in serum. • Total activities of ACP may reach 40-50 times the normal in severe stages of prostate cancer. If the carcinoma is highly localized to the prostate there may only be slight increases in ACP activity. In benign hypertrophy of prostate, enzyme levels are normal. 11. How is the L(+) tartrate utilized in the determination of acid phosphatase? • Tartrate inhibits the activity of non-prostatic ACP so that specificity is enhanced when it is used. 12. Explain heat separation of alkaline phosphate isoenzymes. How does heat effect the liver fraction? bone? placenta? • Heat separation of alkaline phosphatase Isoenzymes  ALP activity is determined by measuring ALP before and after heating serum at 56*C for 10 minutes.  Placental ALP is the most heat stable followed by intestinal, liver, then bone.  Placental ALP will resist heat denaturation at 65*C for 30 min.  If the residual activity after heating is <20% of the total prior to heating then it is bone phosphatase.  If the residual activity after heating is >20% of the total prior to heating then it is Liver phosphatase. K. Enzyme Electrophoresis 1. List the CK isoenzymes. Describe the makeup of each fraction and organs associated with each fraction. How are isoenzymes separated? • CK-1 ( brain, brain subunits)- brain, prostate, uterus, bladdar, placenta • CK-2 (muscle, brain subunits)- heart muscle and skeletal muscle • CK-3 (muscle, muscle subunits)- sketal muscle and heart muscle Page 29
  • 30. 2. How can hemolysis affect the LDH electrophoretic pattern? What is the clinical significance of this? • Hemolysis can cause LDH electrophoresis to have an LD-1 to LD-2 flip. • Using a hemolyzed sample would cause the results to have a LD-1 to LD-2 flip as seen in cases of Myocardial Infarctions and Hemolytic anemia. L. ELECTROLYTES AND TRACE ELEMENTS 1. For each of the following give normal range, panic values, categorized as anion or cation if applicable and categorize as intracellular or extracellular if applicable. • Potassium – Cation • Sodium – Cation • Calcium – Cation • Magnesium – Cation - • Bicarbonate (HCO 3) – Anion • Chloride (Cl) - Anion 2. Describe the relationship between electrolyte balance and water balance. Include the roles of the kidney, hypothalamus, ADH and the renin-aldosterone system. • The electrolyte balance and water balance are directly related. • The plasma sodium concentration depends greatly on the intake and excretion of water. • If the sodium increases it stimulates thirst which will increase the intake of water, and the kidneys have the ability to conserve or excrete large amounts of sodium depending on the blood volume which is directly related to the water volume. • The excretion of water is largely affected by the ADH (which is secreted from the hypothalamus) release in response to the increase in blood volume. • The renin-aldosterone system acts in the kidney to increase the retention of sodium and increase the excretion of potassium which will eventually increase the blood pressure by using the electrolyte to balance the water level. 3. List anticoagulants of choice and the effect of hemolysis if any on the following ions: K, Na, Ca, Mg, HCO3, Cl, P04, Fe. • All electrolytes should be determined using serum or heparinized plasma. o K- hemolysis increases K levels o Mg- hemolysis increases Mg levels o HCO3- hemolysis increases HCO3 levels o P04- hemolysis increases PO4 levels o Fe- hemolysis increases Fe levels • o Na-hemolysis does not effect this significantly Page 30
  • 31. o o Ca- hemolysis does not effect this significantly Cl- hemolysis does not effect this significantly 4. Identify the major functions of sodium, chloride, bicarbonate, and potassium. • Sodium- this electrolyte largely determines the osmolality of the plasma. • Chloride- this electrolyte maintains the electrical balance by balancing the sodium charge, and using the chloride shift with bicarbonate. • Bicarbonate-this electrolyte is used to maintain the acid base balance and buffer the blood. • Potassium- this electrolyte regulates the neuromuscular excitability, contraction of the heart, ICF volume, and hydrogen ion concentration (pH). 5. Define and list conditions associated with each of the following: • Hyponatremia- this is decreased levels of sodium in the blood and is associated with the blood volume status. It results from sodium loss in excess of water loss. • Hypernatremia- elevated levels of sodium in the blood and is associated with increased sodium concentration because of excess water loss. It can be caused by increased sodium intake or decreased water intake. • Hypokalemia- decreased levels of potassium in the blood and is associated with GI or urinary loss of potassium , or with increased cellular use of potassium. This can be caused by vomiting, diarrhea, etc. • Hyperkalemia- increased levels of potassium in the blood and is associated with diabetes mellitus, or metabolic acidosis. • • • 6. What disease process results with increased bicarbonate? decreased bicarbonate? • Metabolic acidosis is related to the decrease in bicarbonate. • Metabolic alkalosis is related to the increase in bicarbonate. 7. Define titration. Explain its application to chloride methods. • Titration is the diluting out of a sample with a liquid reagent of a known strength and measuring the volume necessary to convert the sample through a given reaction. • In the chloride titration method the chloride ions combine with the mercuric ions to form soluble and undissociated mercuric chloride. The proteins in the serum are precipitated with the tungstic acid and an aliquot of the filtrate is titrated with an acidic solution of mercuric nitrate using a color indicator. This color indicator turns violet-blue at the first excess of mercuric ion. Page 31
  • 32. 8. List disease processes in which hyperchloremia and hypochloremia occur but sodium is normal. • Hyperchloremia- this is an increase of serum chloride and occurs in situations where there is an excess loss of bicarbonate ion due to GI losses, RTA, or metabolic acidosis. • Hypochloremia-this is an decrease of serum chloride and occurs with the excess loss of chloride from prolonged vomiting, diabetic ketoacidosis, or aldosterone deficiency. 9. List causes of an increased anion gap? decreased anion gap? • Decreased anion gap is rare, but may be seen in multiple meyloma because of abnormal proteins. It can also be caused by instrument error. • Increased anion gap- this may be caused by uremia, ketoacidosis (seen in starvation or diabetes), posioning due to ingestion of substances like methanol or ethylene glycol, lactic acidosis, or severe dehydration which causes increased plasma proteins or instrument error. 10. Explain why stock standards of sodium and potassium are kept in plastic containers. • Stock standards of Na and K are kept in plastic containers because glass containers leach out the Na and K from the sample. 11. What is an "anion gap". List formulas for its determination. • Anion gap- the difference between unmeasured anions and unmeasured cations. It is useful for indicating an increase in one or more of the unmeasured anions in serum. • AG = Na - (Cl + HCO3) • AG = (Na + K) - (Cl + HCO3) 12. Why is an anion gap routinely performed on all sets of electrolytes? What is an unacceptable gap? What is standard operating procedure when an anion gap is unacceptable? o An anion gap is routinely performed on all sets of electrolytes because it is useful in indicating an increase in one or more of the unmeasured anions in serum, and for QC on th analyzer (an abnormal gap can indicate an analyzer problem if performed on a person in good health). o An acceptable gap is 10-20:1, so greater than or less than that would be considered unacceptable and would need to be rerun. 13. Describe the impact of each of the following on serum potassium levels. • administration of insulin- decrease serum K levels (increases the cellular uptake of K) Page 32
  • 33. • • acidosis- increase serum K levels (excess H enters cell to be buffered and causes K to leave the cell to maintain electro neutrality) alkalosis- decreases the serum K (increases the cellular uptake of K) 14. Discuss the following factors influencing serum calcium and phosphorus levels: • parathyroid hormone- this hormone is used to increase the absorption of calcium and increase the excretion of phosphorus. To increase calcium it breaks down the bone to release Ca (bone resprption), it conserves Ca by increasing the tubular reabsorption in the kidney and it stimulates the renal production of vitamin D which also increases the Ca absorption. To decrease phosphorus the blood concentration the PTH increases the renal excretion. • calcitonin- this hormone is used to decrease calcium levels and increase the phosphorus levels which inhibits the actions of PTH and vitamin D. • Vitamin D (calcitriol)-when the calcium is decreased or the phosphorus is increased this hormone is used to increase calcium by aiding the effects of PTH by causing more calcium to be stored or released. It decreases phosphorus by increasing the absorption of it in the intestines and increasing the reabsroption in the kidneys. • plasma proteins-Albumin is the plasma protein that maintains the appropriate fluid in the tissues, and it binds various substances in the blood like calcium. • serum pH- a decrease in pH will increase the phosphate levels in the serum like seen with antiacids. 15. Discuss calcium, phosphorus and PTH levels related to the following disease states: • bone disease-calcium will be normal to low, phosphorus will be normal to low, and PTH will be normal to high. • malabsorption- calcium will be decreased, Phosphorus will be decreased, and PTH will be increased. • renal failure- calcium will be low to normal, phosphorus will by high, and PTH will also be high. • liver disease- calcium will be decreased, Phosphorus will be decreased, and PTH will be increased. • primary hyperparathyroidism-Calcium will be Increased, Phosphorus will be decreased, and PTH will be high. • secondary hyperparathyroidism-Calcium will be decreased, Phosphorus will be low to normal to high, and PTH will be increased. • primary hypoparathyroidism-Calcium will be decreased, Phosphorus will be increased, and PTH will be decreased. • secondary hypoparathyroidism-Calcium will be decreased, Phosphorus will be increased or decreased, and PTH will be decreased. Page 33
  • 34. 16. What is the physiologic relationship between calcium and phosphorus? Why? What are the physiologic functions of calcium and phosphorus? • Calcium and phosphorus are inversely related in the serum because phosphate is an intracellular anion, and calcium is an extracellular cation.. • Calcium functions in bone matrix, as an enzyme activator, in coagulation and complement, and in muscle contraction. • Phosphorus functions in production of ATP, GTP, CTP, UTP, and DNA structures, as a major body buffer, and in the bone matrix. 17. What is the relationship of calcium levels to alkaline phosphatase activity? • Alkaline phosphatase levels are increased in periods of bone growth or reconstruction which uses up the calcium and causes the serum levels of calcium to decrease. They both also function in bone matrix. 18. Describe three forms of body calcium. To which form is PTH most sensitive? How can only "active" calcium be measured? • Calcium forms- free-ionized Ca, protein bound calcium, and as complexed salts. PTH is most sensitive to ionized calcium. To measure only the active calcium you must measure it under anaerobic conditions because an increase in pH can cause the protein bound Ca to increase which decreases the ionized Ca, and decreasing the pH can cause the protein bound Ca to decrease and the ionized Ca to increase. 19. Define and describe tetany? What are the relationship of magnesium and calcium to tetany. • Tetany- irregular muscle spasms. • Calcium- a rapid decrease in ionized calcium concentration will cause tetany. • Magnesium- this is required along with ATPase for normal Ca uptake following a contration. It is also required for muscle cell stimulation by regulating the acetylcholine which is a potent neurotransmitter. 20. Describe the relationship between parathyroid hormone and magnesium levels. • The parathyroid hormone increases the renal absorption of magnesium and enhanses the absorption of magnesium in the intestine. PTH regulates Ca, Na, and Mg. 21. What is the physiologic role of magnesium? Where is magnesium stored in the body? o Magnesium functions as a cofactor for more than 300 enzymes including those important in glycolosis, transcellular ion transport, neuromuscular transmission, synthesis of carbohydrates and many others. o Magnesium is stored in the bone (53%) and the rest (46%) is in the muscle, soft tissue and other organs. Page 34
  • 35. 22. Discuss the following as they pertain to magnesium: o alcoholism- people who are alcoholics tend to have diets deficient in magnesium or have problems with malabsorption (hypomagnesemia). o malabsorption- this causes a decrease in magnesium because it is not absorbed (hypomagnesemia). o magnesium sulfate therapy- this is given parenterally to severely ill patients. o secondary hypoparathyroidism- this may cause an increased renal excretion of magnesium due to an excess of calcium ions. 23. What is the physiologic function of iron? How is iron transported in the body? How is iron stored in the body? • Iron functions as part of heme in hemoglobin. It is transported by transferrin and stored in the body as ferratin and hemosidrin. 24. Describe iron levels, %Saturation, TIBC, and ferratin levels in the following disease states: • iron deficiency anemia-the % saturation is decreased, TIBC is increased, and ferratin is decreased. • anemia of chronic infection- the % saturation is normal, TIBC is decreased, and ferratin is normal to increased. • hemochromatosis- the % saturation is increased, TIBC is increased, and ferratin is increased. 25. What is the physiologic function of copper? What is Wilson's disease? ceruloplasmin? • Copper functions as an enzyme indicator, it acts on ferroxidase, and it acts it is a component of enzymes or proteins involved in redox reactions. • Wilson’s disease- also known as hepatolenticular degeneration is a genetically determined defect in ATPase where the copper is transported normally from the intestine into the liver, but cannot be transported from the liver into the bile. 26. What is the physiologic function of zinc? How is it measured? • Zinc functions as a cofactor for more than 300 enzymes. It can be measured by atomic absorption. 27. Define: • total iron binding capacity- (amount of transferrin bound already) • An estimate of serum transferrin levels obtained by measuring the total iron binding capability of a patients serum. Since transferrin represents most of the iron binding capacity of serum TIBC it is generally a good estimate of serum transferrin levels. • % saturation- (transferrin saturation in the patients sample) • % saturation = total iron/TIBC x 100 Page 35
  • 36. • • • • 28. • • • • unsaturated iron binding capacity- (the amount of sites available to bind iron in patients sample) UIBC = iron added – excess iron Serum iron = TIBC – UIBC latent iron binding capacity- (estimate of non-reacting iron bound to transferrin) What is the source of blood ammonia? How is it detoxified? What disease processes are associated with increased ammonia? How is ammonia related to Reye's Syndrome? Blood ammonia arises or comes from the deamination of amina acids through the action of degestive and bacterial enzymes on proteins in the intestinal tract. Ammonia is released from metabolic reactions that occur in skeletal muscles during excercize. Ammonia is detoxified by the liver in the urea cycle where ammonia is converted to urea. Hepatic failure, Reye’s syndrome, and urea cycle enzyme deficiencies are associated with increased ammonia. Reye’s syndrome is usually following a viral infection and it uses ammonia levels to correlate with the severity of the disease and prognosis. 29. What is the purpose of renal synthesis of ammonia? • The kidney synthesizes ammonia to compensate for metabolic acidosis. 30. What are the three ketone bodies. When are they formed? List disease processes associated with increased ketones. o acetone, acetoacetic acid, and beta-hydroxybutyric acid. o These are formed as a product of incomplete fat metabolism, and are associated with diabetes mellitus, starvation, and prolonged vomiting. 31. What is lactic acidosis? When does it occur? o Lactic acidosis- an increase or accumulation of lactic acid in the blood. o This occurs if there is improper oxidation of skeletal muscle and other tissues. 32. What role does hemoglobin play as an important buffer system in the body? What is carbonic anhydrase? What is chloride shift? • Hemoglobin buffers the blood by delivering oxygen to the tissues and then taking the carbon dioxide to the lungs to be exhaled. • Carbonic anhydrase- this enzyme catalyzes the reaction of carbon dioxide to bicarbonate and Hydrogen ion. • The chloride shift- the carbon dioxide goes into the RBC and forms carbonic acid, this acid splits into hydrogen ion and bicarbonate. The bicarbonate leaves the cell and makes it more negative outside the cell, and more positive on the inside because of the hydrogen ion. At this point the chloride shifts into the cell to balance the electorneutrality of the cell. Page 36
  • 37. 33. 34. • What are the main factors which influence the oxygen binding ability of hemoglobin? Specifically how do acidosis and alkalosis affect O2 saturation? • If there is increased oxygen then the hemoglobin binds more oxygen and if there is less oxygen the hemoglobin picks up less oxygen and wants to hold on to it. The ability for hemoglobin to bind oxygen depends mostly on the availability of oxygen. • Acidosis- the pH drops drastically and increases the hemoglobin affinity for O2. • Alkalosis- the pH is increased and it decreases the hemoglobin affinity for O2. Why is heparin the anti-coagulant of choice for pH and blood gas work? How does it work? Heparin is used because it holds the ph constant in blood and prevents the change in gals levels in the sample. 35. Why are blood gas specimens placed in ice immediately after collection? • Blood gas specimens are placed on ice immediately after collection because: • The pH decreases with time if it is not placed on ice immediately after drawn. The lower temperatures prevent the cells from undergoing glycolysis. IX. ENDOCRINOLOGY 36. 37. • Give expected T4, T3 uptake, FTI and TSH levels in the following: What is the function of the thyroid hormones in body metabolism? Describe the following disease processes: The thyroid hormone function is to stimulate the metabolism. Without the thyroid hormone the metabolism will decrease and the patient will experience obesity, mental retardation, edema (water in the tissues), decreased body temperature, and anemia. Page 37
  • 38. • • • • • • • cretinism- condition where the patient has a dysfunctional or no thyroid causing mental retardation and death. Autosomal recessive. juvenile myxedema-A dysfunction of the thyroid after birth that has very severe consequences. All 50 states require a T4 screening test for neonates. adult myxedema- A dysfunction of the thyroid later in life (>30 years) causing hair loss, dry skin, yellow pallor, thick tongue, and arterosclerosos. endemic goiter- an enlarged thyroid due to lack of iodine. hashimoto's disease- the most common form of thyroid disease occurring mostly in women 40to 60 years old. Treat with thyroxine. Autoimmune disease. grave's disease- the patient has and AB to TSH receptor on the thyroid gland. This AB causes the thyroid gland to think it is TSH and start to release T3 and T4 even though there is a decrease in actual TSH. thyroid tumors (goiters)- Tumors that cause the dysfunction of the thyroid gland allowing abnormal amounts of hormone to be released. 38. What are the roles of TSH? What amino acids are necessary for synthesis? • Body temperature stimulates the thyroid hormone which stimulates the hypothalamus and causes it to release TRH to the pituitary gland. The TRH causes the pituitary gland to release TSH to stimulate the thyroid gland which then releases T3 and T4. TheT3 and T4 cause the body temperature to rise and act as the shunt to tell the hypothalamus to stop making the TRH and the pituitary to stop making TSH. 39. What is thyroglobulin? thyroid binding globulin? • Thyroglobulin- an iodine contaning protein secreted by the thyroid gland. • Thyroid binding globulin- A protein that binds thyroid hormones. The TBG assay is used to confirm results of T3 and T4, or abnormalities it the relationship of the TT4 and T3U test. They can also be used as tumor markers for thyroid cancer. 40. Briefly describe each of the following groups synthesized by the adrenal cortex. What molecule is necessary for their synthesis? • glucocorticoids- (cortisol) these stimulate gluconeogenesis to increase glucose, encourage the glycogen production and release from the liver, block epinephrine, act as and anti-inflammatory, inhibit WBC migration, phagocytosis, increase hematopoiesis and stabilize lysozome. • mineralocorticoids- (aldosterone) these regulate potassium metabolism and regulate extracellular fluid volume. They also control water secretion by regulating Na absorption and K secretion. • androgens- (testosterone, dehydroepiandrosterone, dehydroepiandosterone sulfate) These function in spermatogenesis and in the formation of the secondary male sex hormones. • estrogens- (estrogen, estradiol, estriol) these function in ovulation, embryo preparation for implantation, and menstruation regulation Page 38
  • 39. 41. Describe the feedback mechanism for cortisol. What is the function of cortisol? What is the role of ACTH? • The hypothalamus is stimulated by low glucose to secrete CRH which causes the anterior pituitary to secrete ACTH to help make 11-deoxycortisol. Cortisol is formed and is found bound to transcortin or as free cortisol. The free cortisol stimulates he glucose production and when the need is met it acts as a shunt to tell the hypothalamus to stop making CRH. 42. Diagram the renin-angiotensin-aldosterone system. What is the purpose of this system? • The afferent arterioles secrete rennin when there is a decrease in blood pressure. The rennin converts angiotensinogen to Angiotensin I which is converted to Angiotensin II. Angiotensin II stimulates aldosterone to be produced and secreted to increase the blood pressure by retaining sodium to passively hold water and increase the blood volume which increases the blood pressure. 43. What hormones make up the 17-hydroxysteriods? the 17-ketosteroids? Why are these tests performed? • Cortisone, cortisol, and 11-dehydroxycorticoids make up 17-hydroxysteroids. • Androgens make up the 17-keytosetriods. • These tests are performed to observe the levels for glucosteriods, testosterone, and other androgens. 44. What are the catecholamines? Where are they synthesized? What is the precursor amino acid? What is their basic function? Catecholamines are epinephrine, norepinephrine, and dopamine. They are synthesized in the adrenal medulla of the adrenal glands. The precursor amino acid is tyrosine. Their basic function is to increase the blood pressure and heart rate when the body is excited in the fight or flight mechanism. They have the ability to break down adipose tissue and glycogen to be used for energy in the fight or flight mechanism. • 45. • • Describe catecholamine levels in the following diseases: Pheochromocytomas-Tumors of the adrenal medulla or the sympathetic ganglia of the adrenal glands which cause overproduction of the catecholamines. Infantile Neuroblastoma- Tumors of the adrenal medulla that cause an increase in catecholamines. 46. What is the function of prolactin? When is it measured? • Prolactin acts to initiate lactation and maintain it in a pregnant female. It is measured to evaluate pituitary tumors, galactorrhea, and fertility. 47. Describe levels of FSH and LH during follicular development, ovulation and the luteal phase? Why and how are these hormones measured? • FSH increases during follicular phase, goes down and rises again just after ovulation, and drops slowly toward luteal phase. Page 39
  • 40. • • LH is baseline in follicular phase, increases in ovulation, and decreases back to baseline during the luteal phase. These hormones are measured by RIA to assess pituitary and gonadal axis, fertility problems, to investigate puberty problems (late puberty), and to investigate pituitary tumors. 48. What is the function of gastrin? How is it related to Zollinger Ellison Syndrome? • Gastrin functions to secrete HCl to digest food, pepsin, intrinsic factor, pancreatic hormones, and bile from the liver. • Zollinger Ellison (ulcers in the stomach or duodenum) syndrome has increased Gastrin levels, so it can be separated from other peptic ulcers because they don’t increase in production of gastrin. 49. What is the function of the androgens? How are they measured? How is testosterone measured? • Androgens function in spermatogenesis and formation of secondary sex characteristics of males. Androgens like testosterone are measured by RIA. 50. Briefly describe the function of the estrogens and progesterone. What test is used for total urinary estrogens? What is its principle? • Estrogen and progesterone both function in getting the uterus ready for embryo implantation. • The Brown method (a colorimetric method using Kober reagent to look for a pink color indicating that estriol is present) is used to measure total estrogen levels. 51. What is the primary site of synthesis of estradiol, and esterone? Explain why these two estrogens are measured? • Estradiol is produced in the ovary of a pre-menopausal woman. • Esterone is produced in the adrenal cortex of the adrenal glands of a postmenopausal woman. • Both are measured to determine menstrual cycle disorders and ovulation in women. 52. What does the T3 uptake measure? Why is this important? • T3 uptake measures- the # of free binding sites in the Thyroxine Binding Globulin. • This is important because it indirectly gives the amount of thyroxine in the serum sample. 53. What is the free thyroxine index? T7? T12? How are they calculated? What is the importance of this calculation. • Free thyroxine index- estimates how much free T4 is in the blood. • T7 and T12- are pseudonyms for FTI (T7 = T3+ T4, and T12 = T3 x T4) • FTI- an index of thyroid status, it provides T4 and T3U which are useful in diagnosing thyroid problems. • Free thyroxine index = (T4) x (% T3 uptake as a decimal) Page 40
  • 41. 54. Discuss the purpose of each of the following steps in hormone determinations: • hydrolysis- remove and solubilize attachments (chemically or enzymatically) • purification- organic solvents purify the hormones • extraction- centrifugation, washing, and ion exchange • estimation- react, detect, and quantitate by various methods 55. What is diurnal variation? How does this impact cortisol levels and the collection of cortisol specimens? • Diurnal variation- levels of analytes rise and fall, they peak early in the day when most of us are asleep. • Cortisol levels and ACTH (anterior pitutitary hormone, Cortocotropin) • They rise between 0600 and 0800 hours, then decrease all day long. At 2000 hrs (8 pm), the level is 2/3 of what it is at 0800 hrs, so the best analysis is made form early morning specimens. 56. Describe Cushing's Syndrome and Addison's disease. Include their impact on diurnal variation, cortisol levels, glucose levels, aldosterone levels, electrolyte and water balance. How may secondary disease states be diagnosed? • Cushinn’s syndrome- Increased- cortisol, diurnal variation, aldosterone, hypertension, and hyperglycemia (glucose), and decreased potassium. • Addison’s disease- Decreased- cortisol, diurnal variation, aldoserone, hypotension, and hypoglycemia (glucose). Increased- potassium. • Addisons disease- Cosyntropin is given to th epatient which causes the cortisol release, which aids in determining if patient has Addisons disease, because you look to see if cotrisol is released by the cortisol stimulating drug. If still no release of cortisol, it is a primary disease like addisons. • Cushings disease- use hiht dexamethasome suppression test where there is a suppression of urine and plasma cortisol which only occurs in cushings disease. 57. When are plasma epinephrine and norepinephrine levels useful? • Epinephrine and Norepinephrine levels are useful in diagnosing stress, increased catacholamines, decreased blood pressure, decreased blood volume, thyroid hormone deficiency, and congestive heart failure. • Decreased catacholimines are found in hypotension. 58. When is it valuable to quantify HCG? What trimester of pregnancy are HCG levels used to monitor fetal health? • It is valuable to quantitate HCG to indicate how far along in pregnancy a patient is, or if pregnant at all. Fetal health can be determined in the 1st and 3rd trimesters by looking at BHCG levels. 59. What is the function of serotonin? When does it increase? What is 5-hydroxy indolacetic acid? How is 5-HIAA measured? • Serotonin is released during coagulation by platelets and is involved in smooth muscle stimulation and vasoconstriction. It is increased with carcinoid tumors that Page 41
  • 42. occur in the Ileum and appendix, 5-HIAA is a metabolite of serotonin and is excreted in the urine. It is measured colorimetrically after reacting it with 1-nitroso2-napthol and nitrois acid (purple color). 60. Why are estriol levels measured? What trimester of pregnancy are these levels useful in? • Estriol levels are measured to help determine pregnancies. Levels in non pregnant females can be measured in the ug’s, but pregnant women have levels in the mg range. These levels are useful in the third trimester (last 4-6 weeks of pregnancy) M. TOXICOLOGY 1. List methods in which urines are screened for drugs of abuse? Why is urine preferred? What is the purpose of extraction? • You can use thin layer chromatography, gas chromatography, and immunoassays to screen for drugs of abuse. Urine is used most often because the drugs are filtered through the kidneys and show up in the urine. 2. Why are chloramphenicol levels monitored? What disease process may result from chloramphenicol. • Chloramphenicol levels are monitored to make sure that ALA synthase is being produced in enough quantity that heme synthase can be produced because chloramphenicol causes a decrease in ALA synthase, heme synthatase, and DNA synthase. This can be associated with disease processes like sideroblastic anemia. 3. What is the principle of the renish heavy metal test? What disease process is associated with lead poisoning? • The principal of the renisch heavy metal is to place a clean coiled copper wire in a solution of 5-10 ml of gastric acid or urine with an equal amount of 2M HCL, then place in a hot water bath for 10 minutes, let sit one hour, and examine the copper wire for color change. ( blue or purple black- antimony; dull black- arsenic; shiny black- bismuth; and silver gray- mercury) Lead poisioning is associated with encephalopathy characterized by cerebral edema and hypoxia. 4. How is carbon monoxide poisoning usually detected? • This is usually detected by spot tests looking for carboxyhemoglobin (giving the patient a cherry red appearance due to its color), or gas chromatography also looking for carboxyhemoglobin. 5. When does bromide toxicity result? How is it measured? How does bromide affect chloride determinations? • This toxicity results from organic and inorganic medication. It is measured by immunoassay or thin layer chromotography, and it gives a false high in chloride determinations. Page 42
  • 43. 6. What is the therapeutic usage of cyclosporine? methotrexate? • Cyclosporin- this is an immunosuppressive drug that is used to suppress host vs. graft rejection of transplant organs. • Methotrexate- this is an antineoplastic drug that is used in therapy and involves the rate of mitosis in normal cells versus neoplastic cells. 7. What are digoxin? digitoxin? Why are their levels so critical? • Digoxin- one of a group of cardiac glycosides obtained from digitalis plants which restores the force of cardiac contraction in congestive heart failure. (drug used to treat cardiovascular problems) • Digitoxin- another cardiac glycoside (less common than digoxin) that is used to treat cardiovascular problems. • Critical levelso Low- digoxin caused the atrium to be less electrically excitable o Moderate- reduce the rate of depolarization in the spontaneously depolarizing conductive fibers. o High- diminishes the depolarization of the ventricular myocardium. 8. Define: • therapeutic range- concentration range of a drug which is beneficial to the patient without being toxic. • peak level- one hour after the dose is given when the drug reaches peak concentration in the body. • trough level- the lowest concentration of drug obtained in the blood, drawn immediately prior to the next dose. • toxic value- drug levels outside of the therapeutic range. • bioavailability- tge fraction of a drug that is absorbed into the systemic circulation. 9. What is theophyline? When is it used? • A bronchodialator used to treat asthma or other chronic obstructive pulmonary diseases N. VITAMINS 1. What disease process is associated with decreased B12 and folic acid? What is the relationship between B12 and folic acid? • Pernicious anemia is associated with decreases B12 and folic acid. • In relationship between B12 an folic acid, B12 is used in the metabolism and needed for the synthesis of folate which is needed for the production of nucleic acids (DNA) Page 43
  • 44. 2. What is the function of Vitamin A? How is it related to beta-carotene? How are both measured? Why are serum beta carotene levels measured? • Vitamin A functions in growth, dim light vision, reproduction, immunity and mucous secretion. Beta-carotene(pro-vitamin A) is the precursor to vitamin A and is composed of two moles of vitamin A. • They are both measured by immunoassay or HPLC. • Beta carotene is measured in serum to indirectly quantitate 4. (P) Describe the methods for measuring Vitamin B12? What is the function of cyanocobalamin? Page 44
  • 45. BLOOD GASES  Purpose  Represents the acid/base status of entire body  Provides information of lung function  Sample type  Whole Blood  Arterial Sample – ABG  Preferred sample  Sites are radial, femoral or brachial artery  Venous & Capillary Blood  Can be used, but not preferred  Assessment performed STAT
  • 46. SPECIMEN COLLECTION & HANDLING  Collected in heparinized plastic syringe (no air bubbles & no clots!!!)  Often Collected by Respiratory Therapy  Collected anaerobically and put on ice. Ice serves to slow cell metabolism. performed at 37o C, to emulate body temperature  Testing
  • 47. PREANALYTICAL CONSIDERATIONS  Air bubbles Causes increases in pO2, pH  Causes decreased in pCO2   Clots   Can not run clotted whole blood on instrumentation Glycolysis Cell respiration causes a decrease in pH, pO2  pCO2 increases   Temperature  pH is temperature dependent. For every 1 degree rise in temperature, the pH decreases about 0.015 units
  • 48. REFERENCE VALUES (ABG) Component Arterial Blood Mixed Venous Blood pH 7.35-7.45 7.31-7.41 pO2 80-100 mmHg 35-40 mmHg O2 Saturation > 95% 70-75% pCO2 35-45 mmHg 41-51 mmHg HCO3- 22-26 mEq/L 22-26 mEq/L Total CO2 23-27 mmol/L 23-27 mmol/L Base excess -2 to +2 -2 to +2
  • 50. DETERMINATION  Three components are directly measured pH  pO2  pCO2   Values that can be calculated and reported include: Total CO2 or bicarbonate ion  Base excess  Oxygen saturation 
  • 51. PH  MEASUREMENT Measure of the hydrogen ion activity based on bicarbonate-carbonic acid buffer system pH electrode has a thin membrane of glass separating two differing H+ concentrations, a H+ exchange occurs in the outer layers of the glass, causing a potential to develop.  A calomel half-cell or reference electrode is also immersed in the solution.  Both the pH and reference electrode are connected through a pH meter. The meter can measure voltage difference between the two and convert to pH units. 
  • 52. PO2 MEASUREMENT Partial pressure of oxygen in the blood  Measured by the O2 electrode to determine oxygen content  pO2 electrode or Clark electrode measures the current that flows when a constant voltage is applied to the system  As dissolved O2 diffuses from the blood a change in current occurs which offers a direct pO2 measurement 
  • 53. PCO2 MEASUREMENT Partial pressure of carbon dioxide in the blood  pCO2 measured in mmHg x 0.03 indicates carbonic acid (H2CO3)  pCO2 > 50 mmHg = HYPO ventilation  pCO2< 30 mmHg= HYPER ventilation 
  • 54. PCO2 MEASUREMENT The pCO2 electrode or Severinghaus electrode consists of a pH electrode with a CO2 permeable membrane covering the glass surface. Between the two is a thin layer of dilute bicarbonate buffer.  Once the blood contacts the membrane and the CO2 diffuses into the buffer, the pH of the buffer is lowered  Change in pH is proportional to the concentration of dissolved CO2 in the blood 
  • 56. CALCULATED PARAMETERS  Siggaard-Anderson nomogram Base Excess  Total CO2 and bicarbonate concentration 
  • 57. BASE EXCESS Determination of amount of base in the blood  Determines the source of acid-base disturbance  Base deficit usually indicates metabolic acidosis   Causes of: Excess bicarbonate  Deficit of bicarbonate 
  • 58. O2 SATURATION  Calculation/Derived   Requires measured pH and pO2 values Measured  Requires a hgb measurement usually obtained by cooximetry  Co-oximetry: measuring at multiple wavelengths to get light absorption spectra
  • 59. REFERENCES     Bishop, M., Fody, E., & Schoeff, l. (2010). Clinical Chemistry: Techniques, principles, Correlations. Baltimore: Wolters Kluwer Lippincott Williams & Wilkins. Carreiro-Lewandowski, E. (2008). Blood Gas Analysis and Interpretation. Denver, Colorado: Colorado Association for Continuing Medical Laboratory Education, Inc. Jarreau, P. (2005). Clinical Laboratory Science Review (3rd ed.). New Orleans, LA: LSU Health Science Center. Sunheimer, R., & Graves, L. (2010). Clinical Laboratory Chemistry. Upper Saddle River: Pearson . 16
  • 60. ELECTROLYTES  Electrolytes  Substances whose molecules dissociate into ions when they are placed in water.  Osmotically active particles  Classification of ions: by charge  CATIONS (+)    In an electrical field, move toward the cathode Sodium (Na), Potassium (K), Calcium(Ca), Magnesium(Mg) ANIONS (-)   In an electrical field, move toward the anode Chloride(Cl), Bicarbonate, PO4, Sulfate 2
  • 61. ELECTROLYTES  General    dietary requirements Most need to be consumed only in small amounts as utilized Excessive intake leads to increased excretion via kidneys Excessive loss may result in need for corrective therapy  loss due to vomiting / diarrhea; therapy required - IV replacement, Pedilyte, etc. 3
  • 62. ELECTROLYTE FUNCTIONS Volume and osmotic regulation  Myocardial rhythm and contractility  Cofactors in enzyme activation  Regulation of ATPase ion pumps  Acid-base balance  Blood coagulation  Neuromuscular excitability  Production of ATP from glucose  4
  • 63. ELECTROLYTE PANEL  Panel consists of:  sodium (Na)  potassium (K)  chloride (Cl)  bicarbonate CO2 (in its ion form = HCO3- ) 5
  • 64. ANALYTES OF THE ELECTROLYTE PANEL  Sodium    (Na)– the major cation of extracellular fluid Most abundant (90 %) extracellular cation Diet  Easily absorbed from many foods 6
  • 65. FUNCTION: SODIUM    Influence on regulation of body water  Osmotic activity  Sodium determines osmotic activity  Main contributor to plasma osmolality Neuromuscular excitability  extremes in concentration can result in neuromuscular symptoms Na-K ATP-ase Pump  pumps Na out and K into cells  Without this active transport pump, the cells would fill with Na+ and subsequent osmotic pressure would rupture the cells 7
  • 66. REGULATION OF SODIUM   Concentration depends on:  intake of water in response to thirst  excretion of water due to blood volume or osmolality changes Renal regulation of sodium  Kidneys can conserve or excrete Na+ depending on ECF and blood volume  by aldosterone  and the renin-angiotensin system  this system will stimulate the adrenal cortex to secrete aldosterone. 8
  • 67. REFERENCE RANGES: SODIUM  Serum   136-145 mEq/L or mmol/L Urine (24 hour collection)  40-220 mEq/L 9
  • 68. SODIUM  Urine testing & calculation:  Because levels are often increased, a dilution of the urine specimen is usually required.  Once a number is obtained, it is multiplied by the dilution factor and reported as (mEq/L or mmol/L) in 24 hr. 10
  • 69. DISORDERS OF SODIUM HOMEOSTASIS   Hyponatremia: < 136 mmol/L  Causes of:  Increased Na+ loss  Increased water retention  Water imbalance Hypernatremia:> 150 mmol/L  Causes of:  Excess water loss  Increased intake/retention  Decreased water intake 11
  • 70. HYPONATREMIA 1. Increased Na+ loss  Aldosterone deficiency  hypoadrenalism  Diabetes mellitus  In acidosis of diabetes, Na is excreted with ketones  Potassium depletion  K normally excreted , if none, then Na  Loss of gastric contents 12
  • 71. HYPONATREMIA 2. Increased water retention Dilution of plasma Na+ Renal failure Nephrotic syndrome Hepatic cirrhosis Congestive heart failure 13
  • 72. HYPONATREMIA 3. Water imbalance Excess water intake Chronic condition 14
  • 73. SODIUM Note:  Increased lipids or proteins may cause false decrease in results. This would be classified as artifactual/pseudo-hyponatremia 15
  • 74. CLINICAL SYMPTOMS OF HYPONATREMIA  Depends on the serum level  Can affect GI tract  Neurological  Nausea, vomiting, headache, seizures,coma  16
  • 75. HYPERNATREMIA 1. Excess water loss  Sweating  Diarrhea  Burns  Diabetes insipidus 17
  • 76. HYPERNATREMIA Increased intake/retention 2. • Excessive IV therapy Decreased water intake 3. • • • Elderly Infants Mental impairment 18
  • 77. CLINICAL SYMPTOMS OF HYPERNATREMIA  Involve the CNS      Altered mental status Lethargy Irritability Vomiting Nausea 19
  • 78. SPECIMEN COLLECTION: SODIUM Serum (sl hemolysis is OK, but not gross)  Heparinized plasma  Timed and random urine  Sweat  GI fluids  Liquid feces (would be only time of excessive loss)  20
  • 79. ANALYTES OF THE ELECTROLYTE PANEL Potassium  (K+) the major cation of intracellular fluid Only 2 % of potassium is in the plasma  Potassium concentration inside cells is 20 X greater than it is outside.  This is maintained by the Na-K pump  exchanges 3 Na for 1 K   Diet  easily consumed by food products such as bananas 21
  • 80. FUNCTION: POTASSIUM  Critically important to the functions of neuromuscular cells  Acid-base balance  Intracellular fluid volume  Controls heart muscle contraction  Promotes muscular excitability Decreased potassium decreases excitability (paralysis and arrhythmias) 22
  • 81. REGULATION OF POTASSIUM  Kidneys  Responsible for regulation. Potassium is readily excreted, but gets reabsorbed in the proximal tubule under the control of ALDOSTERONE Diet  Cell Uptake/Exchange  23
  • 82. REFERENCE RANGES: POTASSIUM  Serum (adults)   Newborns   3.5 - 5.1 mEq/L or mmol/L 3.7 - 5.9 mEq/L Urine (24 hour collection)  25 - 125 mEq/L 24
  • 83. DISORDERS OF POTASSIUM HOMEOSTASIS   Hypokalemia  < 3.5 mmol/L  Causes of:  Non-renal loss  Renal Loss  Cellular Shift  Decreased intake Hyperkalemia  >5.1 mmol/L  Causes of  Decreased renal excretion  Cellular shift  Increased intake  Artifactual/False elevations 25
  • 84. HYPOKALEMIA 1. Non-renal loss  Excessive fluid loss ( diarrhea, vomiting, diuretics )  Increased Aldosterone promote Na reabsorption … K is excreted in its place 26
  • 85. HYPOKALEMIA 2. Renal Loss  Nephritis, renal tubular acidosis, hyperaldosteronism, Cushing’s Syndrome 3. Cellular Shift  Alkalosis, insulin overdose 4. Decreased intake 27
  • 86. MECHANISM OF HYPOKALEMIA  Increased plasma pH ( decreased Hydrogen ion ) RBC H+ K+ K+ moves into RBCs to preserve electrical balance, causing plasma potassium to decrease. 28 ( Sodium also shows a slight decrease )
  • 87. CLINICAL SYMPTOMS OF HYPOKALEMIA Neuromuscular weakness  Cardiac arrhythmia  Constipation  29
  • 88. HYPERKALEMIA 1. Decreased renal excretion    2. Renal disease Addison’s disease Hypoaldosteronism Cellular Shift   Such as acidosis, chemotherapy, leukemia, muscle/cellular injury Hydrogen ions compete with potassium to get into the cells 30
  • 89. HYPERKALEMIA 3. Increased intake  4. Insulin IVs promote rapid cellular potassium uptake Artifactual • Sample hemolysis • Prolonged tourniquet use • Excessive fist clenching 31
  • 90. CLINICAL SYMPTOMS OF HYPERKALEMIA Muscle weakness  Tingling  Numbness  Mental confusion  Cardiac arrhythmias  Cardiac arrest  32
  • 91. SPECIMEN COLLECTION:POTASSIUM Non-hemolyzed serum  heparinized plasma  24 hr urine  33
  • 92. ANALYTES OF THE ELECTROLYTE PANEL  Chloride  (Cl-) the major anion of extracellular fluid  Chloride moves passively with Na+ or against HCO3- to maintain neutral electrical charge  Chloride usually follows Na  if one is abnormal, so is the other 34
  • 93. FUNCTION: CHLORIDE  Body hydration/water balance  Osmotic pressure  Electrical neutrality 35
  • 94. REGULATION OF CHLORIDE  Regulation via diet and kidneys In the kidney, Cl is reabsorbed in the renal proximal tubules, along with sodium.  Deficiencies of either one limits the reabsorption of the other.  36
  • 95. REFERENCE RANGES: CHLORIDE  Serum  98 -107 mEq/L or mmol/L  24 hour urine  110-250 mEq/L  varies with intake  CSF  120 - 132 mEq/L  Often CSF Cl is decreased when CSF protein is increased, as often occurs in bacterial meningitis. 37
  • 96. DETERMINATION: CHLORIDE  Specimen type      Serum Plasma 24 hour urine CSF Sweat  Sweat Chloride Test  Used to identify cystic fibrosis patients    Increased salt concentration in sweat Pilocarpine= chemical used to stimulate sweat production Iontophoresis= mild electrical current that stimulates sweat production
  • 97. DISORDERS OF CHLORIDE HOMEOSTASIS  Hypochloremia  Decreased blood chloride  Causes of :  Conditions where output exceeds input  Hyperchloremia  Increased blood chloride  Causes of:  Conditions where input exceeds output 39
  • 98. HYPOCHLOREMIA  Decreased serum Cl loss of gastric HCl  salt loosing renal diseases  metabolic alkalosis/compensated respiratory acidosis   increased HCO3- & decreased Cl- 40
  • 99. HYPERCHLOREMIA  Increased serum Cl      dehydration (relative increase) excessive intake (IV) congestive heart failure renal tubular disease metabolic acidosis  decreased HCO3- & increased Cl- 41
  • 100. SPECIMEN COLLECTION: CHLORIDE Serum  Heparinized plasma  24 hr urine  Sweat  42
  • 101. ANALYTES OF THE ELECTROLYTE PANEL  Carbon dioxide/bicarbonate (HCO3-)  the major anion of intracellular fluid  2nd most abundant anion of extracellular fluid  Total plasma CO2= HCO3- + H2CO3- + CO2  HCO3- (bicarbonate ion)  accounts for 90% of total plasma CO2  H2CO3- (carbonic acid) 43
  • 102. FUNCTION: BICARBONATE ION  CO2 is a waste product  continuously produced as a result of cell metabolism,  the ability of the bicarbonate ion to accept a hydrogen ion makes it an efficient and effective means of buffering body pH  dominant buffering system of plasma 44
  • 103. REGULATION OF BICARBONATE ION  Bicarbonate is regulated by secretion / reabsorption of the renal tubules  Acidosis : ↓ renal excretion  Alkalosis : ↑ renal excretion 45
  • 104. REGULATION OF BICARBONATE ION  Kidney regulation requires the enzyme carbonic anhydrase which is present in renal tubular cells & RBCs Reaction: CO2 + H2O ⇋ H2CO3 → H+ + HCO–3 carbonic anhydrase Pulmonary Control Renal Control 46
  • 105. REFERENCE RANGE: BICARBONATE ION  Total Carbon dioxide (venous)  23-29 mEq/L or mmol/L   includes bicarb, dissolved & undissociated H2CO3 - carbonic acid (bicarbonate) Bicarbonate ion (HCO3–)  22-26 mEq/L or mmol/L 47
  • 106. SPECIMEN COLLECTION: BICARBONATE ION heparinized plasma  arterial whole blood  fresh serum  Anaerobic collection preferred  48
  • 107. ELECTROLYTE BALANCE Anion gap – an estimate of the unmeasured anion concentrations such as sulfate, phosphate, and various organic acids.  49
  • 108. ELECTROLYTE SUMMARY  cations (+) Na 142  K 5  Ca 5  Mg 2 154 mEq/L   anions (-) Cl 105  HCO324  HPO422  SO4-2 1  organic acids 6  proteins 16  154 mEq/L 50
  • 109. ANION GAP  Anion Gap Calculations 1. Na - (Cl + CO2 or HCO3-)  Reference range: 7-16 mEq/L Or 2. (Na + K) - (Cl + CO2 or HCO3-)  Reference range: 10-20 mEq/L 51
  • 110. FUNCTIONS OF THE ANION GAP  Causes in normal patients  what causes the anion gap?   Increased AG –      2/3 plasma proteins & 1/3 phosphate& sulfate ions, along with organic acids uncontrolled diabetes (due to lactic & keto acids) severe renal disorders Hypernatremia lab error Decreased AG 52  a decrease AG is rare, more often it occurs when one test/instrument error
  • 111. REFERENCES     Bishop, M., Fody, E., & Schoeff, l. (2010). Clinical Chemistry: Techniques, principles, Correlations. Baltimore: Wolters Kluwer Lippincott Williams & Wilkins. http://thejunction.net/2009/04/11/the-how-to-authority-fordonating-blood-plasma/ http://www.nlm.nih.gov/medlineplus/ency/article/002350.htm Sunheimer, R., & Graves, L. (2010). Clinical Laboratory Chemistry. Upper Saddle River: Pearson . 53